Sei sulla pagina 1di 86

PIPE PROBLEMS F = 0.138 (9)0.5 = 0.

414 m2

Find the work posses for a Helium gas at 200C A 29.53” x 39.37” pressure vessel contains ammonia with f = 0.041. Compute the minimum
required discharge capacity of the relief device in kg/hr.
*A. 609 KJ/kg B. 168 KJ/kg C. 229 KJ/kg D. 339 KJ/kg A. 106.71 kg/hr B. 108.71 kg/hr *C. 110.71 kg/hr D. 112.71 kg/hr
Solution:
Solution: C = f D L, kg/s
W = m R T = m (8.314 / M) T C = 0.041(29.53/39.37)(39.37/39.37) = 0.03075 kg/s (3600) = 110.71 kg/hr
For helium, M = 4
W/m = (8.314/4)(20 + 273) = 609 KJ/kg Compute the maximum length of the discharge pipe installed on the outlet of a pressure-relief
device in feet for internal pipe diameter of 0.5 inch and rated discharge capacity is 8 lb/min of
Two kilogram of gas is confined in a 1 m3 tank at 200 kpa and 880C. What type of gas is in air. The rated pressure of relief valve is 16 psig.
the tank? *A. 0.286 ft B. 0.386 ft C. 0.486 ft D. 0.586 ft
A. Helium *B. Ethane C. Methane D. Ethene Solution:
Solution: P = Pg + Patm = 16 x 1.1 + 14.7 = 32.3 psia
PV=mRT L = 9P2d5/16Cr2 = 9(32.3)2(0.5)5/16(8)2 = 0.286 ft
200 (1) = 2 (8.314/M)(88+273)
M = 30 A thermal power plant has a heat rate of 11,363 Btu/kw-hr. Find the thermal efficiency of the
Therefore: the gas is Ethane (C2 H6) plant.
A. 28% *B. 30% C. 34% D. 40%
Find the enthalpy of Helium if its internal energy is 200 KJ/kg Solution:
A.144 KJ/kg B. 223.42 KJ/kg *C. 333.42 KJ/kg D. 168 KJ/kg eth = 3412/Heat rate = 3412/11,363 = 30%
Solution:
R = 8.314/4 = 2.0785 What is the hydraulic gradient of a 1 mile, 17 inches inside diameter pipe when 3300 gal/min
K = 1.667 for helium of water flow with f = 0.03.
Cp = k R/(k - 1) = 1.667(2.0785)/(1.667 – 1) = 5.195 KJ/kg-K *A. 0.00714 B. 0.00614 C. 0.00234 D. 0.0018
Cv = R/(k – 1) = 2.0785/(1.667 – 1) = 3.116 KJ/kg – K Solution:
∆h/∆U = Cp/Cv v = (3300/7.481)/(π/4)(17/12)2(60) = 4.66 ft/s
∆h/200 = 5.195/3.116 L = 1 mile = 5280 ft
∆h = 333.42 KJ/kg hL = fLv2/2_D = 0.03(5280)(4.66)2/2(32.2)(17/12) = 37.7 ft
Hydraulic gradient = 37.7/5280 = 0.007.14
Compute the mass of a 2 m3 propane at 280 kpa and 40˚C.
A. 6.47 kg B. 5.1 kg C. 10.20 kg *D. 9.47 kg Find the loss of head in the pipe entrance if speed of flow is 10 m/s.
Solution: A. 5.10 m B. 10.2 m C. 17.4 m *D. 2.55 m
Propane is C3 H3--------------M = 12(3) + 8(1) = 44 Solution:
PV = m R T Loss at entrance = 0.5 (v2/2g) = 0.5 [102 / 2(9.81)] = 2.55 m
280(2) = m (8.314/44)(40 + 273)
m = 9.47 kg Wet material, containing 220% moisture (dry basis) is to be dried at the rate of 1.5 kg/s in a
continuous dryer to give a product containing 10% (dry basis) . Find the moisture removed,
Compute the air flow in ft3/min of mechanical ventilation required to exhaust an accumulation kg/hr
of refrigerant due to leaks of the system capable of revolving air from the machinery room for *A. 3543.75 kg/hr B. 3513.75 kg/hr C. 3563.75 kg/hr D. 3593.75 kg/hr
a mass of 4 lbs refrigerant. Solution:
*A. 200 B. 210 C. 220 D. 230 Solid in wet feed = solid in dried product
Solution: [1/(1 + 2.2)](1.5) = [1/(1 + 0.1)](x)
Q = 100 x G0.5 ft3/min x = 0.5156 kg/s (total dried product)
Q = 100 x (4)0.5 = 200 ft3/min Moisture removed = 1.5 – 0.5156 = 0.984 kg/s = 3543.75 kg/hr

Compute the free-aperture cross section in m2 for the ventilation of a machinery room if the Copra enters a dryer containing 70% moisture and leaves at 7% moisture. Find the moisture
mass of refrigerant is 9 kg. removed on each pound on solid in final product.
A. 0.314 *B. 0.414 C. 0.514 D. 0.614 A. 6.258 lb B. 1.258 lb C. 4.258 lb *D. 2.258 lb
Solution: Solution:
F = 0.138 G0.5 m2 Solid in wet feed = solid in dried product
0.3x = 1 At 100˚C
x = 3.333 lbs ht = cp t = 4.187 (100) = 418.7 KJ/kg
1 = 0.93y htg = 2257 KJ/kg
y = 1.07527 lb h2 = h1 + x htg = 418.7 + 0.60(2257) = 1,772.9 KJ/kg
Moisture removed = x – y = 3.333 – 1.07527 = 2.258 lb Q = 1(4.187)(100 – 30) + 1(1772.9 – 418.7) = 1,647.29 KJ/kg

Find the enthalpy of water at 212˚F and 14.7 psi if the dryness factor is 30%. Use the
A 1 m x 1.5 m cylindrical tank is full of oil with SG = 0.92. Find the force acting at the bottom approximate enthalpy formula of liquid.
of the tank in dynes. A. 461 Btu/lb *B. 471 Btu/lb C. 481 Btu/lb D. 491 Btu/lb
A. 106.33 x 103 dynes B. 106.33 x 104 dynes C. 106.33 x 105 dynes *D. 106.33 x Solution:
106 dynes ht = (˚F – 32) = (212 – 32) = 180 Btu/lb
Solution: htg = 970 Btu/lb
P = w h = (0.92 x 9.81) (1.5) = 13.5378 kpa h = ht + x htg
F = PA = 13.5378(π/4 x 12) = 10.632 KN = 10,632.56 N x 10,000 dynes/N h = 180 + 0.3(970) = 471 Btu/lb
F = 106.33 x 106 dynes
An air compressor consumed 1200 kw-hr per day of energy. The electric motor driving the
Find the pressure at the 100 fathom depth of water in kpag. compressor has an efficiency of 80%. If indicated power of the compressor is 34 kw, find the
*A. 1,793.96 kpag B. 1,893.96 kpag C. 1,993.96 kpag D. 1,693.96 mechanical efficiency of the compressor.
kpag A. 117.65 % B. 75 % *C. 85 % D. 90 %
Solution: Solution:
H = 100 fathom x 6 = 600 ft P/m = 1200kw-hr/24 hrs = 50 kw
P = w h = (600/3.281)(9.81) = 1,793.96 kpag BP = 50(0.80) = 40 kw
em = 34/40 = 85 %
Find the depth in furlong of the ocean (SG = 1.03) if the pressure at the sea bed is 2,032.56
kpag. A refrigeration system consumed 28,000 kw-hr per month of energy. There are 20 % of energy
*A. 1 B. 2 C. 3 D. 4 is lost due to cooling system of compressor and motor efficiency is 90 %. If COP of the
Solution: system is 6, find the tons of refrigeration of the system.
P=wh A. 43.15 TR B. 46.15 TR *C. 49.15 TR D. 41.15 TR
2,032.56 = (1.03 x 9.81) h Solution:
h = 201.158 m x 3.281 ft/m x 1 yd/3ft x 1 furlong/220yd = 1 furlong P/m = 28,800/(24 x 30) = 40 kw
BP = 40(0.90) = 36 kw
Find the mass of 10 quartz of water. Wc = 36(1 – 0.20) = 28.80 kw
A. 10.46 kg *B. 9.46 kg C. 11.46 kg D. 8.46 kg COP = RE/Wc
Solution: 6 = RE/28.80
V = 10 quartz x 1gal/4quartz x 3.785li/1gal x 1m3/1000li RE = 172.8/3.516 = 49.15 TR
V = 0.0094625 x 10-3m3
w = m/V A 23 tons refrigeration system has a heat rejected of 100 kw. Find the energy efficiency ratio
1000 = m/0.0094625 x 10-3 of the system.
m = 9.46 kg A. 13.42 *B. 14.42 C. 15.42 D. 16.42
Solution:
Find the mass of carbon dioxide having a pressure of 20 psia at 200˚F with 10 ft3 volume. QR = RE + Wc
A. 1.04 lbs B. 1.14 lbs *C. 1.24 lbs D. 1.34 lbs 100 = 23(3.516) + Wc
Solution: Wc = 19.132 kw
PV = m R T COP = RE/Wc = (23 x 3.516) / 19.132 = 4.32
(20 x 144)(10) = m (1545/44)(200 + 460) EER = 3.412 COP = 3.412(4.23) = 14.42
m = 1.24 lbs
A 200 mm x 250 mm, 8-cylinder, 4-stroke diesel engine has a brake power of 150 kw. The
Find the heat needed to raise the temperature of water from 30˚C to 100˚C with 60% quality. mechanical efficiency is 80%. If two of the cylinders were accidentally cut off, what will be
Consider an atmospheric pressure of 101.325 kpa. Use the approximate enthalpy formula of the new friction power?
liquid. A. 31.50 kw B. 33.50 kw C. 35.50 kw *D. 37.50 kw
A. 293.09 KJ/kg B. 1,772.90 KJ/kg C. 1,547.90 KJ/kg *D. 1,647.29 KJ/kg Solution:
Solution: em = BP/IP
0.8 = 150/IP
IP = 187.5 kw Compute the mass of a 2 m3 propane at 280 kpa and 40˚C.
FP1 = IP – BP = 187.5 – 150 = 37.50 kw A. 6.47 kg B. 5.1 kg C. 10.20 kg *D. 9.47 kg
FP1 = FP2 = 37.50 kw Solution:
Propane is C3H6------------------------M = 12(3) + 8(1) = 44
If the energy efficiency ratio of the refrigeration system is 12.6, what is the COP of the PV = m R T
system? 280(2) = m (8.314/44)(40 + 273)
*A. 3.69 B. 4.23 C. 5.92 D. 6.83 m = 9.47 kg
Solution:
EER = 3.412 COP Compute the air flow in ft3/min of mechanical ventilation required to exhaust an accumulation
12.6 = 3.412 COP of refrigerant due to leaks of the system capable of revolving air from the machinery room for
COP = 3.69 a mass of 4 lbs refrigerant.
*A. 200 B. 210 C. 220 D. 230
An air compressor has a power of 40 kw at 4 % clearance. If clearance will increase to 70 %, Solution:
what is the new power? Q = 100 x G0.5 ft3/min
A. 70 kw *B. 40 kw C. 53kw D. 60 kw Q = 100 x (4)0.5 = 200 ft3/min
Solution:
The power of compressor will not be affected with the changes in clearance. Compute the free-aperture cross section in m2 for the ventilation of a machinery room if the
Therefore the power will still be 40 kw. mass of refrigerant is 9 kg.
A. 0.314 *B. 0.414 C. 0.514 D. 0.613
What is the approximate value of temperature of water having enthalpy of 208 Btu/lb? Solution:
A. 138.67 ˚C *B. 115.55 ˚C C. 258.67 ˚C D. 68.67 ˚C F = 0.138 G0.5 , m2
Solution: F = 0.138(9)0.5 = 0.414 m2
h = ˚F – 32
A 29.53” x 39.37” pressure vessel contains ammonia with f = 0.041. Compute the minimum
Find the work posses for a Helium gas at 20°C. required discharge capacity of the relief device in kg/hr.
*A. 609 KJ/Kg B. 168 KJ/Kg C. 229 KJ/Kg D. 339 KJ/Kg A. 106.71 kg/hr B. 108.71 kg/hr *C. 110.71 kg/hr D. 112.71
Solution: kg/hr
W = m R T = m (8.314/M) T Solution:
For Helium, M = 4 C = f D L, kg/s
W/m = (8.314/4)(20 + 273) = 609 KJ/Kg C = 0.041(29.53/39.37)(39.37/39.37) = 0.03075 kg/s (3600) = 110.71 kg/hr

Two kilogram of gas is confined in a 1 m3 tank at 200 kpa and 88˚C. What type of gas is in the Compute the maximum length of the discharge pipe installed on the outlet of a pressure-relief
tank? device in feet for internal pipe diameter of 0.5 inch and rated discharge capacity is 8lb/min of
A. Helium *B. Ethane C. Methane D. Ethene air. The rated pressure of relief valve is 16 psig.
Solution: *A. 0.286 ft B. 0.386 ft C. 0.486 ft D. 0.586 ft
PV=mRT Solution:
200 (1) = 2 (8.314/m)(88 + 273) P = Pg + Patm = 16 x 1.1 + 14.7 = 32.3 psia
M = 30 L = 9P2d5/16Cr2 = 9(32.3)2(0.5)5/16(8)2 = 0.286 ft
Therefore: the gas is Ethane (C2H6)
A thermal power plant has a heat rate of 11,363 Btu/kw-hr. Find the thermal efficiency of the
Find the enthalpy of Helium if its internal energy is 200 KJ/kg. plant.
A. 144 KJ/kg B. 223.42 KJ/kg *C. 333.42 KJ/kg A. 28 % *B. 30 % C. 34 % D. 40 %
D. 168 KJ/kg Solution:
Solution: eth = 3412/heat rate = 3412/11,363 = 30%
R = 8.314/4 = 2.0785
K = 1.667 for helium What is the hydraulic gradient of a 1 mile, 17 inches inside diameter pipe when 3300 gal/min
cp = k R/(k – 1) = 1.667(2.0785)/(1.667 – 1) = 5.195 KJ/kg-K of water flow with f…. 0.03
cv = R/(k – 1) = 2.0785/(1.667 – 1) = 3.116 KJ/kg-K *A. 0.00714 B. 0.00614 C. 0.00234 D. 0.00187
∆h/∆U = cp/cv Solution:
∆h/200 = 5.195/3.116 v = (3300/7.481)/(π/4)(17/12)2(60) = 4.66 ft/s
∆h = 333.42 KJ/kg L = 1 mile = 5280 ft
hL =fLv2/2gD = 0.03(5280)(4.66)2/2(32.2)(17/12) = 37.7 ft
Hydraulic gradient = 37.7/5280 = 0.00714 Find the mass of 10 quartz of water.
A. 10.46 kg *B. 9.46 kg C. 11.46 kg
Find the loss of head in the pipe entrance if speed of flow is 10 m/s. D. 8.46 kg
A. 5.10 m B. 10.2 m C. 17.4 m *D. 2.55 m Solution:
Solution: V = 10 quartz x 1gal/4quartz x 3,785 li/1gal x 1m3/1000 li
Loss at entrance = 0.5 (v2/2g) = 0.5[102/ 2(9.81)] = 2.55 m V = 0.0094625 x 10-3m3
w = m/V
Wet material, containing 220 % moisture (dry basis) is to be dried at the rate of 1.5 kg/s in a 1000 = m/0.0094625 x 10-3
continuous dryer to give a product containing 10% (dry basis). Find the moisture removed, m = 9.46 kg
kg/hr.
*A. 3543.75 kg/hr B. 3513.75 kg/hr C. 3563.75 kg/hr D. 3593.75 kg/hr Find the mass of carbon dioxide having a pressure of 20 psia at 200°F with 10 ft3 volume.
Solution: A. 1.04 lbs B. 1.14 lbs *C. 1.24 lbs D. 1.34 lbs
Solid in wet feed = solid in dried product Solution:
[1/(1 + 2.2)](1.5) = [1/(1 + 0.1)](x) PV = m R T
x = 0.5156 kg/s (total dried product) (20 x 144)(10) = m (1545/44)(200 + 460)
Moisture removed = 1.5 – 0.5156 = 0.984 kg/s = 3543.75 kg/hr m = 1.24 lbs

Copra enters a dryer containing 70% moisture and leaves at 7% moisture. Find the moisture
removed on each pound of solid in final product. Find the heat needed to raise the temperature of water from 30°C to 100°C with 60% quality.
A. 6.258 lb B. 1.258 lb C. 4.258 lb *D. 2.258 lb Consider and atmospheric pressure of 101.325 kpa. Use the approximate enthalpy formula of
Solution: liquid.
Solid in wet feed = solid in dried product A. 293.09 KJ/kg B. 1,772.90 KJ/kg C. 1,547.90 KJ/kg
0.3x = 1 *D. 1,647.29 KJ/kg
x = 3.333 lbs Solution:
1 = 0.93y At 100°C
y = 1.07527 lb hf = Cp t = 4.187(100) 418.7 KJ/kg
Moisture removed = x – y = 3.333 – 1.07527 = 2.258 lbs hfg = 2257 KJ.kg
h2 = hf + xhfg = 418.7 + 0.60(2257) = 1,772.9 KJ/kg
A 1 m x 1.5 m cylindrical tank is full of oil with SG = 0.92. Find the force acting at the Q = 1(4.187)(100-30) + 1(1772.9 – 418.7) = 1,647.20 KJ/kg
bottom of the tank in dynes.
A. 106. 33 x 103 dynes B. 106.33 x 104 dynes C. 106.33 x 105 dynes *D. Find the enthalpy of water at 212˚F and 14.7 psi if the dryness factor is 30%. Use the
106.33 x 106 dynes approximate enthalpy formula of liquid.
Solution: A. 461 Btu/lb *B. 471 Btu/lb C. 481 Btu/lb D. 491
P = w h = (0.92 x 9.81)(1.5) = 13.5378 kpa Btu/lb
F = PA = 13.5378(π/4 x 12) = 10.632 KN = 10,632.56 N x 10,000 dynes/N Solution:
F = 106.33 x 106 dynes hf = (˚F – 32) = (212 – 32) = 180 Btu/lb
hfg = 970 Btu/lb
Find the pressure at the 100 fathom depth of water in kpag. h = hf + x hfg
*A. 1,793.96 kpag B. 1,893.96 kpag C. 1,993.96 kpag h = 180 + 0.3(970) = 471 Btu/lb
D. 1,693.96 kpag
Solution: An air compressor consumed 1200 kw-hr per day of energy. The electric motor driving the
H = 100 fathom x 6 = 600 ft compressor has an efficiency of 80 %. If indicated power of the compressor is 34 kw, find the
P = w h = (600/3.281)(9.81) = 1,793.96 kpag mechanical efficiency of the compressor.
A. 117.65 % B. 75% *C. 85% D. 90%
Find the depth in furlong of the ocean (SG = 1.03) if the pressure at the sea bed is 2,032.56 Solution:
kpag. Pim = 1200kw-hr/24 hrs = 50 kw
*A. 1 B. 2 C. 3 D. 4 BP = 50(0.80) = 40 kw
Solution: em = 34/40 = 85%
P=wh
2,032.56 = (1.03 x 9.81) h
h = 201.158 m x 3.281 ft/m x 1 yd/3ft x 1 furlong/220yd = 1 furlong
A refrigeration system consumed 28,800 kw-hr per month of energy. There are 20 % of energy 208 = F – 32
is lost due to cooling system of compressor and motor efficiency is 90%. If COP of the system ˚F = 240 ˚F = 115.55 ˚C
is 6, find the tons of refrigeration of the system.
A. 43.15 TR B. 46.15 TR *C. 49.15 TR D. 41.15 TR Convert 750˚R to ˚K
Solution: A. 390.33 ˚K B. 395.33 ˚K C. 410.33 ˚K *D. 416.33 ˚K
Pim = 28,800/(24 x 30) = 40 kw Solution:
BP = 40 (0.90) = 36 kw ˚R = 1.8 ˚K
Wc = 36(1 – 0.20) = 28.80 kw 750 = 1.8 ˚K
COP = RE/Wc ˚K = 416.667
6 = RE/28.80
RE = 172.8/3.516 = 49.15 TR An otto cycle has a compression ratio of 8. Find the pressure ratio during compression.
*A. 18.38 B. 16.38 C. 14.38 D. 12.38
A 23 tons refrigeration system has a heat rejected of 100 kw. Find the energy efficiency ratio Solution:
of the system. P1V1k = P2V22
A. 13.42 *B. 14.42 C. 15.42 D.16.42 (V1/V2)k = (P2/P1)
Solution: rkk = rp
QR = RE + Wc rp = (8)1.4 = 18.38
100 = 23(3.516) + Wc
Wc = 19.132 kw A diesel cycle has a cut off ratio of 2.5 and expansion ratio of 4. Find the clearance of the
COP = RE/Wc = (23 x 3.516) / 19.132 = 4.23 cycle.
EER = 3.412 COP = 3.412(4.23) = 14.42 A. 9.11 % B. 5.55 % *C. 11.11 % D.
15.15 %
A 200 mm x 250 mm, 8-cylinder, 4-stroke diesel engine has a brake power of 150 kw. The Solution:
mechanical efficiency is 80 %. If two of the cylinders were accidentally cut off, what will be rk = rc re
the new friction power? rk = 2.5(4) = 10
A. 31.50 kw B. 33.50 kw C. 35.50 kw *D. 37.50 kw rk = (1 + c)/c
Solution: 10 = (1 + c)/c
em = BP/IP c = 11.11 %
0.8 = 150/IP
IP = 187.5 kw A dual cycle has an initial temperature of 30 ˚C. The compression ratio is 6 and the heat
FP1 = IP – BP = 187.5 – 150 = 37.50 kw addition at constant volume process is 600 KJ/kg. If cut-off ratio is 2.5, find the maximum
FP1 = FP2 = 37.50 kw temperature of the cycle.
A. 3638.50 ˚C *B. 3365.50 ˚C C. 3565.50 ˚C D. 3965.50 ˚C
If the energy efficiency ratio of the refrigeration system is 12.6, what is the COP of the Solution:
system? T2 = T1 rkk-1 = (30 +273)(6)1.4-1 = 620.44 ˚K
*A. 3.69 B. 4.23 C. 5.92 D. 60 kw QAV = m cv (T3 – T2)
Solution: 600 = 1(0.7186)(T3 – 620.44)
EER = 3.412 COP T3 = 1455.396 ˚K
12.6 = 3.412 COP rc = T4/T3
COP = 3.69 2.5 = T4/1455.396
T4 = 3638.49 ˚K = 3365.50 ˚C
An air compressor has a power of 40 kw at 4% clearance. If clearance will increase to 7%,
what is the new power? A three stages air compressor compresses air from 100 kpa to 1000 kpa. Find the intercooler
A. 70 kw *B. 40 kw C. 53 kw D. 60 kw pressure between the first and second stage.
Solution: A. 505.44 kpa B. 108.44 kpa C. 316.23 kpa *D. 215.44 kpa
The power of compressor will not be affected with the changes in clearance. Solution:
Therefore power will still be 40 kw. Px = (P12P2)1/3
Px = [(100)2(1000)]1/3 = 215.44 kpa
What is the approximate value of temperature of water having enthalpy of 208 Btu/lb?
A. 138.67˚C *B. 115.67˚C C. 258.67˚C D. 68.67˚C A 10-stages air compressor compresses air from 100 kpa to 800 kpa. Find the intercooler
Solution: pressure between 1st and 2nd stage.
h = ˚F – 32 A. 282.84 kpa B. 113.21 kpa *C. 123.11 kpa D. 333.51 kpa
Solution: A. 1273.29 B. 2173.29 *C. 1373.60 D. 7231.29
Px = (P1s-1P2)1/s SOLUTION:
Px = [(100)10-1(1000)]1/10 = 123.11 kpa h2 = hf + xhfg = 151.53 + 0.85(2415.9) = 2205.045 KJ/ kg
KE1 = ½ m v2 = ½ (5,000/3600)(30)2 = 625 W = 0.625 KW
A 3-stages air compressor compresses air from 100 kpa to 700 kpa. Find the intercooler KE2 = ½ m v2 = ½ (5,000/3600)(15)2 = 156.25 W = 0.15625 KW
pressure between the 2nd and 3rd stage. By energy balance:
*A. 365.88 kpa B. 375.88 kpa C. 385.88 kpa D. KE1 + mh1 = KE2 + mh2 + Q + W
395.88 kpa W = (KE1 – KE2) + m(h1-h2) – Q
Solution: 5000 10,000
Px = (P12P2)1/3 W = (0.625 – 0.156) + (3600)(3195.7 – 2205.045) – 3600 = 1373.60 KW
Px = [(100)2(700)]1/3 = 191.28 kpa
Px/P1 = Py/Px A steam turbine with 85% stage efficiency receives steam at 7 Mpa and 550oC and exhausts as
Py = Px2/P1 = (191.28)2/100 = 365.88 kpa 20 Kpa. Determine the turbine work.
At 7 Mpa and 550oC: h1 = 3530.9 Kj/kg s1 = 6.9486
Carnot cycle A, B and C are connected in series so that the heat rejected from A will be the At 20 Kpa (0.020 Mpa): sf = 0.8320 hf = 251.4 sfg = 7.0766 hfg =
heat added to B and heat rejected from B will be added to C, each cycle operates between 30 2358.3
˚C and 400 ˚C. If heat added to A is 1000 kw, find the work output of C. A. 1,117 KJ/kg B. 1,132 KJ/ kg C. 1,123.34 KJ/ kg *D. 1,054.95
*A. 111.44 kw B. 549.78 kw C. 247.53 kw D. KJ/kg
141.89 kw SOLUTION:
Solution: s1 = s2 = sf + xsfg
e1 = e2 = e3 = (400 – 30)/(400 + 273) = 54.98 % 6.9486 = 0.8320 + x(7.0766)
e1 = W1/QA1 = (QA1 – QR1)/QA1 x = 0.8643
0.5498 = (1000 – QR1)/1000 h2 = 251.40 + 0.8643(2358.3) = 2289.78 KJ/kg
QR1 = 450.22 = QA2 h1 - h2a
0.5498 = (450.22 – QR2)/450.22 eST = h1 – h2
By heat balance: 0.85 =3530.9 – h2a
Qgain = Qloss 3530.9 – 2289.78
mw cp (tb- ta) = mg cpg (t2-t1) h2a = 2475.95 KJ/kg
(0.30)(4.187)( tb – 15) = (0.5)(1.0717)(150 – 80) WT = h1 = h2a = 3530.9 – 2475.95 = 1,054.95 KJ/kg
tb = 44.86 oC
A steam turbine with 80% stage efficiency receives steam at 7 Mpa and 550oC and exhaust as
A 350 mm X 450 mm steam engine running at 280 rpm has an entrance steam condition of 2 20 Kpa. Determine the quality at exhaust.
Mpa and 230 oC and exit at 0.1 Mpa. The steam consumption is 2,000 kg/hr and mechanical At 7 Mpa and 550oC: h1 = 3530.9 Kj/kg s1 = 6.9486
efficiency is 85%. If indicated mean effective pressure is 600 Kpa, determine brake thermal At 20 Kpa (0.020 Mpa): sf = 0.8320 hf = 251.4
efficiency. *A. 96.96% B. 76.34% C. 82.34% D. 91.69%
At 2 Mpa and 230 oC (Table 3): h1 = 2849.6 s1 = 6. 4423 SOLUTION:
At 0.1 Mpa: sf = 1.3026 hf = 417.46 sfg = 6.0568 hfg = 2258 hf2 = 417.46 sfg = 7.0766 hfg = 2358.3
KJ/kg s1 = s2 = sf + sfg
A. 23.34% *B. 15.25% C. 14.16% D. 27.34% 6.9486 = 0.8320 + x(7.0766)
x = 0.8643
SOLUTION: h2 = 251.40 + 0.8643(2358.3) = 2289.78 KJ/kg
VD = 2[3.1416/4 (0.35) 2 (0.45)(280/60)]= 0.4041 m3/sec nST = h1 – h2a
Indicated Power = Pmi x VD = 600 x 0.4041 =242.45 KW h1 – h2
Brake Power = IP (em) = 242.45 (0.85) = 206.08KW 0.80 = 3530.9 – h2a
Brake Power 206.08 3530.9 – 2289.78
etb = BP/ ms (h1-hf2) =206.08/ (2000/3600)(2849.6 – 417.46)=15.25% h2a = 2538.004 KJ/kg
h2a = hf + x hfg
A steam turbine receives 5,000 kg/hr of steam at 5 Mpa and 4000oc and velocity of 30 m/sec. 2538.004 = 251.40 + x (2358.3)
It leaves the turbine at 0.06 Mpa and 85% quality and velocity of 15 m/sec. Radiation loss is x = 96.96%
10,000 KJ/hr. Find he KW developed.
At 5 Mpa and 400oC: h1 = 3195.7 KJ/kg s1 =6.6459 A 16,000KW geothermal plant has a generator efficiency and turbine efficiency of 90% and
At 0.006 Mpa: hf = 151.53 hfg = 2415.9 80%., respectively if the quality after throttling is 20% and each well discharges 400, 000
kg/hr, determine the number of wells are required to produce if the charge of enthalpy if the The indicated thermal efficiency of a two stroke diesel engine is 60%. If friction power is 15%
change of enthalpy at entrance and exit of turbine is 500KJ/kg. of heat generated, determine the brake thermal efficiency of the engine.
A. 4 wells *B. 2 wells C. 6 wells D. 8 wells A. 43% *B. 45 % C. 36% D. 37%
SOLUTION: SOLUTION:
WT = ms(h3 – h4) ne = IP/ Qg
16,000 = ms (500) 0.60 = IP/Qg
0.9(0.8) IP = 0.60 Qg
ms = 44.44 kg/sec BP = IP- FP = 0.60Qg – 0.15Qg = 0.45Qg
ms = 160,000 kg/hr etb = BP/Qg = 0.45Qg/Qg = 45%
160,000 = 0.20 mg
mg = 800,000 kg/hr A 305 mm x 457 mm four stroke single acting diesel engine is rated at 150 KW at 260 rpm.
No. of wells = 800,000/400,000 = 2 wells Fuel consumption at rated load is 0.56 kg/KW-hr with a heating value of 43,912 KJ/kg.
Calculate brake thermal efficiency
A liquid dominated geothermal plant with a single flash separator receives water at 204oC. A. 10.53% B. 27.45% *C. 14.64% D. 18.23%
The separator pressure is 1.04 Mpa. A direct contact condenser operates at 0.034 Mpa. The SOLUTION:
turbine has a polytropic efficiency of 0.75. For a cycle output of 60 MW, what is the mass mf = 0.56 kg/KW-hr x 150 KW = 84 kg/hr = 0.0233 kg/sec
flow rate of the well-water in kg/s?
At 204oC: hf = 870.51 KJ/kg Brake thermal efficiency =
At 1.04 Mpa: hf = 770.38 hfg = 2009.2 hg = 2779.6 sg = 6.5729
At 0.034 MPa: hf = 301.40 hfg = 2328.8 sf = 0.9793 sfg = 6.7463 A waste heat recovery boiler produces 4.8 Mpa(dry saturated) steam from 104°C feedwater.
*A. 2,933 B. 2,100 C. 1,860 D. 2,444 The boiler receives energy from 7 kg/sec of 954°C dry air. After passing through a waste heat
SOLUTION: boiler, the temperature of the air is has been reduce to 343°C. How much steam in kg is
h3 = hg at 1.04 MPa = 2779.6 KJ/kg produced per second? Note: At 4.80 Mpa dry saturated, h = 2796.
Solving for h4: A. 1.30 B. 0.92 *C. 1.81 D. 3.43
s3 = s4 = sf + xsfg SOLUTION:
6.5729 = 0.9793 + x4(6.7463) hf = approximate enthalpy of feedwater
x4 = 0.829 hf = Cpt
h4 = 301.4 + 0.829(2328.8) = 2232.3 KJ/kg hf = 4.187(104)
WT = ms (h3 – h4) hf = 435.45 KJ/kg
60,000 = ms (2779.6 – 2232.3) 0.75 Heat loss = Heat gain
ms = 146.17 kg/sec m gc p(t 1 - t 2) = m s(h - h f)
Solving for x2: (h1 = h2) 7(1.0)(954 – 343) = ms(2796.0 – 436.45)
h1 = h2 = hf + xhfg m s = 1.81 kg/sec
870.51 = 770.38 + x2(2009.2)
x2 = 0..049836 A diesel electric plant supplies energy for Meralco. During a 24-hour period, the plant
ms = x mg consumed 240 gallons of fuel at 28°C and produced 3930 KW-hr. Industrial fuel used is
146.17 = 0.049836 mg 28°API and was purchased at P30 per liter at 15.6°C. What is the cost of the fuel be to produce
mg = 2,933.06 kg/sec one KW-hr?
*A. P6.87 B. P1.10 C. P41.07 D. P5.00
An engine-generator rated 9000 KVA at 80% power factor, 3 phase, 4160 V has an efficiency SOLUTION:
of 90%. If overall plant efficiency is 28%, what is the heat generated by the fuel. SG 15.6C = 141.5/(131.5 + 28) = 0.887
A. 18,800 KW B. 28,800 KW C. 7500 KW *D. 25,714 Density at 15.6°C = 0.887(1kg/li) = 0.887 kg/li
KW SG 28C = 0.887[1-.0007(1 – 15.6)] = .879
SOLUTION: Density at 28°C = 0.879(1 kg/li) = 0.879 kg/li
Gen. Output = pf x KVA = 0.8 x 9000 = 7200 KW V28C / V15.6C = SG15.6C / SG28C
eoverall= Gen. Output 240 / V15.6C = 0.887 / 0.879
Qg V15.6C = 237.835 gallons x 3.785 li/gal = 900.21 li
Cost = [(30)(900.21)] / 3930 = P6.87/KW-hr
0.28 = 7200/Qg
Qg = 25,714.28 KW In a gas turbine unit, air enters the combustion chamber at 550 kpa, 277°C and 43 m/s. The
products of combustion leave the combustor at 511 kpa, 1004°C and 180 m/s. Liquid fuel
enters with a heating value of 43,000 KJ/kg. For fuel-air ratio of 0.0229, what is the combustor In an 9,000 KW hydro-electric plant the over-all efficiency is 88% and the actual power
efficiency of the unit in percent? received by the customer is 110,000 KW-hrs for that day. What is the secondary power could
A. 70.38% B. 79.385% *C. 75.38% D. 82.38% this plant deliver during the entire day?
SOLUTION: A. 58,960 KW-hrs *B. 80,080 KW-hrs C. 65,960 KW-hrs D. 70,960
Heat supplied by fuel = mfQh = 0.0229(43,000) = 984.7 KJ/kg air KW-hrs
Q = heat absorbed by fuel SOLUTION:
Q/m = Cp(T2 – T1) + ½(V22 – V12) Plant Capacity = 9,000(0.88)(24) = 190,080 KW-hrs
Q/m = (1.0)(1004 – 277) + ½[(180) 2 –(43) 2]/1000 =742.28 KJ/kg air Secondary Power = 190,080 – 110,000 = 80,080 KW-hrs

Combustor Efficiency = = 75.38% A Pelton type turbine was installed 30 m below the gate of the penstock. The head loss due to
friction is 12 percent of the given elevation. The length of penstock is 100 m and coefficient of
The specific speed of turbine is 85 rpm and running at 450 rpm. If the head is 20 m and friction is 0.00093. Determine the power output in KW. ( Use Moorse equation)
generator efficiency is 90%, what is the maximum power delivered by the generator. A. 22,273 B. 23,234 C. 32,345 *D. 34,452
A. 450.51 KW B. 354.52 KW C. 650.53 KW*D. 835.57 KW SOLUTION:
SOLUTION: hL = 0.12(30) = 3.6 m
NS = (N√HP)/h5/4 h = 30 – 3.6 = 26.40 m
85 = (450√HP)/(20 x 3.281) 5/4 v = (2gh)1/2 = [(2)(9.81)(26.4)]1/2 = 22.759 m/sec
Hp = 1244.52 hL= (2fLv2)/gD
Generator Output = (1244.52 x 0.746)(0.9) = 835.57 KW 3.6 = (2 x .00093 x 100 x 22.759) / (9.81D)
D = 2.728 m
In Francis turbine, the pressure gage leading to the turbine casing reads 380 Kpa. The velocity
of water entering the turbine is 8 m/sec, if net head of the turbine is 45 m, find the distance Q = Ax v = [ 2](22.759) = 133.03 m3/sec
from center of spiral casing to the tailrace. Power = w Q h = 9.81(133.03)(26.4) = 34,452 KW
*A. 3.0 m B. 3.5 m C. 4.0 m D. 4.5m
SOLUTION : Water flows steadily with a velocity of 3.05 m/s in a horizontal pipe having a diameter of 25.4
cm. At one section of the pipe, the temperature and pressure of the water are 21C and 689.3
Kpa, respectively. At a distance of 304.8 m downstream
h= V2/2g
45 = (380/9.81) + z + [82/(2 x 9.81)] A hydro electric plant having 30 sq. km reservoir area and 100 m head is used to generate
z=3m power. The energy utilized by the consumers whose load is connected to the power plant
during a five-hour period is 13.5 x 106 kwh. The overall generation efficiency is 75%. Find
A turbine has a mechanical efficiency of 93%, volumetric efficiency of 95% and total the fall in the height of water in the reservoir after the 5-hour period.
efficiency of 82%. If effective head is 40 m, find the total head. A. 5.13 m B. 1.32 m C. 3.21 *D. 2.20 m
A. 48.72 m B. 40.72 m *C. 36.22 m D. 34.72 m SOLUTION
SOLUTION: Energy Output = Power x time = (w Q h) x time
eT = emehev 13.5 x 106 = 9.81(Q)(100)(0.75)(5)
0.8 = 0.93(eh)(.95) Q = 3669.725 m3/s
ηh = 0.9055 Volume after 5 hrs = 3669.725(5 x 3600) = 66,055,050 m3
Total head = h eh = (40)(0.9055) = 36.22 m Volume = A x height
66,055,050 = (30 x 106) h
A Pelton type turbine has 25 m head friction loss of 4.5 m. The coefficient of friction head loss H =2.202 m
(from Moorse) is 0.00093 and penstock length of 80 m. What is the penstock diameter?
*A. 1,355.73 mm B. 3,476.12 mm C. 6771.23 mm D. The gas density of chimney is 0.75 kg/m3 and air density of 1.15 kg/m3. Find the driving
1686.73 mm pressure if the height of chimney is 63.71 m.
SOLUTION: A. 0.15 kpa *B. 0.25 kpa C. 0.35 kpa D. 0.45 kpa
h =25- 4.5 = 20.5 SOLUTION:
v = √(2gh) = [(2 x 9.81 x 20.5)1/2] = 20.55 m/sec hw = H(da – dg) = 63.71(1.15 – 0.75) (0.00981) = 0.25 kpa
hL = (2fLv2)/gD The actual velocity of gas entering in a chimney is 8 m/sec. The gas temperature is 25C with a
4.5 = (2)(0.00093)(80)(20.055)2 / 9.81D gas constant of 0.287 KJ/kg-K. Determine the gas pressure for a mass of gas is 50,000 kg/hr
D = 1,355,730 m = 1,355.73 mm and chimney diameter of 1.39m.
A. 95 kpa *B. 98 kpa C. 101 kpa D. 92 kpa
SOLUTION:
Brine enters a circulating brine cooler at the rate of 60 m3/hr at -*C and leaves at -18C.
Vg = A x v = / 4 (1.39)2(8) = 12.139 m3/s
Specific heat of brine is 1.072 KJ/kg-K and specific gravity of 1.12. Determine the tons of
PgVg = mgRgTg
refrigeration.
P(12.139) = (50,000/3600)(.287)(25 +273)
A. 53.5 TR B. 65.3 TR C.33.5 TR *D. 56.9 TR
P = 97.85 kpa
A steam generator with economizer and air heater has an overall draft loss of 25.78 cm of SOLUTION:
water. If the stack gases are at 177C and if the atmosphere is at 101.3 Kpa and 26C, what Density of brine = 1.12(1000 kg/m3) = 1120 kg/m3
theoretical height of stack in meters is needed when no draft fan are used? Assume that the gas m = (1120)(60)/3600 = 18.67 kg/sec
constant for the flue gases is the same as that for air. Q = mcp = 18.67(1.072)(-8 + 18) = 200.11 KW
A 611.10 B. 631.10 *C.651.10 D.671.10 TR = 200.11/3.516 = 56.91 Tons of refrigeration
SOLUTION: A turbo-charged, 16 cylinder, Vee-type diesel engine has an air consumption of 3,000 kg/hr per
w = P/RT cylinder at rate load and speed. This air is drawn in through a filter by a centrifugal
da = (101.325)/[(.287)(26 + 273)] = 1.180 kg/m3 compressor directly connected to the exhaust gas turine. The temperature of the air from the
dg = (101.3)/[(0.287)(177 +273)] = 0.784 kg/m3 compressor is 135C and a counter flow air cooler reduces the air temperature to 45C before it
Draft = (0.2578)(1000) = 257.80 kg/m3 goes to the engine suction heater. Cooling water enters air cooler at 30C and leaves at 40C.
Draft = H(da – dw) Calculate the log mean temperature difference.
257.80 = H(1.18 – 0.784) A. 47.23C B. 87.82C *C. 43.34C D. 65.24C
H = 651.10 m SOLUTION:
A foundation measures 12 ft x 14 ft x16 ft. Find the number of sacks of cement needed for a = 45-30 = 15C
1:2:4 mixture. b = 135 – 40 = 95C
A. 302 B. 404 C. 356 *D. 598
SOLUTION: mean = [ a - b] / [ln( a b)] = [95-15] / ln(95/15) = 43.34C
V = 12 X 14 X 16 = 2,688 ft3 (1 yd3 / 33 ft3) = 99.55 yd3 of concrete Water is flowing in a pipe with radius of 30 cm at a velocity of 5 m/s at the temperature in
For every 1 yd3 concrete, it needs 6 sacks of cement pipe. The density and viscosity of the water are as follows: density 997.9 kg/sec viscosity =
Therefore: 1.131 Pa-s. What is the Reynolds Number for this situation?
No. of sacks = 6(99.55) = 597.33 sacks or 598 sacks *A. 2647 B. 96.2 C. 3100 D. 1140
A rectangular foundation cross-section has a bed plate dimension of 8 ft x 10 ft. The uniform SOLUTION:
clearance on each side is 1 ft. The height of foundation is 4.5 ft. If the weight of the steel bar n = Dvg / v
reinforcements needed is 1/2% of weight of foundation, find the weight of steel bars. Use Where:
concrete density of 2400 kg/m3 . D = 2(0.30) = 0.60 m
A. 173.47 kg *B. 183.47 kg C. 163.47 kg D. 153.47 kg vg = 5 M/SEC
SOLUTION: v = 1.131/997.9 = 0.0011334 m2 / sec
A = (8 + 2) (10 + 2) = 120 m2 Nm = 0.60(5)/0.0011334 = 2,647
V = Ah = 120(4.5) = 540 ft3 = 15.29 m3 Compute the amount of condensate form during 10 minutes warm-up of 180 meter pipe
W = wV = (2400)(15.29) = 36,693.25 kg conveys the saturated steam with enthalpy vaporization hfg = 1,947.8 LJ/kg. The minimum
Weight of steel bars = (1/2%) Wf = 0.005(36,693.25) = 183.47 kg external temperature of pipe is 2C. The final temperature of pipe is 195C. The specific heat of
pipe material is 0.6 KJ/kg-C. The specific weight is 28 kg/m.
A steam pipe having a surface temperature of 250C passes through a room where the A. 249.69 kg B. 982.45 kg *C. 299.64 kg D.
temperature is 27 C. The outside diameter of pipe is 100 mm and emissivity factor is 0.8. 423.45 kg
Calculate the radial heat loss for 3 m pipe length. SOLUTION:
A. 1434.7 W B. 37.46 W *C. 2651.82 W D. 3545.45 W mp = mass of pipe = 28(180) = 5,040 kg
SOLUTION: Heat Loss by steam = Heat loss from pipe
m(hg - hf) = mpcp (t2 – t1)
A = DL = = 0.425m2 m(1947.8) = (5040)(0.6)(195-2)
Solving for heat due to radiation: m = 299.64 kg
Tg = 250 +273 = 523K
T2 = 27 +273 = 300K
Qa = 20,408.4 x 104 AF(T14 – T24), J/hr = 20,408.4 x 104(0.8)(0.7539)[(523)4 – The discharge pressure of an air compressor is 5 times the suction pressure. If volume flow at
(300)4] suction is 0.1 m3/sec, what is the suction pressure if compressor work is 19.57 kw? (use
Qr = 10,266,539.06 j/hr x 1hr/3600sec = 2851.82 W n=1.35)
A. 97 kpa *B.98 kpa C. 99 kpa D.100 kpa
SOLUTION: *A. 233.34 B. 973.17 C. 198.34 D. 204.82
SOLUTION:
W= [(P2/P1)n-1/n – 1] P1V1K = P2V2K
19.57 = 1.35(P1)(0.1)/(1.35-1)[(5)1.35-1/1.35 – 1] 100(V11.4) = 600(6)1.4
P1 = 98 KPa V1 = 28.768 m3/min
VD = 28.768/0.89 = 32.32 m3/min
The initial condition of air in an air compressor is 98 KPa and 27C and discharge air at 450 W = n P1V1/n-1 x [(P2 / P1)n-1/n – 1] = [(1.4 x 100 x 32.32)/(1.4 – 1)] x
KPa. The bpre and stroke are 355 mm and 381 mm, respectively with percent cleared of 8% [(600/100)1.4-1/1.4 – 1]
running at 300 rpm. Find the volume of air at suction. W = 7562.19 KJ/min
A. 541.62 m3/hr B. 551.62 m3/hr C. 561.62 m3/hr *D. W = Pm x Vd
571.62 m3/hr 7562.19 = Pm x 32.32
Pm = 233.34 KPa
SOLUTION:
A water-jacketed air compressed handles 0.343 m3/s of air entering at 96.5 KPa and 21C and
ev = 1 + c – c(P2/P1)1/n = 1 + 0.08 - 0.08(450/98)1/1.4 = 0.842
leaving at 460 KPa and 132C; 10.9 kg/h of cooling water enters the jacket at 15C and leaves at
VD = D2 LN = (0.355)2 (0.381)(300/60) = 0.1885 m3/sec 21C. Determine the compressor brake power.
V1 = 0.1885(0.842) = 0.15878 m3/sec = 571.62 m3/hr A. 26.163 KW *B. 62.650 KW C. 34.44 KW D.
An air compressor has a suction volume of 0.35 m3/sec t 97 KPa and discharges to 650 KPa. 19.33 KW
How much power saved by the compressor of there are two stages? SOLUTION:
A. 18.27 KW B. 16.54 KW C. 13.86 KW *D. T2/T1 = (P2/P1) n-1/n
11.58 KW (132+273) / (21+273) = (480/96.5)n-1/n
SOLUTION: n = 1.249
W = (1.249 x 96.5 x 0.343) / (1.249-1) [(480 / 96.5)1.249-1/1.249 – 1]
W= [(P2/P1)n-1/n – 1] = (1.4 x 97 x 0.35)/(1.4 -1) [(650/97)1.4-1/1.4 – 1] = W = 62.57 KW
85.79 KW Q = heat loss = mcp(t2 – t1) = (10.9/3600)(4.187)(21 – 15) 0.075 KW
For two stages : Brake power = W + Q = 62.57 + 0.076 = 62.65 KW
Px = (P1P2)1/2 = (97 x 650)1/2 = 251.097 KPa
A double suction centrifugal pumps delivers 20 ft3/sec of water at a head of 12 m and running
W= [(Px/P1)n-1/n – 1] = 2(1.4)(97)(0.35)/(1.4 – 1) [(251.0.97/97)1.4-1/1.4 – at 650 rpm. What is the specific speed of the pump?
1] = 74.208 KW A. 5014.12 rpm B. 6453.12 rpm *C. 2770.73 rpm D. 9966.73 rpm
POWER SAVED = 85.79 – 74.208 = 11.582 KW SOLUTION:
N = N(Q)1/2 / h3/4
A twop stage air compressor has an intercooler pressure of 4 kg/cm2. What is the discharge Q = 20/2 ft3/sec x 7.481 gal/ft3 x 60 sec/1min = 4,488.6 gal/min
pressure if suction pressure is 1 kg/cm2? h = 12 x 3.281 = 39.37 ft
A. 3 kg/cm2 B. 9 kg/cm2 C. 12 kg/cm2 *D. 16 kg/cm2 N = (650 x (4,488.6)1/2)/(39.37)3/4
SOLUTION: N = 2,770.73 rpm
Px = (P1P2)1/2 Determine the number of stages needed for a centrifugal pump if it is used to deliver 400
Px2 = P1(P2) gal/min of water and pump power of 15 Hp. Each impeller develops a head of 30 ft.
42 = 16 kg/cm2 A. 6 B. 4 *C. 5 D. 7
SOLUTION:
A two stage air compressor compresses air at 100 KPa and 22C discharges to 750 KPa. If Wp = w Q h
intercooler intake is 105C. Determine the value of n. 15 x 0.746 = 9.81(400 gal/min x 0.00785m3/gal x 1/60)h
A. 1.400 *B. 1.325 C. 1.345 D. 1.288 h = 45.20 m x 3.281 ft/m = 148.317 ft
SOLUTION: Number of stages = 148.317/40 = 4.94 stages = 5 stages
Px = (100 x 750)1/2 =273.86 KPa The suction pressure of a pump reads 3 in. of mercury vacuum and discharge pressure reads
Tx/T1 = (Px/P1)n-1/n 140 psi is use to deliver 120 gpm of water with specific volume of 0.0163 ft3/lb. Determine
(105 + 273)/(22 + 273) = (273.86/100)n-1/n the pump work.
1.281 = (2.6268)n-1/n A. 4.6 KW B. 5.7 KW *C. 7.4 KW D. 8.4 KW
n = 1.326 SOLUTION:
A single acting compressor has a volumetric efficiency of 89%, operates at 500 rpm. It takes in P1 = -3 in Hg x 101.325/29.92 = -10.16 KPa
air at 900 KPa and 30C and discharges it at 600 KPa. The air handled is 8 m3/min measured at P2 = 140 psi x 101.325/14.7 = 965 KPa
discharge condition. If compression is isentropic, find mean effective pressure in KPa. w = 1/v = 1/0.163 = 61.35 lb/ft3 x 9.81/62.3 = 9.645 KN/m3
h = (P2 – P1)/w = (965 +10.16)/9.645 = 101.105 m
Q = 120 gal/min x 3.785/1gal x 1m3/1000li x 1/60 = 0.00757 m3/sec ep = wa Q hs/BP = (1.18 x 0.00981)(1.42)(21.52) / 0.6588 = 53.69%
P = w Q h = 9.645(0.00757)(101.105) = 7.38 KW
A water cooler uses 50 lb/hr of melting ice to cool running water from 80 to 42 . Based
A submersible pump delivers 350 gpm of water to a height of 5 ft from the ground. The pump
were installed 150 ft below the ground level and draw down of 8 ft during the operation. If on te inside coil area, U1 = 110 Btu/hr-ft2- . Find the gpm of water cooled.
water level is 25 ft above the pump, determine the pump power. A. 0.10 GPM B. 0.21 GPM *C. 0.38 GPM D. 0.45 GPM
A. 7.13 KW B. 4.86 KW C. 7.24 KW *D. SOLUTION:
9.27 KW Q = mf L = mwcpw(t1 – t2)
SOLUTION: 50 (144) = mW(1)(80-42)
h = 5 + 150 – (25 – 8) = 138/3.281 = 42.06 m mw = 189.474 lb/hr
Q = 350 gal/min x 0.003785 m3/gal x 1 min/60sec = 0.02246 m3/sec V = (189.474/62.4) (7.48/60) = 0.38 GPM
Wp = w Q h = 9.81(0.02246)(42.06) = 9.27 KW The charge in a Diesel engine consists of 18.34 grams of fuel, with lower heating value of
42,571 KJ/kg, and 409 grams of fuel and products of combustion. At the beginning of
A vacuum pump is used to drain a flooded mine shaft of 20 water. The pump pressure of
water at this temperature is 2.34 KPa. The pump is incapable of lifting the water higher than compression, t1 = 60 . Let rk = 14. For constant cP = 1.11 KJ/kg-C, what should be the cut-
16 m. What is the atmospheric pressure? off ratio in the corresponding ideal cycle?
*A. 159.30 B. 32.33 C. 196.22 D. 171.9 A. 2.05 B. 2.34 C. 5.34 *D. 2.97
SOLUTION: SOLUTION:
Using Bernoulli’s Theorem: QA = mfQh = 0.01283(42,571) = 780,752 KJ
P1/w + V12/2g + z1 = P2/w + V2/2g + z2 T2/T1 = rkk-1
P1/w = P2/w + (V22 - V12)/2g + (z2 - z1) T2 = (60 + 273)1.4-1 = 956.964K
P1/9.81 = 2.34/9.81 + 0 + 16 mt + mg = 409
P1 = 159.30 KPa mt + ma + mf = 409
A submersible, multi-stage, centrifugal deep well pump 260 gpm capacity is installed in a well ma = 409 – 2(18.34) = 372.32 grams
27 feet below the static water level and running at 3000 rpm. Drawdown when pumping at QA = macp(t3 – t2)
rated capacity is 10 feet. The pump delivers the water into a 25,000 gallons capacity overhead 780.752 = 0.37232(1.11)(T3 – 956.964)
storage tank. Total discharge head developed by pump, including friction in piping is 243 feet.
Calculate the diameter of the impeller of this pump in inches if each impeller diameter T3 = 2846,146
developed a head of 38 ft. rC = T3/T2 = 2846.146/956.964 = 2.97
A. 3.28 B. 5.33 *C. 3.71 D. 6.34
SOLUTION:
V= DN
The gain of entropy during isothermal nonflow process of 5 lb of air at 60 is 0.462 Btu/R.
V= Find the V1/V2.
A. 3.85 *B. 0.259 C. 1.0 D. 0.296
D (3000/60) = (2(32.2)(38))1/2 SOLUTION:
D = 0.315 ft = 3.708 inches
A fan pressure of 2.54 cm of water t 1.42 m3 per second of air at static pressure of 2.54 cm of s = m R T ln(V2/V1)
water through a duct 300 mm diameter and discharges it through a duct 275 mm diameter. 0.462 = 5 (53.33/778) ln (V2/V1)
V2/V1 = 3.85
Determine the static fan efficiency if total fan mechanical is 75% and air measured at 25 V1/V2 = 1/3.85 = 0.259
and 60 mm Hg.
A. 50.11% *B. 53.69% C. 65.67% D. 45.34%
SOLUTION: An auditorium seating 1500 people is to be maintained at 80 dry bulb and 85 wet bulb
wA = P/RT = 101.325/(0.287)(25 + 273) = 1.18 kg/m3 temperature when outdoor air is at 91 dry bulb and 75 wet bulb. Solar heat load is
hA = hwww/wA = (0.0254)(1000)/1.18 = 21.52 m
110,000 Btu/hr and supply air at 60 determine the amount of supply air.
vA = 1.42/( /4)(0.3)2 = 20.09 m/s *A. 93,229.17 lb/hr B. 83,229.17 lb/hr C. 73,229.17 D. 63,229.17 lb/hr
Vd = 1.42/( /4)(0.275)2 = 23.9 m/s SOLUTION:
hv = (23.9)2 – (20.09)2 / 2(9.81) = 8.54 m Sensible heat per person = 225 Btu/hr
h = ha + hv = 21.52 + 8.54 = 30.06 m Qa = 225(1500) + 110,000 = 447,500 Btu/hr
eT = wa Q h/BP Qa = m cp(t1 – t2)
0.75 = (1.18 x 0.00981)(1.42)(30.06) / BP 447,500 = ma(0.24)(80 – 60)
BP = 0.6588 KW ma = 93,229.17 lb/hr
A cylindrical pope with water flowing downward at 0.03 m3/s having top diameter of 0.08,
In a Brayton cycle that operates between temperature limits of 300K and 1773K wit k = 1.4, bottom diameter of 0.04 m and a height of 1.5m. Find the pressure between the pipe
determine the temperature at the end of the compression (isentropic) for maximum work of the A. 154.63 Kpa B. 197.93 Kpa *C. 252.44 Kpa D. 243.92 Kpa
cycle. SOLUTION:
A. 700K B. 590.5K *C. 730K D. 350K
SOLUTION: + Z1 = + Z2
For maximum work: T2 = (T1T3)1/2
T2 = (300 x 1773)1/2 = 730K

At 35% solution leaves the absorber and 30% solution enters the absorber. The heat removed
Z1 – Z2 = 1.5 m
from the absorber by cooling water is 547.6 Btu and ammonia is superheated by 10 . Find the Z2 – Z1 = -1.5 m
pound per pound of ammonia gas from the evaporating coils.
A. 11 B. 12 *C. 13 D. 14
SOLUTION:
V1 = = 5.968 m/s
n = lb/lb of ammonia gas from the coils
n = (1 - x2) / (x1 - x2) – 1
n = (1- 0.3) / (0.35 – 0.3) – 1 = 13
A Carnot refrigeration system operates at Tmax / Tmin = 1.5. Find the KW per tons of V2 = = 23.87 m/s
refrigeration.
A. 1.91 B. 2.15 *C. 1.76 D. 1.55
SOLUTION:
P1 – P2 = 252.44 Kpa
Wo / TR = 3.516 / COP = 3.516 / (Tmin / (Tmax – Tmin)
Wo / TR = [3.516(Tmax – Tmin)] / Tmin
Wo / TR = 3.516[(Tmax / Tmin) – 1] = 3.516(1.5-1) = 1.758 KW/TR
Determine the size of pipe which will deliver 8 liters of medium oil (v= 6.10 x 10-6 m2/s)
Assume 8 ft3 of air at 100 psi, 100 are compressed isothermally to a volume of 2 ft3. For assuming laminar flow conditions:
each of end states of the process, find the bulk modulus. A. 622 mm B. 754 mm C. 950 mm *D. 835 mm
*A. 400 and 100 psi B. 400 and 110 psi C. 400 and 120 psi D. 400 and SOLUTION:
130

An empty open can is 30 cm high with a 15 cm diameter. The can, with the open end and V=
down, is pushed under water with a density of 1000kg/m3. Find the water level in the can
when the top of the can is 50 cm below the surface.
A. 17.20 cm *B. 2.12 cm C. 4.20 cm D. 5.87cm Re =
SOLUTION: For laminar flow, Re = 2000
Consider the water pressure
Pw = w h + 1010.325 + (0.8-x)(9.81) + 101.325 = 109.173 – 9.81x
Consider the air pressure 2000 =
P1V1 = P2V2 d = 0.835 m = 835 mm
101.325(Ax0.3) = P2[A(0.3-x)]
The type of flow occupying in a 1 cm diameter pipe which water flows at a velocity of 2.50
P2 = m/s. Use v = 1.13 x 10-6 m2/s for water.
Pw = P2 *A. turbulent B. constant C. laminar D. none of the above

SOLUTION:
109.173 – 9.81x =
9.81x2 – 112.116x + 2.3705 = 0
Re =
By quadratic formula:
X = 0.02118 m = 2.12 cm Re =

Re = 22,124
Since it is greater than 2000 then it is turbulent flow SOLUTION:
T1 = 250 + 273 = 523oK
What is the force is exerted by water jet 60 mm diameter if it strikes a wall at the rate of 15 To = T1 + v2/2000 = 523 = 3802/2000 = 595.2oK
m/s? P1 = 1200 kpa
*A. 636.17 N B.442.62 N C. 764.23 N D. 563.34 N T1/To = (P1/Po)k-1/k
SOLUTION: For CO2: k = 1.289
F=wQv 523/595.2 = (1200/Po)1.289-1/1.289
P0 = 2,136.34 kpa
Q = Av = = 0.0424 m3/s
F = (1000)(0.0424)(15) = 636.17 N Air enters a diffuser with a velocity of 200 m/s. Determine the velocity of sound if air
temperature is 30oC.
A 300 mm diameter pipe discharges water at the rate of 200 li/s. Point 1 on the pipe has a *A. 349 m/s B. 359 m/s C. 369 m/s D. 379 m/s
pressure of 260 kpa and 3.4 m below point 1 is point 2 with a pressure of 300 kpa. Compute SOLUTION:
the head loss between points 1 and 2. C=
A. 4.29 m B. 2.59 m C. 6.32 m *D. 1.87 m
SOLUTION:

hL
Air flows through a nozzle with temperature of entrance of 420oK stagnation temperature of
hL = 468oK. Find the mach number.
A. 0.744 *B. 0.754 C. 0.764 D. 0.774
Water flowing at the rate of 10 m/s from an orifice at the bottom of a reservoir. Find the SOLUTION:
pressure at the bottom of the reservoir. To = T1 + v2/2000Cp
A. 30 kpag B. 40 kpag *C. 50 kpag D. 60 kpag 468 = 420 + v2/2000
SOLUTION: v = 309.838 m/s
h = V2/ 2g = 102/ 2(9.81) = 5.0968 m
P = w h = 9.81(5.0968) = 50 kpag C=
M = v/C = 309.838/410.8 = 0.754
Steam flows through a nozzle at 400oC and 1 Mpa (h = 3263.9 KJ/kg) with velocity of 300
m/s. Find the stagnation enthalpy. Air at 300oK and 200 kpa is heated at constant pressure to 600oK. Determine the change of
A. 3300 KJ/kg B. 3290 KJ/kg *C. 3320 KJ/kg *D. 3309 KJ/kg internal energy.
SOLUTION: A. 245.58 KJ/kg B. 235.58 KJ/kg C. 225.58 KJ/kg *D. 215.58
ho = h + v2/2000 = 3263.9 + 3002/2000 = 3309 KJ/kg KJ/kg
SOLUTION:
Air flows through a nozzle at a speed of 350 m/s. Find the stagnation temperature if entrance ΔU = mCv (T2 – T1) = 1(0.7186)(600 -300) = 215.58 KJ/kg
temperature is 200oC.
A. 241.25oC B. 251.25oC *C. 261.25oC D. 271.25oC An insulated rigid tank initially contains 1.5 lb of helium at 80oF and 50 psia. A paddle wheel
SOLUTION: with power rating of 0.02 hp is operated within the tank for 30 min. Determine the final
To = T1 + v2/2000Cp = (2000 + 273) + 3502/2000(1) temperature.
To = 534.25oK = 261.25 A. 159.22oF B. 169.22oF *C. 179.22oF D. 189.22 oF
SOLUTION:
Carbon dioxide flows through a nozzle with a speed of 400 m/s. Compute the dynamic W = ΔU = m Cv (T2 – T1)
temperature. 0.02 hp (0.50hr)(2545Btu/hr/hp) = 1.5(0.171)(t2 – 80)
A. 92.56oK *B. 94.56oK C. 96.56oK D. 98.56oK t2 = 179.22oF
SOLUTION:
For CO2: Cp = 0.846 KJ/kg-K A 4m2 asphalt pavement with emissivity of 0.85 has a surface temperature of 50oC. Find the
Dynamic temperature = v2/2000Cp = 4002/2000(0.846) = 94.56oK maximum rate of radiation that can be emitted from the surface.
A. 2,068.32 watts B. 2,078.32 watts C. 2,088.32 watts *D. 2.098.32
Carbon dioxide flows through a nozzle with a speed of 380 m/s. The entrance condition of watts
nozzle is 250oC and 1200 kpa. Find the stagnation pressure. SOLUTION:
*A. 2,136.34 kpa B. 2,146.34 kpa C. 2,156.34 kpa D. 2,166.34 Qr = e kev A Ts4
kpa Kev = 5.67 x 10-8 ( Stefan Boltzman constant)
Qr = 0.85(5.67 z 10-8)(4)*50 +273)4 = 2,098.32 watts
Wc = 7.68/2.5 = 3.072 KW
Air at 10oC and 90 kpa enters a diffuser of a jet engine steadily with a velocity of 200 m/s.
The inlet area diffuser is 0.40 m2. Determine the mass flow rate of air. A heat source at 8000K losses 2000 KJ of heat to a sink at 500oK. Determine the entropy
A. 72.79 kg/s B. 74.79 kg/s C. 76.79 kg/s *D. 78.79 kg/s generated during this process.
SOLUTION: *A. 1.5 KJ/K B. 2.5 KJ/K C. -2.5 KJ/K D. 4
W = P/RT = 80/0.287(10 + 273) = 0.985 kg/m3 KJ/K
m = w v A = 0.985(200)(0.40) = 78.79 kg/s SOLUTION:
ΔSsource = -2000/800 = -2.5
Consider a refrigeration whose 40 watts light bulb remains on continuously as a result of a ΔSsink = 2000/500 = 4
malfunction of the switch. If the refrigerator has a COP of 1.3 and the cost of electricity is 8 ΔSgen. = -2.5 + 4 = 1.5 KJ/K
cents per kw-hr., determine the increase in the energy consumption of the refrigerator and its
cost per year if the switch is not fixed. Helium gas is compressed in an adiabatic compressor from an initial state of 14 psia and 50oF
*A. P49.59 B. P47.59 C. P45.59 D. P43.59 to a final temperature of 320oF in a reversible manner. Determine the exit pressure of Helium.
SOLUTION: A. 38.5 psia *B. 40.5 psia C. 42.5 psia D.
COP = RE/Wref 44.5 psia
1.3 = 40/Wref SOLUTION:
Wref = 30.769 watts T2/T1 = (P2/P1)n-1/n
W = Wb + Wref = 40 + 30.769 = 70.77 watts (320 + 460)/(50 +460) = (P2/14)1.587-1/1.587
W = 0.07077 Kw P2 = 40.5 psia
Cost = 0.07077(8760)(P0.08) = P49.59
Air pass thru a nozzle with efficiency of 90%. The velocity of air at the exit is 600 m/s. Find
A 75 hp motor that has an efficiency of 91% is worn out and is replaced by a high-efficiency the actual velocity at the exit.
motor that has an efficiency of 95.4%. Determine the reduction in heat gain of the room due to A. 382 m/s B. 540 m/s C. 458 m/s *D. 568 m/s
higher efficiency under full-load conditions. SOLUTION:
A. 2.24 KW *B. 2.44 KW C. 2.64 KW D. 2.84 KW e = (v2/v3)2
SOLUTION: 0.9 = (v2/600)2
P01 = (75 x 0.746)(0.91) = 50.91 KW v2 = 568.21 m/s
P02 = (75 x 0.746)(0.954) = 53.376 KW
Qreduced = 53.376 – 50.91 = 2.44 KW A 50 kg block of iron casting at 500K is thrown into a large lake that is at a temperature of
258oK. The iron block eventually reaches thermal equilibrium with the lake water. Assuming
A household refrigerator that has a power input of 450 watts and a COP of 2.5 is to cool five average specific hear of 0.45 KJ/kg-K for the iron, determine the entropy generated during this
large watermelons, 10 kg each, to 8oC. If the watermelons are initially at 20oC, determine process.
how long will take for the refrigerator cool them. The watermelons can be treated as a water *A. -12.65 KJ/k B. 16.97KJ/K C. 4.32 KJ/K D. 6.32 KJ/K
whose specific heat is 4.2 KJ/kg-oK. SOLUTION:
A. 2220 seconds B. 2230 seconds *C.2240 seconds D. 2250 ΔSiron = m c ln (T2/T1) = 50(0.45)ln(285/500) = -12.65 KJ/K
seconds ΔSlake = Q/T = [50(0.45)(500-285)]/285 = 16.97 KJ/K
SOLUTION: ΔSgen. = -12.65 + 16.97 = 4.32 KJ/K
COP = RE/Wc
2.5 = RE/450 A windmill with a 12 m diameter rotor is to be installed at a location where the wind is
RE = 1,125 watts blowing at an average velocity of 10 /s. Using standard conditions of air (1 atm, 25oC),
RE = m cp (t2 – t1) determine the maximum that can be generated by the windmill.
450 t = (10 x 5)(4.2)(20-8) A. 68 KW *B. 70 KW C. 72 KW D. 74
t = 2240 seconds KW
SOLUTION:
When a man returns to his wall-sealed house on a summer day, he finds that the house is at w = P/RT = 101.325/(0.28)(25+ 273) = 1.1847 kg/m3
32oC. He returns on the air conditioner which cools the entire house to 20oC in 15 minutes, if m = w A v = 1.1847(π/4 x 122)(10) = 1,1339.895 kg/s
COP is 2.5, determine the power drawn by the airconditioner. Assume the entire mass within KE = v2/2000 = 102/2000 = 0.05 KJ/kg
the house is 800 kg of air for which cv = 0.72 KJ/kg-K, cp = 1.0KJ/kg-K. Power = m KE = 1,1339.895(0.05) = 70 KW
A. 1.072 KW B. 2.072 KW *C. 3.072 KW D. 4.072 KW
SOLUTION: Consider a large furnace that can supply heat at a temperature of 2000oR at a steady rate of
RE = m cv (T2 –T1) = (800/15x60)(0.72)(32-20) 3000Btu/s. Determine the energy of this energy. Assume an environment temperature of 77oF.
RE = 7.66 KW
A. 2305.19 KW *B. 2315.19 KW C. 2325.19 KW D. Assume the atmospheric to be 14.7 psia and the temperature and the volume to remain
2335.19 KW constant.
SOLUTION: *A. 0.026 lb B. 0.046 lb C. 0.066 lb D. 0.086 lb
SOLUTION:
e= = 0.7315 PV=mRT
W = e Q = 0.7315(3000) = 2194.5 Btu/s = 2315.19 KW (20 + 14.7)(144)(0.53) = m1 (53.3)(90 + 460)
m1 = 0.09034 lb
(30 + 14.7)(144)(0.53) = m2(53.3)(90 + 460)
m2 = 0.11634 lb
A heat engine receives hat from a source at 1200oK at a rate of 5000KJ/s and rejects the waste madded = m2 – m1 = 0.11634 – 0.09034 = 0.026 lb
heat to a medium at 3000oK. The power output of the heat engine is 180 KW. Determine the
irreversible rate for this process. A rigid tank contains 20 lbm of air at 20 psia and 70oF. More air is added to the tank until the
A. 190 KW *B. 195 KW C. 200 KW D. 205 KW pressure and temperature rise to 35 psia and 90 oF, respectively. Determine the amount of air
SOLUTION: added to the tank.
e = (1200 – 300) / 1200 = 0.75 A. 11.73 lb *B. 13.73 lb C. 15.73 lb D. 17.73 lb
W = 0.75(500) = 375 KW SOLUTION:
Irreversibilities = 375 – 180 195 KW P1V1 = m1 R1T1
(20 x 144)(V1) = 20 (53.3)(70 + 460)
A dealer advertises that he has just received a shipment of electric resistance heaters for V = 196.17 ft3
residential buildings that have an efficiency of 100 percent. Assuming an indoor temperature P2V2 = m2R2T2
of 21oC and outdoor temperature of 10oC, determine the second law efficiency of these (35 x 144)(196.17) = m2 (53.3)(90 + 460)
heaters. m2 = 33.73 lbs
A. 8.74% B. 6.74% *C. 3.74% D. 4.74% madded = m2 –m1 = 33.73 – 20 = 13.73 lb
SOLUTION:
COP1 = 100% efficient = 1 A rigid tank contains 5 kg of an ideal gas at 4 atm and 40oC. Now a valve is opened, and half
COP2 = (21 + 273) / (21 – 10) = 26.72 of mass of the gas is allowed to escape. If the final pressure in the tank is 1,5 atm, the final
e = COP1 /COP2 = 1/ 26.72 = 3.74 % temperature in the tank is:
*A. -38oC B. -30oC C. 40oC D. 53oC
A thermal power plant has a heat rate of 11,363 Btu/KW-hr. Find the thermal efficiency of the SOLUTION:
plant. PV = m R T
A. 34% B. 24% C. 26% *D. 30% (4 x 9.81)(V) = 5(0.287)(40 + 273)
SOLUTION: V = 11.446 m3
e = 3412 / Heat rate = 3412 / 11363 = 30 % PV = mRT
(1.5 x 9.81)(11.446) = (5/2)(0.287)(T)
A rigid tank contains 2 kmol of N2 and 6 kmol of CO2 gasses at 300oK and 115 Mpa. Find the T = 234.74oK = -38.26oC
tank volume us ideal gas equation.
A. 7.33 m3 B. 5.33 m3 C. 3.33 m3 *D. 1.33 m3 The pressure of an automobile tire is measured to be 200 kpa(gage) before the trip and 220
SOLUTION: kpa(gage) after the trip at a location where the atmospheric pressure is 90 kpa. If the
PmVm = Nm R Tm temperature of the air in the tire before the trip is 25oC, the air temperature after the trip is:
15,000 Vm = (6 + 2)(8.314)(300) *A. 45.6oC B. 54.6oC C. 27.5oC D. 26.7oC
Vm = 1.33 m3 SOLUTION:
T2 / T1 = P2 / P1
A spherical balloon with a diameter of 6 m is filled with helium at 20oC and 200 kpa. T2 / (25+ 273) = (220 +90) / (200 + 90)
Determine the mole number. T2 = 318.55 K
*A. 9.28 Kmol B. 10.28 Kmol C. 11.28 Kmol D. 13.28 Kmol t2 = 45.55oC
SOLUTION:
PV=NRT Water is boiling at 1 atm pressure in a stainless steel pan on an electric range. It is observed
(200)[(4/3)(π)(6/2)3] = N (8.314)(20 + 273) that 2 kg of liquid ater evaporates in 30 mins. The rate of heat transfer to the water is:
N = 9.28 Kmol A. 2.97 KW B. 0.47 KW *C. 2.51 KW D. 3.12 KW
SOLUTION:
The air in an automobile tire with a volume of 0.53 ft3 is at 90oF and 20 psig. Determine the
amount of air that must be added to raise the pressure to the recommended value of 30 psig. Q = mL = (2257) = 2.51 KW
Consider a person standing in a breezy room at 20oC. Determine the total rate of heat transfer system is raised by 15oC. The amount of extra heat that must be supplied to the gas in the
from this person if the exposed surface area and the average outer surface temperature of the cylinder that is maintained at constant pressure.
person are 1.6 m2 and 29oC, respectively, and the convection heat transfer coefficient is 6
W/m2 with emissivity factor of 0.95. SOLUTION:
A. 86.40 watts B. 61.70 watts C. 198.1 watts *D. 168.1 watts A. 0 B. 50 KJ C. 100 KJ *D. 150 KJ
SOLUTION: Q = m cp (t2 – t1) = (1.2 x 8.314)(1)(15) = 150 KJ
Qc = h A (t2 – t1) = (6)(1.6)(29.20) = 86.40 watts
Qr = (0.95)(5.67 x 10-6)[(1.6)(29 + 273)4 – (20 + 273)4] = 81.7 watts A supply of 50 kg of chicken needs at 6oC contained in a box is to be frozen to -18oC in a
Q = Qc + Qr = 86.40 + 81.7 = 168.1 watts freezer. Determine the amount of heat that needs to be removed. The latent heat of chicken is
247 KJ/kg, and its specific heat is 3.32 KJ/kg-oC above freezing and 1.77 KJ/kg-oC below
Water is boiler in a pan on a stove at sea level. During 10 minutes of boiling, it is observed freezing. The container box is 1.5 kg, and the specific heat of the box material is 1.4 Kj/kg-oC.
that 200 grams of water has evaporated. Then the rate of heat transfer to the water is: Also the freezing temperature of chicken is -2.8oC.
A. 0.84 KJ/min *B. 45.1 KJ/min C. 41.8 KJ/min D. 53.5 KJ/min *A. 15,206.4 KJ B. 50.4 KJ C. 15,156 KJ D. 1,863 KJ
SOLUTION: SOLUTION:
Q = m L = (0.2 / 10) (2257) = 45.1 KJ/min Qchicken = 50 [3.32(6 + 2.8) = 247 1.77(-2.8 + 18)] = 15,156 KJ
Qbox = 1.5(1.4)(6 + 8) = 50.4 KJ
An aluminum pan whose thermal conductivity is 237 W/m-C has a flat bottom whose diameter Q = 15,156 + 50.4 = 15, 206.4 KJ
is 20 cm and thickness 0.4 cm. Heat is transferred steadily to boiling water in the pan through
its bottom at a rate of 500 watts. If the inner surface of the bottom of the pan is 105oC, Water is being heated in a closed pan on top of a range while being stirred by a paddle wheel.
determine the temperature of the surface of the bottom of the pan. During the process, 30 KJ of heat is transferred to the water, and 5 KJ of heat is lost to the
A. 95.27 oC *B. 105.27oC C. 115.27oC D. 125.27oC surrounding air. The paddle wheel work amounts to 500 N-m. Determine the final energy of
SOLUTION: the system if its initial energy is 10 KJ.
A = π / 4 ( 0.20)2 = 0.0314 m2 *A. 35.5 KJ B. 45.5 KJ C. 25.5 KJ D. 14.5 KJ
SOLUTION:
Q= Final energy = Qa + ΔU – Qloss + W = 30 + 10 – 5 + 0.50 = 35.5 KJ

A classroom that normally contains 40 people is to be air- conditioned with window air-
500 =
conditioning units of 5 KW cooling capacity. A person at rest may be assumed to dissipate heat
at a rate of about 360 KJ/hr. There are 10 light bulbs in the room, each with a rating of 100
T2 = 105.27oC
watts. The rate of heat transfer to the classroom through the walls and the windows is
estimated to be 15,00 KJ/hr. If the room air is to be maintained at a constant temperature of
For heat transfer purposes, a standing man can be modeled as a 30 cm diameter, 170 cm long
21oC, determine the number of window air- conditioning units required.
vertical cylinder with both the top and bottom surfaces insulated and with the side surface at
A. 1 unit *B. 2 units C. 3 units D. 4 units
an average temperature of 34oC. For a convection heat transfer coefficient of 15 W/m2- oC,
SOLUTION:
determine the rate of heat loss from this man by convection in an environment at 20oC.
Q = total head load = 40(360/3600) + 10(0.100) + 15,000/3600 = 9.167 KW
A. 316.46 watts B. 326.46 watts *C. 336.46 watts D. 346.46 watts
No. of air-conditioning = 9.167/5 = 1.833 = 2 units
SOLUTION:
Qc = k A (t2 – t1) = 15 (π x 0.30 x 1.7) (34 – 20) = 336.46 watts
A 4m x 5m x 6m room is to be heated by a baseboard resistance heater. It is desired that the
resistance heater be able to raise the air temperature in the room from 7 to 23oC within 15
A 5cm diameter spherical ball whose surface is maintained at a temperature of 70oC is
minutes. Assuming no heat losses from the room and an atmospheric pressure of 100 kpa,
suspended in the middle of a room at 20oC. If the convection heat transfer coefficient is 15
determine the required power of the resistance heater. Assume constant specific heats at room
W/m2 – C and the emissivity of the surface is 0.8, determine the total heat transfer from the
temperature.
ball.
A. 2.34 KW *B. 1.91 KW C. 4.56 KW D.
A. 23.56 watts *B. 32.77 watts C. 9.22 watts D. 43.45 watts
6.34 KW
SOLUTION:
SOLUTION:
A = 4 π r2 = 4 π (0.05)2 = 0.0314 m2
w = P / R T = 100 / (0.287)(7 +273) = 1.244 kg / m3
Qc = h A (t2 – t1) = 15 (0.0314)(70 – 20) = 23.56 watts
m = 1.244 (4 x 5 x 6) = 149.28 kg
Qr = (0.80)(5.67 x 10-6)(0.0314)[(70 + 273)4 – (50 + 273)4] = 9.22 watts
Q = m cv (t2 – t1 ) = 149.28 (0.7186)(23 – 7) = 1,716.36 KJ
Q = Qr + Qc = 23.56 + 9.22 = 32.77 watts
Power = 1,716.36 / (15 x 60) = 1.91 KW
A frictionless piston-cylinder device and rigid tank contain 1.2 kmol of ideal gas at the same
A student in a 4m x 6m x 6m dormitory room turns on her 150 watts fan before she leaves the
temperature, pressure, and volume. Now heat is transferred, and the temperature of both
room on a summer day, hoping that the room will be cooler when she comes back in the
evening. Assuming all the doors and windows are tightly closed and disregarding any heat pump the cold- deep water to the surface and through the system heat.Assume a Carnot cycle
transfer through the walls and the windows, determine the temperature in the room when she efficiency and density of cold water to be 1000 kg/m3.
comes back 10 hours later. Use specific heat values at room temperature, and assume the room A. 108 *B. 250 C. 146 D. 160
to be at 100 kpa and 15oC in the morning when she leaves SOLUTION:
A. 28.13oC B. 38.13oC C. 48.13oC *D. 58.13oC e = (Th – TL)/ TH = [(26 + 273) – (15 + 273)] / (26 + 273) = 0.0679
SOLUTION: e =W / Qa
w = P / R T = 100 / (0.287)(15 + 273) = 1.2098 kg / m3 0.03676 = 10, 000 / QA
m = 1.2098(4 x 6 x 6) = 174.216 kg QA = 271, 612. 99 KW
Q = m cv (t2 – t1) QR = Qa – W = 271,812.99 – 10, 000 = 261,813 KW
0.15(10 x 3600) = 174.216 (0.7186)(t2 – 15) QR = m cp (Δt)
t2 = 58.13oC 261, 813 = m (4.187)(3)
M = 20, 643.32 kg/s
A piston cylinder device whose piston is resting on top of a set stops initially contains 0.50 kg Q = 20,843.32 kg/s or 20,843.32 li/s = 20.843 m3/s
of helium gas at 100 kpa and 25oC. The mass of the piston is such that 500 kpa of pressure is H = P / w = 12 / 9.81 = 1.223 m
required to raise it. How much heat must be transferred to the helium before the piston starts Wp = w Q h = 9.81(20.843)(1.223) = 250.12 KW
rising?
A. 1557.13 KJ B. 1657.13 KJ C. 1757.13 KJ *D. 1857.13 A plate – type solar energy collector with an absorbing surface covered by a glass plate is to
KJ receive an incident radiation of 800 W/m2. The glass plate has a reflective of 0.12 and a
SOLUTION: transmissivity of 0.85. The absorbing surface has an absorptivity of 0.90. The area of the
For helium: cv = R / (k-1) = (8.314 / 4) (1.667 – 1) = 3,116 KJ/ kg-K collector is 5m2. How much solar energy in watts is absorbing by the collector?
T2 = (25 + 273)(500 / 100) = 1,490oK A. 2500 B. 2880 C. 3510 *D. 3060
T1 = 25 + 273 = 298oK SOLUTION:
Q = m cv (T2 – T1) = 0.50(3.116)(1490 – 298) = 1857.13 KJ Q = heat absorbed from sun
Q = (800 W/m2)(5 m2)(0.85)(0.9) = 3,060 watts
In order to cool 1 ton (100kg) of water at 20oC in an insulated tank, a person pours 80 kg of
ice at -5oC into the water. Determine the final equilibrium temperature in the tank. The A tank contains liquid nitrogen at -190oC is suspended in a vacuum shell by three stainless
melting temperature and the hat of fusion of ice at atmospheric pressure are 0oC and 333.7 steel rods 0.80 cm in diameter and 3 meters long with a thermal conductivity of 16.3 W/m2-C.
kJ/kg, respectively. If the ambient outside the vacuum shell is 15oC, calculate the magnitude of the conductive
*A. 12.43oC B. 14.43oC C. 16.43oC D. 18.43oC heat flow in watts along the support rods.
SOLUTION: *A. 0.168 B. 0.0587 C. 0.182 D. 0.176
Qwater =Qice SOLUTION:
1000(4.187)(20 –te) = 80(2.09)(0 + 5) + 80(333.7) + 80(4.187)(te – 0) Q = h A (Δt) = 16.3 (π/4 x 0.0082)(15 – (-190)) = 0.168 watts
te = 14.43oC An elastic sphere containing gas at 120 kpa has a diameter of 1.0 m. Heating the sphere causes
it to expand to a diameter of 1.3 m. During the process the pressure is proportional to the
A fan is powered by a 0.5 hp motor and delivers air at a rate of 85 m3/min. Determine the sphere diameter. Calculate the work done by the gas in KJ.
highest value for the average velocity of air mobilized by the fan. The density of air to 1.18 A. 41.8 B. 50.6 *C. 87.5 D. 35.4
kg/m3. SOLUTION:
PαD
A. 18.23 m/s *B. 21.12 m/s C. 25.34 m/s D. P=kD
32.23 m/s 120 = k(1)
SOLUTION: K = 120
P=wQh P = 120 D
0.50(0.746) = ( 1.18 x 0.00981)( 85 / 60) (h) V = 4/3 π(D/d)2 = 4/24 π D3
h = 22.74 m dV = (12/24) π D2 dD

v= = 21.12 m/s W= D3dD


W = 87.47 KJ
An Ocean – Thermal Energy Conversion power plant generates 10,000 KW using a warm
surface water inlet temperature of 26oC and a cold deep- water temperature of 15oC. ON the An ideal gas with a molecular weight of 7.1 kg/kg mol is compressed from 600 kpa and
basis of a 3oC drop in the temperature of the warm water and a 3oC rise in the temperature of 280oK to a final specific volume of 0.5 m3/kg. During the process the pressure varies
the cold water due to removal and addition of heat, calculate the power required in KW to according to p = 620 + 150v + 95v2 where p is in kpa and v in m3/kg. Calculate the work of
compression in KJ/kg?
A. 32.8 B. 28.7 C. 35.6 *D. 33.3
SOLUTION: A simple Rankine cycle produces 40 MW of power, 50 MW of process heated and rejects 50
V1 = R T / P = (8.314 / 7.1)(280) / (600) = 0.546 m3/kg MW of heat to the surroundings. What is the utilization factor of this cogeneration cycle
neglecting the pump work?
W= A. 50% B. 60% *C. 64% D. 80%
SOLUTION:
QA = WT + WprocessWp = 40 + 50 + 50 = 140 KW

UF = = 64 %
One cubic meter container contains a mixture of gases composed of 0.02 kg-mol of oxygen
and 0.04 kg-mol of helium at a pressure of 220 kpa. What is the temperature of this ideal gas The rate of heat transfer to the surroundings’ from a person at a rest is about 400 KJ/hr.
mixture in degrees Kelvin? Suppose that the ventilation system fails in an automobile in an auditorium containing 120
*A. 441 B. 350 C. 400 D. 450 people and assuming that the energy goes into the air of volume 1500 m3 initially at 300oK
SOLUTION: and 101 kpa, calculate the rate in oC/min of air temperature change.
V = V1 + V2 A. 0.81 B. 0.53 *C. 0.63 D. 1.0
SOLUTION:
VT = Q = m cv Δt
PV = m RT
1= 101(1500) = m (0.287)(300)
m = 1,759.58 kg
T = 441 oK Q = m cv Δt
120(400 / 60) = 1759.58 (0.7186)(Δt)
Methyl alcohol (CH3OH) is burned with 25% excess air. How much unburned oxygen in kg- Δt = 0.633 oC / min
mol-oxygen / kg-mol fuel will there be in the products if the combustion is complete?
A. 0.35 B. 0.45 *C. 0.37 D. 0.65 An insulated box containing helium gas falls from a balloon 4.5 km above the earth’s surface.
SOLUTION: Calculate the temperature rise in oC of the helium when box hits the ground.
CH3OH + O2 + (3.76)N2 = CO2 + H2O + (3.76)N2 A. 15.2 B. 12.6 C. 25.3 *D. 14.1
CH3OH + 1.5O2 + 1.5(3.76)N2 = 1 CO2 + 2H2O + 1.5(3.76)N2 SOLUTION:
Consider 25% excess air: Cv of helium = 3118.9 j/kg-C
CH3OH + 1.25(1.5)O2 + 1.25(1.5)(3.76)N2 = 1CO2 + 2H2O + 1.25(1.5)(3.76)N2 + m g h = m cv Δt
0.25(1.5)O2 m (9.81)(4500) = m (3118.9) Δt
Unburned O2 = 0.25(1.5) = 0.375 Δt = 14.15 oC

A 12 DC electrical motor draws a current of 15 amps. How much work in KJ does this motor Consider two Carnot heat engines operating in series. The first engine receives heat from the
produce over a 10 minute period of operation? reservoir at 2500oK and rejects the waste heat to another reservoir at temperature T. The
*A. 108.0 B. 129.6 C. 216.0 D. 318.2 second engine receives heat by the first one, convert some of it to work, and rejects the rest to
SOLUTION: a reservoir at 300oK. If thermal efficiencies of both engines are the same, determine the
W=E=QV temperature T.
W = (15 x 10 x 60)(12) = 108,000 J = 108 KJ *A. 849oK B. 578 oK C. 763 oK D. 976 oK
SOLUTION:
A 4 liter (2-liter per revolution at standard pressure and temperature) spark ignition engine has et = e2
a compression ratio of 8 and 2200 KJ/kg heat addition by the fluid combustion. Considering a
cold air-standard Otto cycle model, how much power will the engine produce when operating
at 2500 rpm? T = 849oK
*A. 166.53 hp B. 73.12 hp C. 97.4 hp D. 148 hp
SOLUTION: An ideal gas mixture consists of 2 kmol of N2 and 6 kmol of CO2. The mass fraction of CO2
W = 1.2 kg/m3 (standard density of air) is:
M = 2 li/rev x 2500 rev/min x 1.2 kg/m3 x 1 m3/ 1000 li x 1 min/ 60 sec = 0.10 kg/s A. 0.175 B. 0.250 *C. 0.825 D. 0.750
e = W / QA SOLUTION:
e = 1 -1 / 8 1.4 -1 = 0.5647
0.5647 = W / 2200 6(44)
W = 1,242.34 KJ/kg (0.10 kg/s) = 124.23 KW = 166. 53 hp Mass fraction of CO2 = = 0.825
6(44) + 2(28) Air entered a turbojet engine at 200 m/s at a rate of 20 kg/s, and exists at
An ideal gas mixture consists of 2 kmol of N2 and 6 kmol of CO2. The 800 m/s relative to the aircraft. the thrust developed by the engine is:
apparent gas constant of mixture is: A. 6KN *B. 12KN C.16KN D. 20KN
*A. 0.208 B. 0.231 C. 0.531 D. 0.825 Solution:
Solution: Thrust developed = m(v2 – v1) = 20(800 – 200) = 12,000N = 12KN
M= (2/8)(28) + (6/8)(44) = 40 A thermal power has a net power 10MW. The backwork ratio of the plant is
R= 8.314/M = 8.314/40 = 0.208 KJ/kg-K 0.005. Determine the compressor wor.
A Carnot cycle operates between the temperature limits of 300OK and A. 50.15KW B. 50.35KW *C.50.25KW D. 50.45KW
1500OK, and produces 600 KW of net power. Solution:
The rate of entropy changes of the working fluid during the heat addition Wnet= WT + WP
process is: BW= WP / WT
A. 0 B. 0.4KW/K *C. 0.5KW/K D.2.0KW/K 0.005 =WP / WT
Solution: WP= 0.005WT
W= ( s) (TH – TL) Wnet= WT - WP
600 = ( s) (1500 – 300) 10,000 = WT – 0.005WT
( s)= 0.50 KW/K WT= 10,050.25 KW
Air in an ideal Diesel cycle is compressed from 3L to 0.15L and then it WC= 0.005(10,050.25) = 50.25KW
expands during the constant pressure heat addition process to 0.3L. Under A heat engine receives heat from a source at 1200oK at a rate of 500KJ/s
cold air standard conditions, the thermal efficiency of this cycle is: and rejects the waste heat to a sink at 300oK. If the power output of the
rk = 3/0.15= 20 engine is 200KW, the second law efficiency of the heat engine is:
rc= 0.3/0.15= 2 A.35% B.40% *C.53% D.75%
Solution:
1 rck- 1 1 21.2 - 1 ea= 200/500 = 0.40
e=1- = 1- et= (TH – TL)/TH = (1200 – 300)/1200 = 0.75
= 0.6467 es= 0.40/0.75= 53.33%
rkk-1 k(rc – 1) 201.4-1 1.4 (2- A water reservoir contains 100,000 kg of water at an average elevation of
1) 60 m. The maximum amount of electric power that can be generated from
=64.67% this water is:
Helium gas is an ideal Otto cycle is compressed from 20oC and 2L to 0.25L A.8KWh *B.16KWh C.1630KWh D.58,
and its temperature increases by an additional 800oC during the addition 800KWh
process. The temperature of helium before the expansion process is: Solution:
*A. 1700oC B. 1440oC C. 1240oC D.880oC P= m h = (100,000 x 0.00981)(60)= 58,860 KJ
Solution: P= 58,860 KJ x KWh/3600 KJ = 16.35KWh
rk =2/0 25 = 8 A house is maintained at 22oC in winter by electric resistance heaters. If
T2 = (20+273) (8)1.667-1 =1,172K the outdoor temperature is 5oC, the second law of efficiency of the
T3 = T2 + 800 = 1172 + 800 = 1972oK resistance heaters is:
t3 = 1699oC = 1700oK A.0% *B.5.8% C.34% D.77%
In an ideal Brayton cycle has a net work output of 150KJ/kg and backwork Solution:
ratio of 0.4. If both the turbine and the compressor had an isentropic ea= 100% of resistance heaters
efficiency of 80%, the net work output of the cycle would be. et= (22 – 5)/(22 + 273) = 5.8%
A. 50KJ/kg *B. 75KJ/kg C. 98KJ/kg D.120KJ/kg es= 5.8/100 = 5.8%
Solution: A thermoelectric refrigerator that resembles a small ice chest is powered
Backwork ratio = WO/WT by a car battery, and has a COP of 0.10. If the refrigerator cools at 0.350L
0.40 = WO/WT canned drink from 20OC to 4OC in 30 min. determine the average electric
WO = 0.40 WT power consumed by the thermoelectric refrigerator.
Wnet = WT - WO *A.130 watts B.110 watts C.120 watts
150 = WT – 0.4 WT D.140 watts
WT =250 KJ/kg Solution:
WT ‘=250(0.8) = 200KJ/kg (1 x 0.35)
WP = 0.40(200) =100KJ/kg Q= m cp(t2 – t1) = (4.187)(20 – 4) = 13 watts
WP’ = 100/0.80 =125 KJ/kg 30 x 60
Wnet= WT’ – WC’= 200 – 125 = 75 KJ/kg COP= RE/Wc
0.10= 13/0.10= 130 watts $ Savings per month = (1200 – 500) (0.085) = $59.50
A Carnot refrigerator operates in a room in which the temperature is 25OC An ammonia simple saturation cycle operates with a suction pressure of
and consumes 2 kW of power when operating. If the food compartment of 291.6 kPa and a condenser pressure of 1204 kPa develops 15 tons of
the refrigerator is to be maintained at 3OC, determine the rate of heat refrigeration. Determine the theoretical horsepower of the compressor.
removal from the food compartment. The following enthalpies have been found: condenser entrance =
*A.1504.8 kJ/min B.12.86 kJ/min C.1625 kJ/min 1653 kJ/kg, exit =346.6 kJ/kg, compressor entrance = 1450.2 kJ/kg, exit=
D.9.57 kJ/min 1653kJ/kg.
Solution: A.7.23 hp *B.13 hp C. 15 hp
COP= TL /TH– TL = (3 + 273)/ (25 + 273) – (3+273) =12.54 D.8.23 hp
QL= COP x W = 12.54 x 2(60) = 1504.8 kJ/min Solution:
A household refrigerator with EER 8.0 removes heat from the refrigerated m= Qe/ (h1 – h4) = (15 x 3.52) / (1450.2 – 346.6) = 0.0478 kg/s
space at a rate of 90 kJ/min. Determine the rate of heat transfer to the W= m (h2 – h1) = (0.0478)(1653 – 1450.2) / 0.746 = 13 hp
kitchen air. An ammonia ice plant operates between a condenser temperature of 35OC
A.101.25 kJ/min B.63.05 kJ/min *C.128.46 kJ/min and evaporator of -15OC. It produces 10 metric tons of ice per day from
D.80 kJ/min water at 30OC to ice at -5OC. Assuming simple saturation cycle, determine
Solution: the horsepower of the motor if the adiabatic efficiency of the compressor
COP= EER /3.412 = 8/ 3.412 = 2.34 ηc=0.85 and mechanical efficiency ηm=0.95. The specific heat of ice is
COP= QL /QH – QL = 2.34 = 90 / QH – 90 2,094kJ/kg. OC and the latent heat is 335kJ/kg.
QH=128.46Kj/min From the table for ammonia the following enthalpies are: condenser
An air-conditioning system is used to maintain a house at 75OF when the entrance = 1703 kJ/kg, exit= 366.1 kJ/kg; compressor entrance=
temperature outside is 95OF. The house is gaining heat through the walls 1443.9kJ/kg, exit = 1703kJ/kg
and windows at a rate of 1250 Btu/min, and the heat generation rate A.17.68 hp B.18.61 hp C.15.5 hp *D.21.9 hp
within the house from people, lights and appliances amounts to 350 Solution:
Btu/min. Determine the minimum power input required for this air- qe=cpa (te – tf) lhlatentlcph(tf – ts)
conditioning system. = (4.187) (30 – 0) + 335 + (2.094) [(0- (-5)] = 471.08 kJ/kg
A.10.06 hp B.1.36 hp *C.1.41 hp D.7.94 hp Qe= (10 x 1000) (471.08) / 24 = 196,283.33 kJ/hr= 54.523kJ/s
m= Qe / (h2 – h4) = (54.523)/ (1443.9 – 366.1) = 0.05059kg/s
W= m (h2 – h1) = (0.0509) (1703 – 1443.9)/ 0.746 = 17.68 hp
Solution: Wmotor= 17.68 / (0.85) (0.95) =21.9 hp
QL= 1250 + 350 = 1600 Btu/min A Freon 22 air conditioning under standard operating conditions of 35OC is
COP= TL / TH – TL= (75 + 460)/(95 + 460) - (75+460) = 26.75 condensing and 5OC evaporating temperatures. The volume flow rate
W= QL /COP = (1600 / 26.75) / 42.4 = 1.41 hp entering the compressor is 23.72 L/s. determine the refrigerating capacity
A refrigeration system is to cool bread loaves with an average mass of 450 if the refrigerating effect is 164 kJ/kg. From the table for R22 the specific
g from 22OC to -10OC at a rate of 500 loaves per hour by refrigerated air. volume at the compressor entrance is 40.36L/kg.
Taking the average specific and latent heats of bread to be 2.93 kJ/kg, OC A.393.3 TR B.79.3 TR C.96.4 TR *D.27.4 TR
and 109.3 kJ/kg, respectively, determine the product load. Solution:
A.541.7 kJ/min B.351.6 Kj/min *C.761.5 kJ/min m= V1/v1= 23.72 / 40.36 = 0.5877 kg/s
D.409.9 kJ/min Qe= m (qe) = 0.5877 (164)/ 3.52 = 27.4 TR
Solution:
Mbread= (500 breads/h) (0.45 kg / bread) = 225 kg/h
Qtotal= Qbread + Qfreezing
= (mcp∆T)breadl (mhlatent)bread = (225)(2.93)[22-(-10)] l (225)
(109.3) The refrigerant volume flow rate at the entrance of compressor were
Qtotal= 45,688.5 kJ/h = 761.5 kJ/min obtained from a test on a twin cylinder single acting 15 cm x 20 cm, 320
A house that was heated by electric resistance heaters consumed 1200 rpm compressor ammonia refrigerating plant 33 L/s. Determine the
kWh of electric energy in a winter month. If this house were heated instead volumetric efficiency of the compressor.
by a heat pump that has an average performance factor, PF of 2.4, A.77.65% *B.87.6% C.97.6 TR D.65.65%
determine how much money the homeowner would be saved that month. Solution:
Assume a price of 0.085$/kWh for electricity. VD = (π D2 L/4) N= (π /4) (0.15)2(0.2) (320) (2) = 2.26 m3/min
A. $42.5 *B. $59.50 C.$109 D.$97.75 nv =V1/VD = 33/2.26 (1000/60) = 0.876 or 87.6 %
Solution:
W= QH/PF = 1200kWh / 2.4 = 500 kWh
A twin cylinder ammonia compressor with volume displacement of 14,726 m = Qc/qc = 50/1154.4 = 0.0433kg/s
cm3 operates at 300 rpm. Condenser and evaporator pressure are 1200 Solving for refrigerating effect using energy balance about the
kPa and 227 kPa respectively. Specific volume of refrigerant at the evaporator h5 – h4 = 366.1kJ/kg
entrance of compressor is 528.26 L/kg. Compression process is polytrophic
with a 1.20 and clearance factor of compressor is 2%. Determine qe= h1 – h5 = 1442.6 – 366.1 = 1076.5kJ/kg
horsepower required. Therefore, the refrigerating capacity
A.60hp B.70hp C.80hp D.90hp Qe= m qe = [(0.0433) (1076.5)] / 3.52 = 13.24 TR
Solution:
nv= 1+c-c(p2/p1)1/n= 1 + 0.02 – 0.02(1200/227)1.20= 0.8725 An actual refrigerating cycle using R12 as working fluid, the refrigerant
VD= (π D2 L/4) N= (0.014726) (320) (2) = 9.424 m3/min flow rate is 0.05 kg/s. Vapor enters the expansion valve at 1.15 MPa, 40OC
V1= (nv) VD- (0.8725) (9.424) - 8.2224 m3/min (h = 238.5kJ/kg) and leaves the evaporator at 175 kPa, -15OC (h =
W= (n p1 V1 /1-n) [(p2/p1) n-1/n-1] 345kJ/kg). The electric input to motor driving the compressor is measured
= [(1.20 x 2.27 x 8.2224)/ (1-1.20)] [(1200/2.27)1.2-1/1.2 -1] = 3582 and found 3.0 kW. Motor efficiency at this load is 92% and mechanical
kJ/min efficiency 82%. Determine the actual coefficient of performance for this
W= 3582/ (60) (0.746) = 80 hp cycle.
A.1.58 B.2.36 C.1.78 D.1.34
A reversed Carnot cycle has a refrigerating COP of 2.5. Determine the ratio Solution:
TH/TL? Qe= (0.05) (345 - 238.5) = 5.325kW
A.1.4 B.1.5 C.1.25 D.1.2 W = (3) (0.92) (0.82) = 2.26kW
Solution: COPactual= 5.325 / 2.26 = 2.36
COP= TH/ (TH-TL)
1/COP = (TH-TL)/TL= TH/TL-1 An ammonia refrigeration system the temperature in the evaporator is
TH/TL =1 + 1/COP =1 +1/2.5 =1.4 -12OC and the ammonia at the evaporator entry is 0.1511 dry while at exit
is 0.95 dry. If the rate of ammonia circulation is 5.64 kg/min, determine the
Three thousand cubic feet per minute of air are circulated over an air- refrigerating capacity of the system. Enthalpy of saturated liquid and vapor
cooled condenser. If the load on the condenser is 64,800 Btu/hr, compute at -12OC is 144.929kJ/kg and 1447.74kJ/kg respectively.
the temperature rise of the air passing over the condenser. Specific volume A.17.82 B.34.82TR C.27.82TR
of standard air (13.34ft3/lb) D.4.82TR
A.10OF B.15OF C.20OF D.25OF Solution:
Solution: h4= hf4 + x4 (hg4 – hf4) = 144.929 + (0.1511) (1447.74 – 144.929)
Qc= m c ∆t = 341.78
∆t = Qc/ m c = 64,800/ [3000 (60) / (13.34)] (0.24) = 20OF h1= hf1 + x1 (hg1 – hf1) = 144.929 + (0.95) (1447.74 – 144.929) =
1382.6kJ/kg
Saturated vapor ammonia at -16OC (h1 = 1442.60 kJ/kg) leaves the Qe = m (h1 – h4) = (5.64) (1382.6 – 341.78) / 211 = 27.82 TR
evaporator and enters the compressor at -6OC (h1 = 1465kJ/kg. The
refrigerant leaves the condenser as saturated liquid at 40OC (h4= 390.6 A two stage cascade vapor compression refrigeration system uses
kJ/kg) and enter the expansion valve at 35OC (h5 =366.1kJ/kg). Heat ammonia in the low-temperature loop and R-12 in the high-temperature
rejected from the compressor amount to 50kW. The work to compressor is loop. The ammonia provides 15 tons cooling. If the high-loop temperature
208kJ/kg, while the heat loss from compressor is 33kJ/kg. If 95kJ/kg of heat requires 10.12 kW compressor power and low-loop 15.93 kW, determine
are lost in the piping between the compressor discharge and condenser the COP of the system.
inlet, determine the refrigerating capacity of the system. A.2.027 B.5.22TR C.3.314TR D.9.1TR
A.49.5 TR B.46.61 TR C.12.88 TR D.13.24 TR Solution:
Solution: Wtotal= 10.12 + 15.93 = 26.05 kW
Solving for the enthalpy at the exit of compressor using energy COP = 15 (3.52) /26.05 = 2.027
balance about the compressor
h2 = h1 + w - qwc = 1465 + 208 – 33= 1640 kJ/kg When a man returns to his well-sealed house on a summer day, he finds
Solving for the enthalpy at the entrance of condenser using energy that the house is at 32OC. He turns on the air conditioner, which cools the
balance about piping from compressor exit to condenser entrance entire house to 20OC in 15min. If the COP of the air-conditioner system is
h3= h2 – q2-3= 1640 – 95 = 1545kJ/kg 2.5, determine the power drawn by the air conditioners. Assume the entire
Solving for heat rejected in condenser using energy balance about mass within the house is equivalent to 800 kg of air for which c= 0.72kJ/kg
the condenser OC
qc = h3 – h4 = 1545 – 390.6 = 1154.4kJ/kg A.7.68Kw B.3.07kW C.19.2kW D.12.03kW
Solution: m = 18kg
Qe = m c (t1 – t2) / ∆time = (800) (0.72) (32 – 20) / (15 x 60) = A condenser vacuum gauge reads 715 mm Hg when the barometer stands
7.68kW at 757 mm Hg. State the absolute pressure in the condenser in kN/m or
W= Qe / COP = 7.68 / 2.5 = 3.07kW kPa.
A.5.6kPa B.5.9kPa C.6.5kPa D.5.2kPa
It is desired to double the COP of a reversed Carnot engine for cooling from Solution:
5.0 by raising the temperature of the heat addition while keeping the Pabs = Patm – Pvac = 757 – 715 = 42 mm Hg (101.325kPa/760 mm
temperature of the heat rejection constant. By what percentage must the Hg) = 5.60kPa
temperature of heat addition be raised? Determine the force in Newton in a piston of 465 mm2 area with a pressure
A.10.1% B.9.1% C.8.1% D.7.1% of 0.172MPa.
Solution: A.65N B.72N C.80N D.111
Percent increase of the temperature = (TL – TL) TL = TL / TL – 1 of Solution:
heat addition F= PA – (0.172MPa) (105 Pa/MPa) (465 mm2) (m2 / 105 mm2)
Original COP: 5 = TL / (TH – TL); (TH – TL) TL = 1/5; TH / TL -1 = 1/5 F = 79.98N
= 02; TH / TL = 1.2 One piston of a hydraulic press has an area of 1cm2. The other piston has
eq.1 an area of 25cm2. If a force of 130N is applied on the smaller piston, what
Doubling the COP: will be the total force on the larger piston is both piston surfaces are the
10 = TL / (TH – TL); (TH – TL) TL = 1/10; TH / TL – 1 =1/10 = 0.1; same level?
TH/TL=1.1 A.6N B.175N C.3750N D.4250N
Dividing eq.1 and eq.2: Solution:
(TH/TL) (TH/TL) = 1.2 / 1.1 = 1.091 F1/A1 = F2/A2
Percent Increase of heat addition = 1.091 – 1 = 0.091 or 9.1% (150/1) = (F2/ 25)
An ammonia water-cooled compressor receives the refrigerant at specific F2 = 3750N
volume 62 L/kg. It has a piston displacement rate of 3m3/min. If a squirrel If the pressure of a confined gas at a constant temperature is tripled, what
cage motor is running at 1200 rpm drives the compressor and average will happen to the volume?
piston speed is 490m/min, calculate size of cylinder bore. A. The volume will be tripled B. The
A.20.4 cm B.26.0 cm C.16.13 cm D.13.6 cm volume will remain
Solution: C. The volume will be reduced to one-third of its original value D.
Piston speed = 2LN = 490 = 2(L) (1200); L= 0.204 m = 20.4 cm The volume is constant
Vp = (π D2 L/4) N = (π/4) D2 (0.204) (1200) =5 Solution:
D = 0.1613m = 16.13cm P1V1 = P2V2
If the initial volume of an ideal gas is compressed to one-half its original P1V1 = (3P1) V2
volume and to twice its temperature, the pressure: V2 = (1/3) V1
A. Doubles B. Quadruples C. Remains constant
D. Halves The work done on air is 10.86kJ/kg, determine the compressor power if it is
Solution: receiving 272 kg/min if air.
P1V1/T1 = P2V2/T2 A.36.72 hp B.49.23 hp C. 29.54 hp D.66 hp
P1V1/T1= P2 (1/2 V1) / (2T1) Solution:
P2 = 4 P1 W= (10.86) (272.60) = 49.232 kJ/s or kW
If the gage pressure of a medium is 30kPa (vacuum) and the atmospheric = 49.232 kW (1 hp/ 0.746kW) = 65.99 hp
pressure is 101.3kPa, the absolute pressure will be: A water tank of 18 ft long and 4 ft high, calculate the pressure at the
A.131.3kPa B.-71.3kPa C.71.3kPa D.- bottom of the tank.
131.3kPa A.1.733psi B.1.999psi C.2.337psi D.3.773psi
Solution: Solution:
Pabs = Patm - Pvac P= (62.4 lbf/ft3) (4ft) (1ft2 / 144 in2) = 1.733psi
= 101.3kPa - 30kPa = 71.3kPa The pressure of 750 mm Hg in kN/m2
If a particle has a velocity of 4 meters per second and a kinetic energy of A.90 B.100 C.103 D110
144 Joules, then the mass, in kilograms of this particle must be: Solution:
A.44 B.16 C.18 D.24 P= 750 mm Hg (101.325 kPa/ 760 mm Hg)
Solution: = 99.99 kPa
KE = (1/2) (m) (V2) / 2k A double purpose tank 18 ft wide, 24 ft long and 4 ft depth is filled with
144 – (1/2) (m) (4)2 / [(2) (1)] water. What is the weight of water in the tank in long tons?
A.49tons B.48tons C.54tons D.50tons The equivalent weight of mass 10 kg at a location where the acceleration
Solution: of gravity is 9.77 m/sec2
W = (62.4 lb/ft2) [(18) (24)(4)] ft3 = 107,827.2 lbf A. 97.7N B.79.7N C.77.9N
= 107,827.2 lb (1ton/ 2200 lb) = 49tons D.977N
Solution:
Oil flow through a 16 tubes on a single cooler with a velocity of 2 m/s, the Weight = mg/k = [(10) (9.77)] /1 =97.7N
internal diameter of the tube is 30mm and oil density is 0.85gm/ml. Find A transportation company specializes in the shipment of pressurized
the volume flow in liters per sec. gaseous materials. An order is received from 100 liters of a particular gas
A.22.62 B.32.22 C. 62.22 D.42.62 at STP (32OF and 1 atm). What minimum volume tank is necessary to
Solution: transport the gas at 80OF and maximum pressure of 8 atm?
Volume flow rate = (3.1416) (0.015)2 (2) (16) = 0.02262 m3/s or A.16 liters B.14 liters C.10 liters D.12 liters
22.62 liters/s Solution:
P1V1/ T1 = P2V2/ T2
A substance temperature was 620OR. What is the temperature in OC? [(1) (100)/ (32 + 460)] / [(8) (V2)/ (80 + 460)
A.50.7 B.45.54 C.71.11 V2 = 13.72 liters
D.94.44 100 g of water are mixed with 150 g of alcohol (density = 790 kg/m3).
Solution: What is the specific volume of the resulting mixtures, assuming that the
T, OC = [(620 – 460) – 32] (5/9) = 71.11 fluids mixed completely?
A.0.82x10-3 cu.m/kg B.0.88x10-3 cu.m/kg C.0.63x10-3 cu.m/kg
Unknown volume of container gas of gas of 1 atmosphere is allowed to D.1.16x10-3 cu.m/kg
expand to another container of 10 m3 volume at 500 mm Hg at constant Solution:
temperature. Find the unknown volume. Mass of mixtures = 100 + 150 =250 g
A.6.58 m3 B.6.75 m3 C.5.67 m3 D.7.65 m3 Volume of mixture = [(0.100) / (1000)] + [(0.150)/ (790)] = 0.00029
Solution: m3
P1V1 – P2V2 Specific volume of mixture = (0.00029)/ (0.250) = 1.16x10-3
(760) V1 = (500) (10) cu.m/kg
V1= 6.58 m3 How much does 30 lbm weigh on the moon? (gmoon = 5.47 ft/s2
A.20 lbr B.3.2 lbr C.3.4 lbr
An iron block weighs 5 Newton and has volume of 200 cm3. What is the D.5.096 lbr
density of the block? Solution:
A. 2458kg/m3 B. 2485 kg/m3 C. 2584 kg/m3 Weight = mg/k = {[(30) (5.42)]/32.174} = 5.1 lbf
D. 2549 kg/m3 A 10 kg block is raised vertically 3 meters. What is the change in potential
Solution: energy?
Density = specific weight [at sea level or near the surface of the A.320 J B.350 kg-m2/s2 C.294 J
earth] D.350 N-m
= (5N/200 cm) (10 cm3/m3) (1kg/9.8066N) = 2549.30 kg/m3 Solution:
PE = mgs/k = {[(10) (9.8066) (3)]/1} = 294.2 J
If air is at a pressure of 22.22 psia and at temperature of 800 OR, what is How many cubic meters is 100 gallons of liquid?
the specific volume? A.3.7850 cu.m B.0.1638 cu.m C.0.3785 cu.m
A.11.3 ft3/lbm B.33.1 ft3/lbm C.13.3 ft3/lbm D.1.638 cu.m
D.31.3 ft3/lbm Solution:
Solution: 100gal (785 liters/gal) (1m3/1000 liters) = 0.3785 m3
Pv = RT Steam turbine is receiving 1014 lbm/hr of steam, determine the
v = (53.34) (800) / [(22.22) (144)] = 13.33 ft3/lbm horsepower output of the turbine if the work done by steam is 251 Btu/lbm
A.100 hp B.462.7 hp C.200 hp D.6002.7
The specific gravity of mercury is 13.55. What is the specific weight of hp
mercury? Solution:
A.123.9 kN/m3 B.139.2 kN/m3 C.139.9 kN/m3 W= (251 Btu/lbm) (1014 lbm/hr) 1hp/2545 Btu/hr) = 100 hp
D.193.2 kN/m3 What is the resulting pressure when one pound of air at 15 psia and 200OF
Solution: is heated at constant volume to 800OF?
? = (13.55) (9.8066) = 132.88 kN/m3 A.52.1 psia B.15 psia C.28.6 psia D.36.4 psia
Solution:
P1/T1 = P2/T2 ANSWER: D
P2 = [(800+460) / (200+460)] (15) = 28.64 psia As a rule of thumb, for a specified amount of compressed air, the power
A bicycle tire has a volume of 600 cm3. It is inflated with carbon dioxide to consumption of the compressor decreased by ______ for each 3OC drop in
pressure of 551.43kPa at 20OC. How many grams of CO2 are contained in the temperature inlet air to the compressor.
the tire? Rcoz = 0.18896 kJ/kg.K A. 1% B. 1.5% C. 2% D. 2.5%
A.5.98 g B.6.43 g C.4.63 g ANSWER: A
D.3.83 g Modern way of detecting air compressor leak is by using
Solution: A. Soup and water B. Air leak detector C. Acoustic leak detector
m = rv/RT = [(551.43) (600)/ (102)] / [(0.18896) (20+273)] = D. Ammonia leak detector
0.00598 kg or 5.98 g ANSWER: C
For foundation of stacks, the maximum pressure on the soil is equal to the
Is the most common dryer used which consist of rotating cylinder inside pressure due to the weight and the .
which the materials flow while getting in contact with hot gas. A. Soil movement B. Wind movement C. Ground movement
A. Tower dryer B. Centrifugal dryer C. Tray D. Engine movement
dryer D. Rotary dryer ANSWER: B
ANSWER: D Foundation bolts of specified size should be used and surrounded by a pipe
Is the ratio of the mass water-vapor in air and the mass of air if it is sleeve with an inside diameter of at least
saturated is called: A. 3 times the diameter of engine bolt B. 3 times the diameter
A. Humidity ratio B. Mass ratio C. Vapor ratio D. of anchor bolt
Relative humidity C. 2 times the diameter of engine bolt D. 2 times the diameter
ANSWER: D of anchor bolt
The hands feel painfully cold when the skin temperature reaches: ANSWER: B
A.8OC B.10OC C.12OC D.14OC For multi stage compression of an ideal Brayton cycle, the backwork ratio
ANSWER: B will
The refrigerant used in steam jet cooling is: A. Increase B. Decrease C. Remain the same D.
A. Steam B.R-11 C. Ammonia D. Water none of these
ANSWER: D ANSWER: B
The total heat of the air is a function of Type of turbine that has a specific speed below 5
A. WB temperature B.DP temperature C.DB temperature D.WB A. Impulse turbine B. Propeller turbine C. Francis turbine D.
depression Deriaz turbine
ANSWER: A ANSWER: A
Boiling point of Freon-12 at atmospheric pressure is: A high discharge type of turbine
A. 21OF B.15OF C.5OF D.28OF A. Impulse turbine B. Francis turbine C. Propeller turbine D.
ANSWER: A Deriaz turbine
Which of the following is NOT a type of water cooled condenser in ANSWER: C
refrigeration? Use to minimize the speed rise due to a sudden load rejection
A. Double pipe B. Double shell C. Shell and coil D. Shell A. needle valve B. wicket gate C. Shut-off valve
and tube D. jet deflector
ANSWER: B ANSWER: D
Component of absorption refrigeration system in which the solution is Is the speed of a turbine when the head on the turbine is one meter?
cooled by cooling water: A. specific speed B. rated speed C. utilized speed
A. Rectifier B. Generator C. Evaporator D. unit speed
D. Absorber ANSWER: D
ANSWER: D
Cascade refrigeration cycle is often used in industrial process where Is a fluid property which refers to the intermolecular attraction by which
objects must be cooled to temperature below: the separate particles of the fluid arc held together?
A. -46OC B. -56OC C. -66OC D. -76OC A. Cohesion B. Adhesion C. Surface tension D.
ANSWER: A hypertension
Type of refrigerant control designed to maintain a pressure difference while ANSWER: A
the compressor is operating.
A. Thermostatic expansion valve B. Automatic expansion valve C. Which of the following is NOT the cause of black smoke in diesel engine?
Using low side float flooded system D. Capillary tube
A. fuel valve open too long B. high compression pressure C. carbon B.5 20°C
in exhaust pipe D. overload on engine C.6 20°C
D. 020°C
Which of the following is not a method of starting a diesel engine? ANSWER C
A. Manual rope, crank and kick B.Electric (battery) C. Compressed air D. Using The most common gases employed in Stirling and Ericsson cycles are:
another generator A. Air and helium B. Oxygen and helium C. Hydrogen and helium D. Nitrogen
ANSWER D and helium
Two-stroke engine performs to complete one cycle ANSWER C
A. Suction and discharge stroke B. Power and exhaust stroke C. Compression and power In most common designs of Gas turbine, the pressure ratio ranges from
stroke A. 10 to 12
D. Suction and exhaust stroke B. 11 to 16
ANSWER C C. 12 to 18
A type of geothermal plant used when there is a presence of brine extracted from underground D. 15 to 20
A. Dry geothermal plant B. Double-flesh geothermal plant C.Single-flash geothermal
ANSWER B
plant In Brayton cycle, the heat is transformed during what process?
D. Binary geothermal plant
ANSWER D A. constant temperature
Is the most important safety device on the power boiler B. sentropic process
A. Check valve C .isobaric process
B. Gate valve D.isochoric process
C. Safety valve ANSWER C
D. Globe valve The fuel injection process in diesel engine starts when piston
ANSWER C A. Is at the TDC
During hydrostatic test, the safety valves should be B. Leaving TDC
A. Removed C. Approaches TDC
B. Open D. Halfway of the stroke
C. Closed ANSWER C
D. Partially closed If the cut-off ratio of diesel cycle increases, the cycle efficiency will
ANSWER A A. Decrease
Where deaerating heaters are not employed, it is recommended that the temperature of the B. Increase
feed less than C. Remains the same
A. 197°C D. None of these
B. 102°C ANSWER A
C. 104°C The fuel used in a power plant that is used during peak periods
D. 106°C A. Gas
ANSWER A B. Solid
Is a reaction during which chemical energy is released in the form of heat? C. Liquid
A. Cosmic reaction. B.Ethnic reaction C.Endothermic reaction D.Exothermic D. None of these
reaction ANSWER C
ANSWER D Typical compression ratio of Otto cycle is
By reheating the steam in an ideal Rankine cycle the heat rejected will: A. 6
A. Increase B. Decrease C. Remains the same D. None of these above B. 8
ANSWER A C. 10
By increasing the boiler pressure in Rankine cycle the moisture content at boiler exit will: D. 12
A. Increase ANSWER B
B. Decrease If Joule Thompson coefficient is equal to zero, then the process will become
C. Remains the same A. Isentropic
D. None of these above B. Isenthalpic
ANSWER A C. Isobaric
Presently the highest steam temperature allowed at the turbine inlet is about D. Isothermal
A. 40°C ANSWER D
If the fluid passed through a nozzle its entropy will: C. Hallway of aggress
A. Increase D. Pathway of aggress
B. Decrease ANSWER B
C. Remains the same Any device or portion of the equipment used to increase refrigerant pressure
D. None of these A. Pressure relief device
ANSWER C B. Pressure imposing element
Refrigerants consisting of mixtures of two or more different chemical compounds, often used C. Pressure lift device
individually as refrigerants for other applications D. Pressure limiting device
A. Suspension ANSWER B
B. Compound reaction The quantity of refrigerant stored at some point is the refrigeration system for operational,
C. Blends service, or standby purposes
D. Mixing of refrigerants A. Pressure vessel
ANSWER C B. Pumpdown charge
Pairs of mating stop valves that allow sections of a system to be joined before opening these C. Liquid receiver
valves or separated after closing them D. Accumulator
A. Check valve ANSWER B
B. Gate valve Secondary refrigerant is a liquid used for the transmission of heat, without a change of state,
C. Safety valve and having no flash point or a flash point above as determined from ASTM
D. Companion valve A. 150°F B. 160°F C. 180°F D. 200°F
ANSWER D ANSWER A
An enclosed passageway that limits travel to a single path A service valve for dual pressure-relief devices that allows using one device while isolating
A. Corridor the other from the system maintaining one valve in operation at all times
B. Hallway A. Three-way valve
C. Lobby B. Two-way valve
D. Tunnel C. One-way valve
ANSWER A D. Four-way valve
For Immediate Dangerous to Life or Health (IDHL) the maximum concentration from which ANSWER A
unprotected persons have time to escape within without escape-impairing symptoms or Tubing that is unenclosed and therefore exposed to crhsing, abrasion, puncture, or similar
irreversible health damage after installation
A. 16 minutes A. Protected tubing
B. 1 minute B. Bare tubing
C. 20 minutes C. Open tubing
D. 30 minutes D. Unprotected tubing
ANSWER D ANSWER D
The volume as determined from internal dimensions of the container with no allowance for the Refers to blends compromising multiple components of different volatile that, when used in
volume of internal parts refrigeration cycles, change volumetric composition and saturation temperature as they
A. Internal allowance volume evaporate (boil) or condense at constant pressure
B. Internal gross volume A. Zeoline
C. Internal interference volume B. Blending
D. Internal fots volume C. Composition
ANSWER B D. Zertropic
A waiting room or large hallway serving as a waiting room ANSWER D
A. Terrace Is a premises or that portion of a premise from which, because they are disabled, debilitated,
B. Test room or confined, cooperate cannot readily leave without the assistance of others
C. Compound room A. Institutional occupancy
D. Lobby B. Public assembly occupancy
ANSWER D C. Residential occupancy
A continuous and unobstructed path of travel from any in a building or structure to a public D. Commercial occupancy
way ANSWER A
A. Average of aggress Is one in which a secondary coolant is in direct contact with the air or other substance to be
B. Mean o aggress cooled or heated
A. Double indirect open spray system D. Liquid suction heat exchanger
B. Indirect closed system ANSWER C
C. Indirect open spray system A type of valve connected from discharge of compressore directly to suction that is normally
D. Indirect vented closed system closed and will open automatically only if there is high discharge pressure
ANSWER B A. Check valve
Refrigerant number R-744 is: B. Solenoid valve
A. Butane C. King valve
B. Carbon monoxide D. Relief valve
C. Propane ANSWER B
D. Carbon dioxide Use to increase the capacity of condenser
ANSWER D A. Water regulating valve
Refrigerant number R-1150 is: B. Desuperheating coils
A. Propylene C. Liquid-suction heat exchanger
B. Ethane D. Condenser heating coils
C. Ethane ANSWER B
D. Methyl formale Is use to subcooled the refrigerant from the condenser
ANSWER B A. Liquid subcooler
Refrigerant number R-40 is: B. Condenser subcooler
A. Chlorodifluoromethane C. Desuperheating coils
B. Difluoromethane D. Liquid receiver
C. Ammonia ANSWER A
D. Chloromethane Which of the following is NOT a part of low pressure side in refrigeration system?
ANSWER D A. Liquid line
When the air duct system serves several enclosed spaces, the permissible quantity of B. Refrigerant flow control
refrigerant in the system shall not exceed the amount determined by using the total volume of C. Evaporator
those spaces in which the airflow cannot be reduced to less than of its maximum D. Suction line
when the fan is operating ANSWER A
A. One-quarter Which of the following is NOT a part of high pressure side in refrigeration system?
B. One half-quarter A. Compressor
C. Three-quarter B. Condenser
D. One fourth-quarter C. Liquid line
ANSWER A D. Suction line
The space above a suspended ceiling shall not be include 1 in calculating the permissible ANSWER D
quantity of refrigerant in the system unless such space is continuous and is part of the air Which of the following is NOT a part of condensing unit?
return system A. Compressor
A. Partition B. Discharge line
B. Plenum C. Condenser
C. Separator D. Liquid line
D. Plate divider ANSWER D
ANSWER B By subcooling the refrigerant in refrigerant system, the compressor power per unit mass will
Which of the following is NOT a possible location of service valve? A. Increase
A. Suction of compressor B. Decrease
B. Discharge of compressor C. Remains the same
C. Outlet of liquid receiver D. None of these
D. Outlet of condenser ANSWER C
ANSWER D Superheating the refrigerant in refrigeration system without useful cooling, the refrigeration
A cool to serve with evaporator that is use to prevent the liquid refrigerant entering the effect per unit mass will
compressore A. Increase
A. Accumulator B. Decrease
B. Liquid superheater C. Remains the same
C. Drier loop D. None of these
ANSWER C A. Discharge line
Which of the following is NOT a type of air-cooled condenser? B. Liquid line
A. Shell and tube C. Between condenser and liquid receiver
B. Natural draft D. Suction line
C. Forced draft ANSWER B
D. Induced draft Use to detects a vibration in current caused by the ionization of decomposed refrigerant
ANSWER A between two opposite-charged platinum electrodes
A type of refrigerant control typically used in household refrigeration A. Electronic detector
A. Thermostatic expansion valve B. Halide torch
B. Automatic expansion valve C. Bubble method
C. Capillary tube D. Pressurizing
D. High side float ANSWER B
ANSWER C Joints and all refrigerants containing parts of a refrigerating system located in an air duct
Type of condenser that operates like cooling tower carrying conditioned air to and from an occupied space shall be constructed to withstand a
A. Air-cooled condenser temperature of without leakage into the airstream
B. Evaporative condenser A. 550°F
C. Shell and tube condenser B. 600°F
D. Water-cooled condenser C. 650°F
ANSWER B D. 700°F
The major problem of heat pump is ANSWER D
A. Refrigerant used Refrigerant piping crossing an open space that affords passageway in any building shall be
B. Outside air less than_________ above the floor unless the piping is located against the ceiling of such
C. Supply air space and is permitted by the authority having jurisdiction
D. Frosting A. 2.2 m
ANSWER D B. 3.2 m
Dominant refrigerant used in commercial refrigeration system C. 4.2 m
A. R-11 D. 5.2 m
B. R-22 ANSWER A
C. R-12 Methyl chloride shall not be in contact with
D. R-502 A. Aluminum
ANSWER D B. Zinc
Cascade refrigeration system are connected in C. Magnesium
A. Series D. All of these
B. Parallel ANSWER D
C. Series-parallel Shall not be in contact with any halogenated refrigerants
D. Parallel-series A. Aluminum
ANSWER A B. Zinc
Is use to heat up the solution partially before entering the generator in absorption refrigeration C. Magnesium
system D. All of these
A. Rectifier ANSWER C
B. Absorber Are suitable for use in ammonia system
C. Regenerator A. Copper
D. Pump B. Aluminum and its alloy
ANSWER C C. Plastic
The COP of actual absorption refrigeration system is usually D. Cast iron
A. Less than 1 ANSWER B
B. Less than 2 If a pressure-relief device is used to protect a pressure vessel having an inside dimension of 6
C. Less than 3 in or less, the ultimate strength of the pressure vessel so protected shall be sufficient to
D. Less than 4 withstand a pressure at least the design pressure
ANSWER A A. 2 times
Sight glass is often located at: B. 3 times
C. 4 times A. Inside the tube
D. 5 times B. Outside the tube
ANSWER B C. Inside the shell
Seats and discs shall be limited in distortion, by pressure or other cause, to a set pressure D. Outside the shell
change of not more than in a span of five years ANSWER A
A. 1%
B. 5% In use if extracted steam upon condensation gets subcooled
C. 10% A. Trap
D. 50% B. Deaerator
ANSWER B C. Filter
Liquid receivers, if used, or parts of a system designed to receive the refrigerant charge during D. Drain cooler
pumpdown charge. The liquid shall not occupy more than of the volume when
temperature of the refrigerant is 90°F ANSWER D
A. 80% Needs only single pump regardless of number of heaters
B. 85%
C. 90% A. Open heater
D. 95% B. Closed heater
C. Mono heater
ANSWER C D. Regenerative heater
The discharge line (B4) shall be vented to the atmosphere through a fitted to its upper ANSWER B
extremity Is also known as deaerator
A. Nozzle A. Open heater
B. Convergent-divergent nozzle B. Closed heater
C. Pipe C. Reheat heater
D. Diffuser D. Regenerative heater
ANSWER D ANSWER A
Convert fossil fuels into shaft work Dissolve gases like makes water corrosive react with metal to form iron oxide
A. Nuclear power plant A. O2 and N2
B. Gas turbine power plant B. O2 and CO
C. Dendrothermal power plant C. O2 and CO2
D. Thermal power plant D. N2 and SO2
ANSWER D ANSWER C
Ultimate strength drops by 30% as steam temperature raises from for unalloyed steel A cycle typically used in paper mills, textile mills, chemical factories, sugar factories and rice
A. 300 to 400°C mills
B. 400 to 500°C
C. 600 to 700°C A. Cogeneration cycle
D. 700 to 800 °C B. Combined cycle
C. By-product cycle
ANSWER B D. Cascading cycle
Recent practice limits steam temperature to ANSWER A
A. 438°C When process steam is basic need and power is byproduct, this cycle is known as
B. 538°C
C. 638°C A. Cogeneration cycle
D. 738°C B. Combined cycle
C. By-product cycle
ANSWER A D. Cascading cycle
In a closed feed water heater, the feed water pass through ANSWER C
A type of turbine employed where steam continuously extracted for process heating ANSWER D
A. Back pressure turbine Which of the following is lowest grade of coal?
B. Gas turbine
A. Peat
C. Steam turbine
B. Lignite
D. Passout turbine
C. Sub-bituminous
ANSWER D D. Bituminous
Which of the following is used for Binary cycle power generation for high temperature ANSWER B
application Which of the following helps in the ignition of coal?
A. Mercury A. Moisture
B. Sodium B. Ash
C. Potassium C. Fixed carbon
D. All of these D. Volatile matter
ANSWER D ANSWER D
critical temperature of mercury is is the ratio of fixed carbon and volatile matter
A. 1160°C A. Air-fuel ratio
B. 1260°C B. Fuel ratio
C. 1360°C C. Combustion ratio
D. 1460°C D. Carbon-volatile ratio
ANSWER D ANSWER B
Critical pressure of mercury is A suspension of a finely divide fluid in another
A. 100 Mpa A. Filtration
B. 108 Mpa B. Floatation
C. 128 Mpa C. Emulsion
D. 158 Mpa D. Separation
ANSWER B ANSWER C
Method used in converting heat directly to electricity by magnetism Contains 90% gasoline and 10% ethanol
A. Electronic induction A. Gasohol
B. Magnetodynamic B. Gasonol
C. Magnetohyrdodynamic C. Gasothanol
D. Thermoelectric D. Gasethanol
ANSWER C ANSWER A
Which of the following is NOT a material used for thermoelectric elements Process used commercially in coal liquefaction
A. Bismuth telluride A. Tropsch process
B. Lead telluride B. Fisher process
C. Zinc telluride C. Fisher-Tropsch process
D. Germanium D. Mitch-Tropsch process
ANSWER C ANSWER C
A type of coal formed after anthracite Is an organic matter produced by plants in both land and water
A. Lignite A. Bio-ethanol
B. Bituminous B. Biomass
C. pear C. Petroleum
D. graphite D. Biodegradable
ANSWER B ANSWER A
In thermal power plant, induced draft fans are located at the PH of water varies with
A. Exit of furnace A. Pressure
B. Foot if the stack B. Temperature
C. Above the stock C. Density
D. Top of the stock D. Volume
ANSWER B ANSWER B
In thermal power plant, forced draft fans are installed at the Ph value of is usually maintained for boiler water to minimized corrosion
A. Foot of the stack A. 8.5
B. Top of the stack B. 9.5
C. Exit of the preheater C. 10.5
D. Inlet of the preheater D. 11.5
ANSWER D ANSWER C
Known as drum less boiler
What type of turbine that has a degree of reaction of ?
A. La Mont boilers
A. Impulsive turbine
B. Fire tube boiler
B. Reaction turbine
C. Forced circulation boiler
C. Rarsons turbine
D. Once-through boiler
D. Deriaz turbine
ANSWER D
ANSWER C
Reduces the steam temperature by spraying low temperature water from boiler drum
The cooling water is made to fall in series of baffles to expose large surface area for steam fed
A. Reheater
from below to come in direct contact
B. Preheater
C. Desuperheater A. Spray condenser
D. Superheater B. Surface condenser
C. Jet condenser
ANSWER C
D. Barometric condenser
Carbon dioxide can be removed by:
ANSWER D
A. Deaeration
Show the variation of river flow (discharge) with time
B. Aeration
C. Evaporation A. Hydrograph
D. Vaporization B. Hyetograph
C. Mass curve
ANSWER B
D. Flow duration curve
Is often used to absorb silica from water
ANSWER A
A. Sorbent
Is an open channel erected on surface above the ground
B. Rectifier
C. Silica gel A. Canal
D. Magnesium hydroxide B. Tunnel
C. Penstock
ANSWER D
D. Flume
Presence of excess hydrogen ions makes the water
ANSWER D
A. Acidic
Type of turbine used up to 300 m head
B. Alkalinity
C. Base A. Impulsive turbine
D. Hydroxicity B. Francis turbine
C. Propeller turbine A. End of combustion
D. Deriaz turbine B. Middle of combustion
C. Beginning of combustion
ANSWER D
D. Beginning of interaction
A turbine that has a diagonal flow ANSWER A
A. Impulsive turbine In a compression ignition engine, the detonation occurs near the
B. Francis turbine A. End of combustion
C. Propeller turbine B. Middle of combustion
D. Deriaz turbine C. Beginning of combustion
D. Beginning of interation
ANSWER D ANSWER C
Oil is optimized either by air blast or pressure jet at about Morse test is use to measure the of multi-cylinder engine
A. Brake power
A. 60 bar B. Indicated power
B. 70 bar C. Friction power
C. 80 bar D. Motor power
D. 90 bar
ANSWER B
ANSWER B Ignition delay can be minimized by adding to decrease engine knocking
A. Ethel ether
Type of solid injection that use single pump supplies fuel under high pressure to a fuel header
B. Ethyl chloride
A. Common rail injection C. Ethyl nitrate
B. Individual pump injection system D. Ethyl oxide
C. Distributor system ANSWER C
D. Single rail injection For the submerged plane surface, the point on the surface where the resultant force acts is
ANSWER A called the
A. Center of buoyancy
Water flow in diesel engine that is caused by density differential B. Center of gravity
A. Thermosiphon cooling C. Center of pressure
B. Thermostat cooling D. Center of attraction
C. Pressurized water cooling ANSWER C
D. Evaporating cooling At any point in fluid at rest, the pressure is the same in all direction. This principle is known
as:
ANSWER A A. Bernoulli Principle
Type of lubrication system in diesel engine in which oil from pump is carried to a separate B. Archimedes Principle
C. Pascal’s Law
storage tank outside the engine cylinder and used for high capacity engine. D. Torricelli’s Law
A. Mist lubrication system ANSWER C
B. Wet pump lubrication system The hot-wire manometer is used to measure
C. Splash system A. Pressure in gases
D. Dry sump lubrication system B. Pressure in liquids
C. Wind velocities at airports
ANSWER D D. Gas velocities
Produces extreme pressure differentials and violent gas vibration ANSWER D
The pitot static tube measures
A. Vibration
A. The static pressure
B. Detonation
B. The gage pressure
C. Explosion
C. The total pressure
D. Knocking
D. The dynamic pressure
ANSWER B ANSWER A
In a spark ignition engine, the detonation occurs near the The terminal velocity of a small sphere setting in a viscous fluid varies as the
A. First power of its diameter
B. Inverse of fluid viscosity C. Cross-compound turbine
C. Inverse square of the diameter D. Back pressure turbine
D. Inverse of the diameter ANSWER D
ANSWER B A governor with 0% regulation is termed as:
Pressure drag results from A. Isochronous governor
A. Skin friction B. Synchronous governor
B. Deformation drag C. Isenchronous governor
C. Breakdown of potential flow near the forward stagnation point D. Isobarnous governor
D. Occurrence of wake ANSWER A
ANSWER A The speed of regulation for most turbine-generators is adjustable from
The pressure coefficient is the ratio of pressure forces to A. 2 to 6%
A. Viscous forces B. 4 to 8%
B. Inertia forces C. 6 to 10 %
C. Gravity forces D. 8 to 12%
D. Surface tension force ANSWER A
ANSWER B Poppet valves of steam turbine are used for extraction pressures of
Tranquil flow must always occur A. 20 to 120 psig
A. Above the normal depth B. 20 to 150 psig
B. Above the critical depth C. 20 to 130 psig
C. Below the normal depth D. 20 to 140 psig
D. Below the critical depth ANSWER B
ANSWER B When both bearings of steam engine are on one side of the connecting rod, the engine is
Which of the following head loss coefficient among the following types of entrance? referred to as
A. Bell mouth A. Center-crank engine
B. Square edge B. Side crank engine
C. Reentrant C. Under crank engine
D. It depends D. Standard crank engine
ANSWER C ANSWER B
What waste water treatment method involves of algae from stabilization pond effluents? When the valve in steam engine is in mid-position of its travel, it will cover the steam port by
A. Sedimentation an amount known as
B. Floatation A. Steam lap
C. Filtration B. Partial lap
D. Microscreening C. Full lap
ANSWER D D. Angular lap
The number of nozzles will depend on the quantity of steam required by the turbine. If nozzle ANSWER A
occupy the entire arc of the ring, the turbine is said to have A type of governor in steam engine that do not control the actual admission of steam to the
A. Partially full peripheral admission cylinder but controls the pressure of the steam
B. One-half full peripheral admission A. Flyball governor
C. Maximum peripheral admission B. Variable cut-off governor
D. Full peripheral admission C. Throttling governor
ANSWER D D. Shaft governor
Tandem compound units may also have two low pressure castings that produces ANSWER C
A. Single flow By inter-cooling using two stage compressor of brayton cycle, the backward ratio will
B. Double flow A. Increase
C. Triple flow B. Decrease
D. Quadruple flow C. Remains the same
ANSWER C D. None of these
A type of turbine used for driving pumps, fans, and other auxiliaries in power plant commonly ANSWER B
operate at exhaust pressure approximating atmospheric On dynamic similitude, the relation which represents the ratio of inertia force to pressure force
A. Tandem compound turbine is
B. Passout turbine A. Froude number
B. Cauchy number D. Thermo-hydrograph
C. Euler number ANSWER C
D. Raynolds number Entrance losses between tanks and pipe or losses through elbows, fittings and valves are
ANSWER C generally expresses as a function of
What is the maximum velocity in a sewer flowing full? A. Kinetic energy
A. 0.6 m/sec B. Pipe diameter
B. 0.9 m/sec C. Friction factor
C. 1.2 m/sec D. Volume flow rate
D. 1.8 m/sec ANSWER A
ANSWER A The air that contains no water vapor is called
A temporary structure constructed to exclude water from the site of the foundation during its A. Zero air
excavation and construction is called: B. Saturated air
A. Caisson C. Dry air
B. Retaining wall D. Humid air
C. Coffer dam ANSWER C
D. Earth dam In psychometric chart, the constant enthalpy lines coincide with constant temperature lines at
ANSWER C temperature
Which is not a physical characteristics of water? A. Above 50°C
A. Total suspended and dissolved solids B. Below 40°C
B. Turbidity C. Below 50°C
C. Color D. Above 10°C
D. Hardness ANSWER C
ANSWER D The amount of moisture in air depends on its
Which dam is best for weak foundation? A. Pressure
A. Gravity B. Volume
B. Arch C. Temperature
C. Buttress D. Humidity
D. Earth ANSWER C
ANSWER C The deep body temperature of healthy person is maintained constant at
What is the volume of water which will drain freely the aquifer? A. 27°C
A. Specific yield B. 37°C
B. Reservoir yield C. 47°C
C. Safe yield D. 48°C
D. Secondary yield ANSWER B
ANSWER A Air motion also plays important role in
What is the line defined by water level in a group of artesian walls? A. Surroundings
A. Water table B. Cooling
B. Peizometric surface C. Human comfort
C. Specific yield D. None of these
D. All of the above ANSWER C
ANSWER B During simple heating and cooling process has a humidity ratio
Select the one that is positive indication of pollution of a river A. Increasing
A. Acidity B. Decreasing
B. Oxygen content C. Constant
C. Chloride content D. None of these
D. Nitrite content ANSWER C
ANSWER C The follows a line of constant wet bulb temperature on the psychometric chart
Which instruments is used to measure humidity of the atmosphere continuously? A. Evaporative cooling process
A. Barograph B. Condensing process
B. Thermograph C. Direct cooling process
C. Hydrograph D. None of these
ANSWER A In the absence of any work interactions between a system and its surroundings, the amount of
A vapor which is not about to condense is called a net heat transfer is equal
A. Mixture of vapor and liquid A. To the change in total energy of a closed system
B. Critical vapor B. To heat and work
C. Superheated vapor C. Energy interactions
D. None of these D. None of these
ANSWER C ANSWER A
Passing from the solid phase directly into vapor phase is called The constant volume and constant pressure specific heats are identical for
A. Condensation A. Compressible substance
B. Fusion B. Incompressible substance
C. Sublimation C. Compressible gas
D. None of these D. None of these
ANSWER C ANSWER B
Robert Boyle observed during his experiments with a vacuum chamber that the pressure of The velocity of fluid is zero at the wall and maximum at the center because of the
gases is inversely proportional to their A. Velocity effect
A. Temperature B. Viscous effect
B. Pressure C. Temperature effect
C. Volume D. None of these
D. None of these ANSWER B
ANSWER C For steady flow devices, the volume of the control volume is
Is energy in transition A. Increase
A. Heat B. Decrease
B. Work C. Constant
C. Power D. None of these
D. None of these ANSWER C
ANSWER A The work done in a turbine is since it is done by the fluid
Is the mode of energy transfer between solid surface and the adjacent liquid or gas which is in A. Positive
motion, and it involves combine effects of conduction and fluid motion B. Negative
A. Conduction C. Zero
B. Convection D. None of these
C. Radiation ANSWER A
D. None of these Reheating process in Brayton cycle, the turbine work will
ANSWER B A. Increase
Radiation is usually considered as B. Decrease
A. Surface phenomenon C. Remains the same
B. Surface interaction D. None of these
C. Surface corrosion ANSWER A
D. None of these Which of the following is the chemical formula of ethanol
ANSWER A A. C7H16
Work is between the system and the surroundings B. C2H6O
A. Work interaction C. C7H8
B. Energy interaction D. C6H12
C. Heat interaction ANSWER B
D. None of these Which of the following is the chemical formula of Heptane?
ANSWER B A. C7H16
Is a process during which the system remains in equilibrium at all times B. C2H6O
A. Quasi-equilibrium C. C7H8
B. Static equilibrium D. C6H12
C. Dynamic equilibrium ANSWER D
D. None of these Which of the following is the chemical formula of Toluene?
ANSWER A A. C7H16
B. C2H6O B. Work
C. C7H8 C. Heat added
D. C6H12 D. Heat rejected
ANSWER C ANSWER D
As the air passes through a nozzle, which of the following will increase? By decreasing the temperature source of Carnot cycle, which of the following will not be
A. Temperature affected?
B. Enthalpy A. Efficiency
C. Internal energy B. Work
D. Mach number C. Heat added
ANSWER D D. Heat rejected
As the air passes through a diffuser, which of the following will decrease? ANSWER C
A. Temperature By superheating the refrigerant in vapor compression cycle with useful cooling, which of the
B. Enthalpy following will increase? (Use per unit mass analysis)
C. Internal energy A. Condenser pressure
D. Mach number B. Evaporator pressure
ANSWER D C. Quality after expansion
As the air passes through a nozzle, which of the following will decrease? D. Heat rejected from condenser
A. Entropy ANSWER D
B. Velocity By superheating the refrigerant in vapor compression cycle with useful cooling, which of the
C. Internal energy following will decrease? (Use per unit mass analysis)
D. Mach number A. Refrigerant effect B. COP C. Compressor power D. Mass flow rate
ANSWER C ANSWER D
As the air passes through a diffuser, which of the following will increase? By superheating the refrigerant in vapor compression cycle without useful cooling, which of
A. Density the following will decrease? (Use per unit mass analysis)
B. Entropy A. Heat rejected B. COP C. Compressor power D. Specific volume at
C. Mach number suction
D. Velocity ANSWER B
ANSWER A By superheating the refrigerant in vapor compression cycle without useful cooling, which of
As the air passes through a diffuser, which of the following will NOT be affected? the following will increase? (Use per unit mass analysis)
ANSWER B A. Refrigerant effect
After passing through a convergent-divergent nozzle, the temperature of air will: B. COP
A. Increase C. Compressor power
B. Decrease D. Mass flow rate
C. Remains the same ANSWER C
D. None of these By superheating the refrigerant in vapor compression cycle without useful cooling, which of
ANSWER B the following will not be affected? (Use per unit mass analysis)
After passing though a convergent-divergent nozzle, the density of air will: A. Refrigerant effect
A. Increase B. COP
B. Decrease C. Compressor power
C. Remains the same D. Mass flow rate
D. None of these ANSWER C
ANSWER B By sub-cooling the refrigerant in vapor compression cycle at condenser exit, which of the
After passing through a convergent-divergent nozzle, the match number of air will: following will increase? (Use per unit mass analysis)
A. Increase A. Refrigerating effect
B. Decrease B. Specific volume at suction
C. Remains the same C. Compressor power
D. None of these D. Mass flow rate
ANSWER B ANSWER A
By increasing the temperature source of Carnot cycle, which of the following will not be By sub-cooling the refrigerant in vapor compression cycle at condenser exit, which of the
affected? following will decrease (Use per unit mass analysis)
A. Efficiency
A. Coefficient of performance B. Heat rejected from condenser ANSWER: C
C. Refrigerating effect D. Mass flow rate By increasing the boiler pressure in Rankine cycle, which of the following will decrease? (Use
ANSWER: D per unit analysis)
By increasing the vaporizing temperature in vapor compression cycle, which the following A. heat rejected B. pump work C. cycle efficiency D. moisture
will increase? (Use per unit mass analysis) content
A. mass flow rate B. COP C. specific volume D. compressor ANSWER: A
work By superheating the steam to a higher temperature in Rankine cycle, which of the following
ANSWER: B will decrease? (Use per unit analysis)
By increasing the vaporizing temperature in vapor compression cycle, which the following A. moisture content at the turbine exhaust B. turbine work C. heat added
will decrease? (Use per unit mass analysis) D. heat rejected
A. Refrigerating effect B. COP C. evaporator temperature ANSWER: A
D. temperature difference between evaporator and compressor By superheating the steam to a higher temperature in Rankine cycle, which of the following
ANSWER: D will increase? (Use per unit analysis)
By increasing the condenser pressure in vapor compression cycle, which of the following will A. moisture content at the turbine exhaust B. pump work C. condenser
increase? (Use per unit mass analysis) pressure D. cycle efficiency
A. Refrigerating effect B. COP C. Specific volume at suction ANSWER: D, B
D. Compressor power By reheating the steam before entering the second stage in Rankine cycle, which of the
ANSWER: D following will decrease?
By increasing the condenser pressure in vapor compression cycle, which of the following will A. Turbine work B. moisture content after expansion C. heat
decrease? (Use per unit mass analysis) added D. heat rejected
A. moisture content after expansion B. compressor power C. heat ANSWER: B
rejected in the condenser D. mass flow rate What Rankine cycle is modified with regeneration, which of the following will increase?
ANSWER: A A. turbine work B. heat added C. heat rejected D. cycle
If the pressure drop in the condenser increases in a vapor compression cycle, which of the efficiency
following will increase? (Use per unit mass analysis) ANSWER:D
A. mass flow rate B. compressor power C. heat rejected in the condenser Is the combination of base load and peaking load?
D. specific volume at suction A. rated load B. intermediate load C. combine load D.
ANSWER: C over-all load
If the pressure drop in the condenser increases in a vapor compression cycle, which of the ANSWER: B
following will decrease? (Use per unit mass analysis) Sum of the maximum demand over the simultaneous maximum demand?
A. Refrigerating effect B. mass flow rate C. heat rejected in the condenser A. use factor B. capacity factor C. demand factor D. diversity
D. Compressor power factor
ANSWER: B ANSWER: D
If the pressure drop in the condenser increases in a vapor compression cycle, which of the Regenerative with feed heating cycle with infinite number of feed water heaters thus
following will not be affected? (Use per unit mass analysis) efficiency is equal to?
A. compressor power B. mass flow rate C. heat rejected in the A. otto cycle B. stirling cycle C. erricson cycle D. carnot
condenser D. COP cycle
ANSWER: A ANSWER: D
If the pressure drop in the evaporator increases in a vapor compression cycle, which of the A type of turbine used in desalination of sea water.
following will increase? (Use per unit mass analysis) A. back pressure turbine B. passout turbine C. peaking turbine D.
A. Refrigerating effect B. vaporizing temperature reaction turbine
C. heat rejected in the condenser D. COP ANSWER: A
ANSWER: C State that when conductor and magnetic field move relatively to each other, an electric voltage
If the pressure drop in the evaporator increases in a vapor compression cycle, which of the is induced in the conductor.
following will decrease? (Use per unit mass analysis) A. Maxwell’s law B. Kirchhoff’s law C. Faraday’s law D.
A. specific volume at suction B. compressor power C. heat rejected in Newton’s law
the condenser D. COP ANSWER: C
ANSWER: D Transfers heat directly to electrical energy by utilizing thermionic emissions.
By lowering the condenser pressure in Rankine cycle, which of the following will decrease? A. thermionic motor B. thermionic generator C. thermionic converter D.
(Use per unit analysis) thermionic cell
A. Pump work B. turbine work C. heat rejected D. cycle ANSWER: B
efficiency
Is the largest group of coal containing 46-89% of fixed carbon and 20% to 40% volatile A. radiant superheater B. desuperheater C. convective superheater
matter. D. pendant superheater
A. anthracite B. sub-anthracite C. bituminous D. sub- ANSWER: D
bituminous Regenerative superheater is a storage type of heat exchangers have an energy storage
ANSWER: C medium called:
When 1 gram of coal is subjected to a temperature of about 105C for a period of 1 hour, the A. matrix B. regenerator C. boiler D. recuperator
loss in weight of the sample gives the: ANSWER: A
A. Volatile matter B. ash C. fixed carbon D. moisture content Stirling cycle uses a ____ as working fluids.
ANSWER: D A. incompressible gas B. incompressible fluids C. compressible
When 1 gram of sample of coal is placed in a crucible and heated 950C and maintain at the refrigerant D. compressible fluids
temperature for 7 minutes there is a loss in weight due to elimination of: ANSWER: D
A. volatile matter and moisture B. ash C. fixed carbon D. moisture In stirling process the heat added is added during?
content A. isobaric process B. isentropic process C. isothermal process
ANSWER: A D. heat process
Consist of hydrogen and certain hydrogen carbon compounds which can be removed from ANSWER: C
coal by heating. Brayton cycle is also known as:
A. moisture content B. product of combustion C. ash D. volatile A. carnot cycle B. joule cycle C. diesel cycle D. rankine
matter cycle
ANSWER: D ANSWER: B
By heating 1 gram of coal in an uncovered crucible until the coal is completely burned, the Is applied to propulsion of vehicles because of certain practical characteristics.
____ will formed. A. diesel cycle B. otto cycle C. carnot cycle D. brayton
A. volatile matter and moisture B. ash C. fixed carbon D. moisture cycle
content ANSWER: D
ANSWER: B
Caking coal are use to produce coke by heating in a coke oven in the absence of ____ with Heat exchangers typically involve
volatile matter driven off. A. no work interactions B. no heat interactions C. no energy
A. air B. oil C. oxygen D. nitrogen interactions D. none of these
ANSWER: A ANSWER: A
Gindability of standard coal is: A device that is used to convert the heat to work is called
A. 80 B. 90 C. 100 D. 110 A. adiabatier B. regenerator C. heat engines D. none of
ANSWER: C these
ANSWER: C
Major constituent of all natural gases is: The objective of a heat pump is to maintain a heated space at
A. ethane B. methane C. propane D. cethane A. Low temperature B. high temperature C. medium
ANSWER: B temperature D. none of these
Two types of fans are: ANSWER: B
A. centrifugal and axial B. reciprocating and axial C. centrifugal and A device that violates the second law of thermodynamics is called
rotary D. tangential and rotary A. perpetual motion machine of second kind B. perpetual motion machine of third
ANSWER: A kind C. perpetual motion machine of first kind D. none of these
Enthalpy of substance at specified state due to chemical composition. ANSWER: A
A. Enthalpy of reaction B. enthalpy of combustion C. enthalpy of formation A process is called ____ if no irreversibility’s occur outside the system boundaries during the
D. enthalpy of product process.
ANSWER: C A. externally reversible B. internally reversible C. reversible D.
A type of boiler used for super critical pressure operation. none of these
A. La Mont boiler B. Once- through-circulation boiler C. Force ANSWER: A
circulation boiler D. Natural circulation boiler An energy interaction which is not accompanied by entropy transfer is
ANSWER: B A. energy B. heat C. work D. none of these
Economizer in a water tube boiler is heated by: ANSWER: C
A. electric furnace B. electric current C. incoming flue gas D. outgoing A ____ is used in aircraft engines and some automotive engine. In this method, a turbine
flue gas driven by the exhaust gases is used to provide power to compressor or blower at the inlet.
ANSWER: D A. discharging B. turbo charging C. supercharging D. scavenging
Receives heat partly by convection and partly by radiation. ANSWER: B
The only devices where the changes in kinetic energy are significant are the Superheating the steam to higher temperatures decreases the moisture content of the steam at
A. compressor B. pumps C. nozzles and diffusers D. none of the ____.
these A. turbine inlet B. compressor inlet C. compressor exit
ANSWER: C D. turbine exit
The distance between TDC and BDC in which the piston can travel is the ANSWER: D
A. Right extreme position B. displacement stroke C. stroke of Regeneration also provides a convenient of dearating the feedwater to prevent?
the engine D. swept stroke A. boiler explosion B. boiler scale production C. boiler corrosion D.
ANSWER: C compressor damage
In compression-engine, the combustion of air-fuel mixture is self-fuel ignited as a result of ANSWER: C
compressing the mixture above its Can be apply steam turbine cycle(rankine). Gas turbine cycle(brayton) and combined cycle.
A. self developed temperature B. mixing temperature C. self feed A. hydroelectric plant B. nuclear power plant C. cogeneration plant
temperature D. self ignition temperature D. tidal power plant
ANSWER: D ANSWER: C
In a rankine cycle with fixed turbine inlet conditions. What is the effect of lowering the
The thermal efficiency of an ideal Otto cycle depends ____ of the working fluid. condenser pressure, the heat rejected will:
A. the pressure ratio of the engine and the specific heat ratio A. increase B. decrease C. remains the same D. none of
B. the temperature ratio of the engine and the specific heat ratio these
C. the moles ratio of the engine and the specific heat ratio ANSWER: B
D. the compression ratio of the engine and the specific heat ratio In an ideal rankine cycle with fixed boiler and condenser pressure. What id the effect of
ANSWER: D superheating the steam to a higher temperature, the pump work input will.
As the number of stages is increased, the expansion process becomes A. increase B. decrease C. remain the same D. none of
A. isentropic B. isothermal C. isometric D. polytropic these
ANSWER: B ANSWER: C
The fact that total energy in any one energy system remains constant is called the principle of
Aircraft gas turbines operate at higher pressure ratio typically between ____?
A. 6 to 8 B 12 to 24 C. 10 to 18 D. 10 to 25 A. conversion of energy B. second law of thermodynamics C. conservation of
ANSWER: D mass D. zeroth law of thermodynamics
The first commercial high-pass ratio engines has a bypass ratio of ANSWER: A
A. 1 B. 3 C. 5 D. 7 A process for which the inlet and outlet enthalphies are the same
ANSWER: C A. isntropic B. enthalphy conservation C. throttling D. steady state
The single-stage expansion process of an ideal Brayton cycle without regeneration is replaced ANSWER: C
by a multistage expansion process with reheating the samepressure limits. As a result of
modification, thermal efficiency will: The sum of energies of all the molecules in system, energies that in several complex forms.
A. increase B. decrease C. remain constant D. none of A. kinetic energy B. internal energy C. external energy D. flow work
these ANSWER: B
ANSWER: B A system that is completely impervious to its surrounding. Neither mass nor energy cross its
Which of the following is/are the application of Brayton cycle. boundaries.
A. propulsion system B. automotive turbine engine C. aircraft turbine A. Open system B. closed system C. adiabatic system D.
engineD. all of these isolated system
ANSWER: D ANSWER: D
It is used as working fluid in high-temperature applications of vapor cycles. A device used to measure small and moderate pressure difference.
A. helium B. deuterium C. mercury D. water A. manometer B. bourdon gage C. barometer D. piezometer
ANSWER: C ANSWER: A
The superheated vapor enters the turbine and expands isentropically and produces work by A vapor having a temperature higher that the saturation temperature corresponding to its
the rotating shaft. The ____ may drop during the process. pressure.
A. density B. viscosity of fuel C. temperature and pressure A. superheated pressure B. saturated vapor C. super saturated vapor
D. none of these D. subcooled vapor
ANSWER: C ANSWER: A
Only ____ of the turbine work output is required to operate the pump. The energy or stored capacity for performing work possessed by a moving body, by virtue of
A. 0.01% B. 0.02% C. 0.03% D. 0.04% its momentum.
ANSWER: D A. internal energy B. work C. gravitational potential energy
D. kinetic energy
ANSWER: D
A thermodynamic process wherein temperature is constant and the change in internal energy is PIPE ELEMENTS
zero.
A. isobaric process B. isometric process C. isothermal process Which of the following compressors are so widely used for today’s refrigeration system?
D. polytropic process Ans. Centrifugal
The science terminology concerned with precisely measuring energy and enthalpy A valve sometimes known as the magnetic valve.
A. thermodynamics B. chemistry C. calorimetry D. none of these Ans. Solenoid valve
ANSWER: C Which of the following valve use to regulate the flow of refrigerant to the evaporator?
The rate of doing work per unit time Ans. Expansion valve
A. torque B. power C. force D. moment Which of the following use control the flow of refrigerant gas from the evaporator coil. This
ANSWER: B type of control valve is also known as back pressure regulator of an evaporator.
A vapor having a temperature higher than the saturation temperature corresponding to the Ans. Suction line regulator
existing pressure. Which of the following refrigerant control is used to limit the flow of gas to the compressor to
A. superheated vapor B. saturated vapor C. wet vapor D. none of the above prevent surge of excessive load from overloading the compressor.
ANSWER: A Ans. Hold-back valve
It is the work done in pushing a fluid across a boundary, usually or out of a system. Which of the following refrigerant control is used in the liquid suction or discharge to interrupt
A. mechanical work B. nonflow work C. flow work D. electrical work the flow on demand from any one of the several types of temperature or pressure sensing
ANSWER: C devices.
A liquid that has a temperature lower that the saturation temperature corresponding to the Ans. Solenoid valve
existing pressure. What is the pressure drop from the receiver to the expansion valve?
A. subcooled liquid B. saturated liquid C. unsaturated liquid D. water Ans. 5 psi
ANSWER: A If any of the electricity controlled device in a Freon system malfunction, which following
In this type of boiler, the water passes through the tubes while the flue gases burn outside the valves will also automatically shut off?
tubes. Ans. Solenoid valve
A. water-tube boiler B. fire-tube boiler C. steam generator D. electric The relief valve on a CO2 machine is located:
boiler Ans. On the discharge pipe between the compressor and the discharge valve.
ANSWER: A When checking zinc plates in a condenser, one should:
It shows the water level in the boiler drum. Ans. Clean the plate and renew worn-out ones.
A. water column B. try cocks C. gauge glass D. all of the above A refrigerant should have a
ANSWER: C Ans. High latent heat
It prevents damage to the boiler by giving warning of low water. The mechanical energy of a device is the ratio of:
A. safety valve B. fusible plug C. relief valve D. try cocks Ans. Mechanical energy input to the mechanical energy output
ANSWER: B When the winding or circuit is open, what is the resistance reading?
it is heat exchanger which utilizes the heat of the flue gases to preheat air needed for Ans. Infinity
combustion. If the meter scale of the ohmmeter reads 0 ohms and the range adjustment is R 10, what is the
A. economizer B. feedwater heater C. reheater D. air preheater resistance reading?
ANSWER: D Ans. 100
It is the subject that deals with the behavior of moist air. What is the resistance reading of an electric motor in good condition?
A. psychrometer B. psychometry C. refrigeration D. pneumatics Ans. With resistance
ANSWER: B The basic unit of electrical pressure is:
Air whose condition is such that any decreases in temperature will result in condensation of Ans. Volt
water vapor into liquid. What is the most commonly used conductor?
A. saturated air B. unsaturated air C. saturated vapor D. moist air Ans. Copper
ANSWER: A Which of the following is the effect of superheating the refrigerant?
It is the warm water temperature minus the cold water temperature leaving the cooling tower Ans. Increase is COP
A. approach B. terminal difference C. cooling range D. Which of the following statement is true in ice making capacity?
LMTD Ans. Ice making capacity is always proportional to the refrigerating effect
ANSWER: C Which of the following has the largest heat load in cold storage room?
The surrounding air ____ temperature is the lowest temperature to which water could possibly Ans. Product heat load
cooled in a cooling tower. Which of the following is a halocarbon refrigerant?
A. dry-bulb B. wet-bulb C. dew-point D. saturated temperature Ans. Methyl chloride
ANSWER: B A refrigerant system in which pressure- imposing element is mechanically operated?
Ans. Compressor Ans. Blue
Which of the following compressors have the compressing element and drive seals in a single Where is the compound gauge installed in a refrigerant system?
housing? Ans. Suction line
Ans. hermitically sealed compressor A gauge used to measure pressure both below and above atmospheric pressure
Which of the following consists of two mating helically grooved male and female grooves? Ans. Compound gauge
Ans. Helical rotary-screw What is the valve position if the valve stem is turned all the way in?
What is the another name of helical-rotary screw compressors? Ans. Front-seated
Ans. Lysholm type What is the valve position if the valve stem is turned all the way out?
At what head is the helically-rotary compressor are designed to operate? Ans. Back-seated
Ans. High head pressure Ohm’s law states that resistance is inversely proportional to:
The bulk quantity of oil separated from the refrigerant in the helically-rotary compressor falls Ans. Current
by? To prevent corrosion within the flue gas, what must be prevented from condensing?
Ans. Gravity to a sump Ans. Vapor
If the temperature in the icebox is too high, the trouble could be: Ans. Evaporative cooling
Ans. Automatic control not functioning properly # An adiabatic saturation process. This process can be produced with _____, which is essentially
If any of the electrically controlled devices in a Freon system malfunction, which of the a regular thermometer with its bulb wrapped in wet cotton or gauze.....
following valves also automatically shut-off? Ans. Sling psychrometer
Ans. Solenoid valve # A high velocity gas is defined as a gas moving with a velocity in excess off approximately
Natural ice in an ice box was used in the early Ans. 300ft/s/ 100m/s
Ans. 19th century # The theoretical maximum velocity is achieved when.....
The amount of matter present in a quantity of any substance is called: Ans. All internal and pressure energies are converted to kinetic energies
Ans. Mass # If the gas flow is adiabatic and frictionless (i.e. reversible)
What pressure is indicated by a barometer? Ans. Both A and B
Ans. Atmospheric pressure # A property by which the refrigerant remains at its original chemical form/original condition is
The basic unit measurement for heat is: imposed by the operation.....
Ans. BTU Ans. Stability of refrigerant
The force that acts upon a unit of area is termed: # The maximum temperature at which a gas is condense into liquid. Above temperature, a vapor
Ans. Pressure imposed by the operation.....
What is the physical state of a refrigerant entering the flow control? Ans. Stability of refrigerant
Ans. Sub-cooled # Which of the following is the name given for halogenated hydrocarbon?
What method of heat transfer does not depend on molecular motion? Ans. Halocarbon
Ans. Radiation # Which of the following refrigerants are produced by carrier corporation?
Where s the filter drier is located? Ans. Carrene
Ans. After the condenser # Which of the following refrigerants is produced by E.L.DU FONT?
A motor drive device which removes the heat-laden vapor refrigerant from the evaporator is Ans. Freon
the: #Which of the following is the other name of R 718?
Ans. Compressor Ans. Water
The function of the compressor is to: # Energy per unit volume stored in a deformed material is called:
Ans. Squeeze out refrigerant from the evaporator Ans. Strain energy
In which part of the compressor is the piston attached to the crankshaft? # The ratio of ultimate failure strain to yielding strain is known as:
Ans. Connecting rod Ductility
This is a compressor which has an off center-rotor. # One ton of refrigeration is equal to:
Ans. Rotary Ans. 3.5KW
Which of the following compressor is commonly used in household refrigerators? # With what cycle does air refrigeration works?
Ans. Hermetic Ans. Bell-coleman cycle
What is the length of the tubing extended from the part to be swaged # What cycle does air refrigeration works?
Ans. ½ inch Ans. Coal
The best method of making leak proof connection is: # What is produced by fission process?
Ans. Silver brazing Ans. Radiation
The refrigerant that should not be used copper or brass tubing: # Loss of power is due to:
Ans. R-717 Ans. Low injection pressure
The color of the hose connected to suction line or process be # A branch system of pipes to carry waste emissions away from the combustion chamber.
Ans. Exhaust manifold Ans. Acoustic leak detector
# The type of filter where the filtering elements is replaceable. # For foundation of stacks, the maximum pressure on the soil is equal to the pressure due to the
Ans. Metal-edge filter weight and the _______.
# When four events takes place in one revolution of a crankshaft of an engine, the engine is called Ans. Wind movement
Ans. Two stroke engine # Foundation bolts of specified size should be used and surrounded by a pipe sleeve with an inside
# Which of the following does not belong to the group. diameter of at least
Ans. Time injection system Ans. 3 times the diameter of anchor bolt
# The function of a super heater is to: # For multi stage compression of an ideal Brayton cycle, the back work ratio will
Ans. Exchange heat to increase energy to the flow during an adiabatic, internally Ans. decrease
reversible process # Type of turbine that has a specific speed below 5.
# What is true about change in entropy? Ans. Impulse turbine
Ans. it is always zero # A high discharge type of turbine
# How does an adiabatic process compare to an isentropic process? Ans. Propeller turbine
Ans. Both heat transfer = Q, isentropic reversible # Use to minimize the speed rise due to a sudden load rejection
# Adiabatic heat transfer within a vapour cycle refers to : Ans. jet deflector
Ans. The transfer of energy from one stream to another in a heat exchanger that the energy of # Is the speed of a turbine when the head on the turbine is one meter.
the input streams equal to the energy of output. Ans. Unit speed
# All of the following process are irreversible except: # Is a fluid property which refers to the intermolecular attraction by which the separate particles of the
Ans. An isentropic compression of a perfect fluid fluid arc held together.
# Refers to the system where the evaporator coils are placed in the ice box Ans. Cohesion
Ans. Direct system # Which of the following is NOT the cause of black smoke in diesel engine?
# What is the usual dehydrating agent in Freon system? Ans. high compression pressure
Ans. Activated alumina # Which of the following is not a method of starting a diesel engine?
# Refers to the system where the evaporator coils are indicated in a brine solution and brine is pumped Ans. Using another generator
thru the ice box # Two-stroke engine performs _____ to complete one cycle.
Ans. Indirect system Ans. compression and power stroke
# Is the most common dryer which consist of rotating cylinder inside which the materials flow # A type of geothermal plant used when there is a presence of brine extracted from undergroungd
while getting in contact with hot gas. Ans. Binary geothermal plant
Ans. Rotary dryer # Is the most important safety device on the power boiler.
# Is the ratio of the mass of water-vapor in air and mass of air if it is saturated is called: Ans. Safety valve
Ans. Relative humidity # During hydrostatic test, the safety valves should be
# The hands feel painfully cold when the skin temperature reaches Ans. removed
Ans. 10°C # Where deaerating heaters are not employed, it is recommended that the temperature of the
# The total heat of air is a function of feed water be not less than _______
Ans. WB temperature Ans. 197°C
# Boiling point of Freon-12 at atmospheric pressure is: # What is the possible cause of too much suction pressure?
Ans. 21°F Ans. Shortage refrigerant
# Which of the following is NOT a type of water cooled condenser in refrigeration? Where is the oil separator located?
Ans. Double shell Ans. Between the compressor and condenser
# Component of absorption refrigeration system in which the solution is cooled by cooling water. # What is the purpose of the low pressure cutout switch?
Ans. Absorber Ans. To cut compressor in and out at present pressure
# Cascade refrigeration cycle is often used in industrial process where objects must be cooled to # Freon unit will tend to short cycle when operating under
temperature below: Ans. Light loads
Ans. -46°C # What cause hot suction line?
# Type of refrigerant control designed to maintain a pressure difference while the compressor is Ans. Insufficient refrigerant
operating. # When changing a Freon system which valve is not used?
Ans. Capilliary tube Ans. King valve
# As a rule of thumb, for a specified amount of compressed air, the power consumption of the # Where is the scale trap located?
compressor decreased by ____________for each 3°C drop in the temperature inlet air to the Ans. Between the king valve and expansion valve
compressor. # Where is the solenoid valve located?
Ans. 1 percent Ans. Between the scale trap and thermal expansion valve
# Modern way of detecting air compressor leak is by using # Water tube boilers have how many fusible plugs?
Ans. 4 # The water vapour mass is often reported in
# What is the main cause of air pollution as a result of burning fuel? Ans. Grains of water
Ans. Nitrogen dioxide # Approximately how many grains of water are there in 1 pound?
# Refers to the increase of enthalpy of a substance when it undergoes some phase change at Ans. 7000
constant pressure and temperature. # The ratio of actual humidity ratio to the saturated humidity ratio at the inlet temperature and p
Ans. Heat of vaporization
# Which of the following keeps moisture from passing thru the system? Ressure is known as
Ans. Dehydrator Ans. Degree of saturation
# What is the lowest temperature to which water could possibly be cooled in a cooling tower? # What is another term for “degree of saturation”?
Ans. Temperature of adiabatic saturation Ans. Percentage of humidity
#What do you call the intake pipe to a hydraulic turbine from a dam? # The partial pressure of the water vapour divided by the saturation pressure?
Ans. Penstock Ans. Relative humidity
# An ideal fluid is one that # The point where sonic velocity has been achieved (M=1) is known as:
Ans. is frictionless and incompressible Ans. Critical point
# What principal was used by McLeod gauge which is used for low pressure measurement/ # The ratio of the thrust in equal mass flow rate is known as...
Ans. Boyles Law Ans. Effective exhaust velocity
#The difference between the brake and hydraulic power is # Is the velocity of gas dropped from supersonic to subsonic, gas will experience
Ans. Friction power Ans. Shock wave.
# Product of motor pump efficiency is known as # An adiabatic flow with friction, which would be approximately as flow through an insulated duct...
Ans. Over-all efficiency Ans. Fan no flow
# The most efficient cross-section of an open channel is the one that # What is an inevitable by product of the combustion of hydro carbon fuel?
Ans. All of the above Ans. Water vapour
#Refers to the structure across an open channel over which water flows. # A compound pressure gauge is used to measure:
Ans. Weir Ans. Positive and negative pressure
# When can we say that a trapezoidal weir is a “cipoletti” weir? # Which of the following best describes a Curtis turbine?
Ans. When side slope 4 vertical and 1 horizontal Ans. It is a velocity pressure compound turbine
# A weir is broad-crested if: # What causes low head pressure?
Ans. The weir length is greater than half of the head Ans. Too much cooling water and in sufficient refrigeration gas
# A feature of a dam over where water is discharged is called: # What causes high head pressure?
Ans. Spillway Ans. All of the above
# Laminar flow occurs when the Reynolds number is approximately less than # A commercial pipe which has the characteristics of being immune to electrolysis and corrosion, light
Ans. 2100 in weight weak structurally.
# Turbulent flow occurs when the Reynolds number exceeds: Ans. Asbestos cement pipe
Ans. 4000 # A commercial pipe which is durable, water tight, low maintenance and smooth interior.
# Laminar flow is typical when Ans. Concrete pipe
Ans. All of the above # A commercial pipe which has the characteristics of being corrosion, and scour and erosion
# When the Reynolds number is between 2100 to 4000, the flow is said to be in Resistance.
Ans. Critical zone Ans. Vitrified clay pipe
# Which of the following characterizes a laminar flow? # A commercial pipe which that is ductile, high strength, shock resistant, very smooth internally.
Ans. All of the above Ans. Steel pipe
# Without a pump and turbine, which of the following rules is true in a frictionless environment about a
energy and hydraulic gradient? # A commercial pipe that is chemically insert, resistant to corrosion, very smooth, light weigth and very
Ans. All of the above cheap
# What is the latent heat of vaporization of water? Ans. Plastic (PVC and ABS)
Ans. 970 Btu # Which commercial pipe that is primarily used for water condensate, and refrigerant lines easier to
# A dry bulb temperature at which water starts to condense but when moist appears in a constant bent by hand, good thermal conductivity
pressure process Ans. Brass and copper pipe
Ans. Dew point temperature # A fluid pathway that expose parts of the fluid to the atmosphere is the:
# A mixture of dry air and saturated vapour is known as: Ans. Open channel
Ans. Saturated air # The ratio of the area in flow to the width of the channel at fluid surface in an open channel is
# What is another term for “humidity ratio”? known as:
Ans. Specific humidity Ans. Hydraulic depth
# The most efficient cross-section for an open channel is known as: Ans. Hot-wire anemometer
Ans. Semi-circular section # Which of the following is used for R-12?
# The most efficient section rectangular section has: Ans. All of the above
Ans. A width twice the depth # Which of the following tubing (pipes) usually used in ammonia refrigerant?
# What is the maximum power that the motor can provide Ans. Ferrous pipe
Ans. Rated horsepower # Which of the following tubing (pipes) usually used in halo-carbon refrigerant?
# Which of the following is a possible cause of cavitation Ans. K and L copper tubing
Ans. All of the above # Properties of superheated vapour can be found in?
# Is a reaction during which chemical energy is released in the form of heat Ans. Superheated table
# When forebay is not part of the generating plant design, it will be desirable to produce a ________ in # Properties of non-reacting gas mixture are given by:
order to relieve the effect of rapid changes in flow rate. Ans. Volumetric weight for molecular weight and density and geometric weighing for all
Ans. Surge chamber the
#What is used to keep the turbine in a hydroelectric generating plant up to 15 feet above the tail properties except entropy.
water surface? # The relation between the total volume of a mixture of a non-reacting gases for initial volume is
Ans. Draft tube given by:
# If a draft tube is not employed, water may be return to the tail water by way of a chamber known as Ans. Amagat’s law
the: # Which of the following is the first definition of enthalpy?
Ans. Tail race Ans. The amount of useful energy in a system.
# An impulse turbine consists of a rotating shaft on which buckets of blades are mounted. What is # A consists press thermodynamics process obeys:
the term used for rotating shaft? Ans. Charles law
Ans. Turbine runner # The volume of an ideal is halved, while it’s temperature is double, what happens to the pressure?
# Which of the following refrigerant that belong to group B refrigerants and harmful or toxic Ans. Pressure is multiplied by 4
refrigerants? # A liquid boils when its pressure equal.
Ans. All of the above Ans. Ambient pressure
# Which of the following refrigerants are combustible? # A system composed of ice and water at zero °C is said to be:
Ans. All of the above Ans. All of the above
# Which of the following refrigerants is used for frozen foods and ice cream display cases # A heat of fusion for a pure substance is ________.
Warehouses and food freezing plants, medium temperature display cases truck refrigeration and heat Ans. The energy required to melt the substance.
pumps? #The heat of vaporization involves the change of enthalpy due to:
Ans. R-502 Ans. The change in phase from liquid to gas.
# The refrigerant that was formerly the most widely used for air conditioning and refrigeration. It # The heat of sublimation involves the change of enthalpy due to:
was the principal refrigerant for automotive air conditioning. Ans. The change in phase from solid to gas.
Ans. R-12 # The unit for absolute viscosity which is dyne-sec/cm3 is also known as:
# How do we know that there is shortage of refrigerant considering that we are in the liquid line? Ans. Poise
Ans. Vapor bubbles in the sight glass appear # What is the unit of absolute viscosity?
# The amount of power actually entering a fluid is known as: Ans. Pa-second
Ans. All of the above # The ratio of absolute viscosity to mass density is also known as:
#The input power to the pump will be: Ans. Kinetic viscosity
Ans. Equal to the output power of the electric motor driving the pump # What is the kinetic viscosity?
# A bent or curved carries fluid from a container at a high elevation to another container at a lower Ans. m2/s
elevation. # Stroke is a unit of kinetic viscosity which is equivalent to:
Ans. Siphon Ans. cm2/s
# A water path, usually a large diameter pipe used to channel water around or through an obstruction # What are volatile liquids?
feature. Ans. The liquid that vaporizes immediately
Ans. Culvert # When does boiling occur?
# Which of the following principles governs the distributions of flow between the two branches in a Ans. When the liquid temperature is increased to the point that the vapour present is equal to
pipe system. the local ambient pressure.
Ans. All of the above # The liquid boiling pressure is dependent on:
# A method used to determine the network flow in a multi-hoop piping system. Ans. Both A and B
Ans. Hardy-cross method # Referred as the mass flow the rate divided by the generator output in kilowatts
# A device used to measure velocity by determining the cooling effect of fluid flowing over an Ans. Water rate of stream rate
electrically heated tungsten. # The power after the auxiliary loads has been removed is known as:
Ans. Net electrical output # The hot-wire manometer is used to measure
# _________ of a turbine is the ratio of actual to ideal energy extraction? Ans. gas velocities
Ans. Both A and B # The pitot static tube measures
# What is the approximate maximum practical metallurgical limit on superheat? Ans. the static pressure
Ans. 1150°C and 625°C # The terminal velocity of a small sphere setting in a viscous fluid varies as the
# What is used to increased the mean effective temperature at which heat is added without producing Ans. inverse of fluid viscosity
significant expansion in the liquid-vapor region. # Pressure drag results from
Ans. Superheat Ans. skin friction
# If some of the heat energy from these waste product is recovered and used for space heating or # The pressure coefficient is the ratio of pressure forces to:
cooling the process is called Ans. inertia forces
Ans. Cogeneration # Tranquil flow must always occur
# In cogeneration, the recovered heat Ans. above the critical depth
Ans. Is used as heat # Which of the following head loss coefficient among the following types of entrance?
# The ratio of useful energy to the energy input is called? Ans. re-entrant
Ans. Fuel utilization # What waste water treatment method involves of algae from stabilization pond effluents?
# The ratio of the energy of a turbine to the recovered heat is known as: Ans. microscreening
Ans. Power to heat ratio # The number of nozzles will depend on the quantity of steam required by the turbine. If the
# If the recovered heat is used to vaporized water in a vapour cycle this is called: nozzles occupy the entire arc of the ring, the turbine is said to have:
Ans. Combined cycle Ans. full peripheral admission
# What is the best coolant for high temperature gas reactor? # Tandem compound units may also have two low-pressure casting that produces:
Ans. Helium (He) Ans. triple flow
# What is the liquid metal most frequently used as coolant, in liquid metal reactor? # By inter-cooling using two stage compressor of Brayton cycle, the backwork ratio will:
Ans. Sodium (Na) Ans. decrease
# Combustion power cycles differ from vapour power in that: # On dynamic similitude, the relation which represents the ratio of inertia force to pressure force is:
Ans. The combustion product cannot be returned into their initial condition for reuse. Ans. Euler number
# A closed system using a fixed amount of ideal air as the working fluid? # What is the maximum velocity in a sewer flowing full?
Ans. Air standard cycle Ans. 0.6 m/sec
# Which of the following is not a specific compound but is a mixture of octane and lighter # A temporary structures constructed to exclude water from the site of the foundation during its
hydrocarbon? excavation and construction is called:
Ans. Gasoline Ans. coffer dam
# A series of process that eventually brings the system back to its original condition is known as? # Which is not a physical characteristics of water?
Ans. Cycle Ans. hardness
# The _____ of a power cycle is defined as the ratio of usual work output to the supplied input # Which dam is best for weak foundation?
energy? Ans. buttress
Ans. Thermal efficiency # What is the volume of water which will drain freely from the aquifer?
# Is use to heat up the solution partially before entering the generator in absorption refrigeration Ans. specific yield
system. # What is the line defined by the water level in a group of artesian wells?
Ans. regenerator Ans. peizometric surface
# The COP of actual absorption refrigeration system is usually # Select the one that is a positive indication of pollution of a river.
Ans. less than 1 Ans. chloride content
# Sight glass is often located at: # Which instruments is used to measure humidity of the atmosphere continuously?
Ans. liquid line Ans. hydrograph
# Use to detects a vibration in current caused by the ionization of decomposed refrigerant between # Entrance losses between tank and pipe or losses through elbows, fittings and valves are generally
two opposite-charged platinum electrodes. expresses as a function of:
Ans. Electronic detector Ans. kinetic energy
# The ability of oil to mix with refrigerants # The air that contains no water vapour is called:
Ans. miscibility Ans. dry air
# For the submerged plane surface, the point on the surface where the resultant force acts is called
the
Ans. center of pressure # What effectively states that it is impossible to built a cylindrical engine that will have a thermal
# At any point in fluid at rest, the pressure is the same in all directions. This principle is known as: efficiency of 100%?
Ans. Pascal’s Law Ans. Kelvin-Planc statement of second law
# Refers to the maximum possible work that can be obtained from a cycle. Ans. Q positive in, W positive out and ∆U negative for decreased internal energy
Ans. Availability # The heat transfer term in the first law of thermodynamics may be due to any of the ff. except:
# The difference between the maximum and the actual work output is known as? Ans. Internal heat generation
Ans. Process inversibility # A system that experiences no mass crossing the system boundaries called:
# The study of the property of atmospheric air? Ans. Closed system
Ans. Psychrometry # A substance in which a substance is allowed to enter and have is most properly called:
# The temperature of the air that has gone through an adiabatic saturation process is known as? Ans. Open system
Ans. Wet-bulb temperature # First and second law of thermodynamics are:
# How often should compressor oil be changed? Ans. Energy equations
Ans. At least semi-annually # A constant temperature thermodynamic process obeys:
# Which of the following is the possible effect of the weak solution of brine in a refrigeration system? Ans. Boyles Law
Ans. Corrosion # A series of process that eventually bring the system back to the original condition is called a:
# In Freon refrigeration system, where is the scale trap located? Ans. Cycle
Ans. On the suction side of the compressor # If the refrigerant leaves the evaporator with a quality of < 1, the cycle is known as:
# At what pressure is the high pressure cut-off in the Freon (R-12) Ans. Wet vapor compression cycle
Ans. 125-150 psi # For even lower temperature, are required as in commercial freezing equip. What is used as
# What is the other name for brine refrigeration system? refrigerant?
Ans. Indirect refrigeration system Ans. Ammonia
# What is the indication that there is air in the refrigeration system? # What is another name for Air-Refrigeration cycle?
Ans. Unusual high head pressure Ans. Brayton cycle
# A Freon-12 leaking can be detected by halide torch. What color will it turn to in the presence of a # What is the major disadvantage of Air-Refrigeration cycle?
refrigerant if this torch has a normal blue flame? Ans. High power consumption
Ans. Green # What refrigeration is practically used when a large quantity of waste-inexpensive heat is avail?
# All of the following process are irreversible except Ans. Heat-driven refrigeration cycle
Ans. Inelastic tension and release of steel bar # For an absorption cycle, how many working fluids are required?
# Which of the following is a point function except Ans. Two (2)
Ans. Work # In absorption cycle, what is needed in NH3 water system to remove any remaining traces of
# All heat transfer process require medium of energy exchange except absorbents from the refrigerant and is placed between generator and condenser.
Ans. Radiation Ans. Rectifier
# Thermal conduction is described by: # Which is a “TEMA” stands for?
Ans. Fourier Law Ans. All of the above
# Convection is described by: # The acronym “ TEMA” stands for?
Ans. Newton’s Law Ans. Tubular Exchangers Manufacturing Association
# Radiant heat transfer is described by: # The acronym “API” stands for?
Ans. The Stefan-Boltzman Law Ans. American Petroleum Institute
# The equivalence of ratios of emissive power to absorptivity equilibrium is described by # What occurs when the exit temperature of the cold fluid is above the exit temperature of
Ans. Kirchoff’s Law the hot fluid?
# The temperature potential between temperature difference. Ans. Temperature cross
Ans. The logarithmic temperature difference # Are heat exchangers whose purpose is to heat with condensing stream.
# A thermodynamic process whose deviation from equilibrium is infinitesimal at all times. Ans. Closed feedwater heater
Ans. In quasi-equilibrium # Refers to the corrosion, precipitation of compound in solution, setting of particular solid
# A thermodynamic property best describes the molecular activity of a substance? and biological activity that adhere to a heat transfer
Ans. Internal energy Ans. Fouling
# The combination of conditions that best describes thermodynamic process is given: # Fouling in a heat exchanger industries is commonly known as:
Ans. Has successive states thru which the system passes Ans. Silent thief
# A substance whose properties are uniform throughout is called a: # An operation with 1 fluid flow rate substantially known as:
Ans. Pure substance Ans. Down turn
# All of the following are thermodynamic properties except: # Indicate the false statement?
Ans. Modulus Ans. Duo to viscosity, liquid cannot resist instantaneously change velocity
# A process that is adiabatic and reversible is also called: # Ideal liquid are assumed to be?
Ans. Isentropic Ans. Newtonian fluids
# The first law of thermodynamics for a closed system is Q= ∆U + W, the sign convention is: # Which of the following is a Newtonian fluids?
Ans. All of the above Ans. energy
# Thermal radiation heating body can be absorb, reflected or transmitted, this is known as # Is a process during which the system remains in equilibrium at all times
Ans. Radiation Conservation law Ans. Quasi-equilibrium
# The rate of thermal emitted per unit area of a body is known as: # In the absence of any work interactions between a system and its surrounding, the amount of
Ans. Emissive power net heat transfer is equal
# States that for any two bodies in thermal equilibrium, the ratio of emissive power to Ans. to the change in the total energy of a closed system
absorptivity are # The constant volume and constant pressure specific heats are identical for
Equal. Ans. incompressible substance
Ans. Kirchoff’s Radiation law # The velocity of fluid is zero at the wall and maximum at the center because of the
# Bodies that radiate at absorptivity of 1 are known as: Ans. viscous effect
Ans. Black bodies or ideal radiators # For steady flow devices, the volume of the control volume is
# Which of the following does not radiate at the ideal level? Ans. constant
Ans. Real body # The work done in a turbine is _____ since it is done by the fluid.
# The ratio of actual to ideal emissive power is known as: Ans. positive
Ans. Emissivity # Reheating process in Brayton cycle, the turbine work will
# What is the refrigerator’s main function? Ans. increase
Ans. To cool this low temperature area # As the air passes through a nozzle, which of the following will increase?
# The rate of energy removal from the low temperature area is known as: Ans. mach number
Ans. Either of the refrigeration capacity or refrigerating effect # As the air passes through a diffuser, which of the following will decrease?
# The refrigeration capacity is measured in refrigeration ton where one ton is _____ heat Ans. mach number
remove # As the air passes through a nozzle, which of the following will decrease?
Ans. All of the above Ans. internal energy
# The refrigeration ton is derived from the heat flow required to melt the ton of ICE in: # As the air passes through a diffuser, which of the following will increase?
Ans. 24 hours Ans. density
# In psychrometric chart, the constant- enthalpy lines coincide with constant-temperature # As the air passes through a diffuser, which of the following will NOT be affected?
lines at temperature Ans. entropy
Ans. below 50°C # After passing through a convergent-divergent nozzle, the temperature of air will:
# The amount of moisture in air depends on its Ans. decrease
Ans. temperature # After passing through a convergent-divergent nozzle, the density of air will:
# The deep body temperature of healthy person is maintained constant at Ans. decrease
Ans. 37°C # After passing through a convergent-divergent nozzle, the mach number of air will:
# Air motion also plays important role in Ans. decrease
Ans. human comfort # By increasing the temperature source of Carnot cycle, which of the following will not be
# During simple heating and cooling process has a_____ humidity ratio. affected?
Ans. constant Ans. heat rejected
# The ________ follows a line of constant wet-bulb temperature on the psychrometric chart. # By decreasing the temperature sink of Carnot cycle, which of the following will not be affected?
Ans. evaporative cooling process Ans. heat added
# A vapor which is not about to condense is called a Ans. Exothermic reaction
Ans. superheated vapor # By reheating the steam in an ideal Rankine cycle the heat rejected will:
# Passing from the solid phase directly into vapor phase is called Ans. increase
Ans. sublimation # By increasing the boiler pressure in Rankine cycle the moisture content at boiler exit will:
# Robert Boyle observed during his experiments with a vacuum chamber that the pressure of Ans. increase
gases is inversely proportional to their # Presently the highest steam temperature allowed at the turbine inlet is about ______.
Ans. volume Ans. 620°C
# _____ is energy in transition. # Two most common gases employed in Stirling and Ericson cycles are:
Ans. Heat Ans. Hydrogen and helium
# Is the mode of energy transfer between a solid surface and the adjacent liquid or gas which is in # In most common designs of Gas turbine, the pressure ratio ranges from:
motion, and it involves combine effects of conduction and fluid motion. Ans. 11 to 16
Ans. convection # In Brayton cycle, the heat is transformed during what process?
# Radiation is usually considered as Ans. isobaric process
Ans. surface phenomenon # The fuel injection process in diesel engine starts when the piston_______.
# Work is _____ between the system and the surroundings. Ans. approaches TDC
# If the cut-off ratio of diesel cycle increases, the cycle efficiency will: Ans. Rankine vapor cycle
Ans. decrease # What can be considered to be theoretically exact and can be used to determine local thermal film
# The fuel used in a power plant that is used during peak periods. coefficient at a distance X from the leading edge of an isothermal plate in laminar flow?
Ans. liquid Ans. Pohlhausen Solution
# Typical compression ratio of Otto cycle is # Which one is exact and can be derived from the Pohlhausen correction by setting cube root of
Ans. 8 Prandtl number equal to 108 and is useful in gases and water?
# If Joule Thompson coefficient is equal to zero, then the process will become Ans. Blausius Solution
Ans. isothermal # What another term for skin friction coefficient?
# If the fluid passed through a nozzle its entropy will: Ans. Fanning friction factor
Ans. remains the same # The Darcy friction factor is ________.
# Which of the following is NOT a possible location of service valve? Ans. 4 times
Ans. outlet of condenser # The ______ for submerged bodies in the sum of the skin friction coefficient is?
# A coil in series with evaporator that is use to prevent the liquid refrigerant entering the Ans. Drag coefficient
compressor. # What can predict the average film coefficient along the entire length of laminar flow?
Ans. Drier loop Ans. Labarsky-Kauffman Correlation
# A type of valve connected from discharge of compressor directly to suction that is normally # When can we say that the laminar flow is “Fully Developed”?
closed Ans. When the difference between the surface (wall) and the mean fluid temperature is
And will open automatically only if there is high discharge pressure. Constant
Ans. solenoid valve # Where is the accumulator usually located?
# Use to increase the capacity of condenser. Ans. Before the suction line
Ans. desuperheating coils # A device used to remove moisture and foreign particles inside the refrigeration system.
# Is use to subcooled the refrigerant from the condenser. Ans. Filter drier
Ans. liquid subcooler # What is the standard value of the atmospheric pressure at the atmospheric sea level?
# Which of the following is NOT a part of low pressure side in refrigeration system? Ans. 14.7 psi
Ans. liquid line # Requirements are classified according to:
# Which of the following is NOT a part of high pressure side in refrigeration system? Ans. Manner of absorbing heat
Ans. suction line # A push or pull on any thing is called:
# Which of the following is NOT a part of condensing unit? Ans. Force
Ans. liquid line # Anything that takes up space has mass
# By subcooling the refrigerant in refrigeration system, the compressor power per unit mass will Ans. Matter
Ans. remains the same # Factor use in calculating the overall heat transfer through the tube walls of the condenser tube of
# Superheating the refrigerant in refrigeration system without useful cooling, the refrigeration Another heat transfer surface.
effect Ans. Fouling factor
Per unit mass will # What could be the possible cause of the abnormal discharge temperature?
Ans. remains the same Ans. All of the above
# By subcooling the refrigerant in refrigeration system, the specific volume at compressor suction # Where is the relief valve located?
will Ans. At the discharge side of the compressor
Ans. remains the same # What will you do before starting a refrigeration system?
# Pressure loss due to friction at the condenser, the compressor power per unit mass will Ans. Vent the condenser
Ans. remains the same # What is the purpose of the expansion valve bypass?
# Which of the following is NOT a type of air-cooled condenser? Ans. To control the refrigerant to the evaporator in case the automatic valve fails.
Ans. shell and tube # How many feed water lines are connected to the boiler?
# A type of refrigerant control typically used in household refrigeration. Ans. 2
Ans. Capilliary tube # Exhaust gases from the engine possesses what type of energy?
# Type of condenser that operates like a cooling tower. Ans. Kinetic energy
Ans. evaporative condenser # What is the instrument used to measure density?
# The major problem of heat pump is Ans. Hydrometer
Ans. frosting # Engine overall efficiency is ________.
# Dominant refrigerant used in commercial refrigeration system Ans. Brake thermal efficiency
# The Carnot refrigeration cycle is: # What is the ratio of the fuel usage rate to the power generated?
Ans. All of the above Ans. Specific fuel consumption
# The vapour compression cycle is essentially a reverse of: # What is the ratio of the air mass that enters the engine to each mass of fuel?
Ans. Air-fuel ratio Ans. Loss of boiler efficiency
# The ohmmeter is an instrument used to measure # How are tubes secured in a fired tube boiler will result in:
Ans. Resistance Ans. Rolled and beaded over
# The ammeter is an electrical instrument used to measure # A tube in a fire tube boiler is surrounded by:
Ans. Current Ans. Water
# The electrical pressure that pushes the electrical current or objections. # What is the formula for the factor of evaporation?
Ans. Voltage Ans. H-b/970.3
# The voltmeter is an instrument used to measure: # What can cause a super heater tube to overheat?
Ans. Voltage Ans. Dirty tube
# When the winding is shorted, what is the resistance reading? # How many square feet of heating surface does a 3-in fire tube 20 ft long with a 3/16 in . wall
Ans. Zero ohm have?
# Which tools are used to enlarge the end of the tubing to connect a fitting? Ans. 13.7
Ans. Flaring tools # In a high temperature hot water boiler; what could cause a tube to over heat?
# The tubing is never used when the fluid temperature goes beyond 100 to 175 mm? Ans. Poor circulation
Ans. Plastic # A down corner is:
# The tubing to be swaged is damped in a Ans. Found in a water tube boiler
Ans. Flaring block # Extreme firebox temperature changes cause:
# The hand tools used to lightened and loosen the fitting are Ans. Spalling
Ans. Wrenches # Low water cu. off:
# In soldering process, What substance is used to prevent oxidation? Ans. Shuts the burner when the boiler is low on water
Ans. Flux # Steam coming from the bottom by cock would indicate:
# The process of applying molten metal to heated metals is called: Ans. Low water level
Ans. Soldering # Why is the method to cool water called condenser not practical?
# What is the physical state of the refrigerant? Ans. Water is expensive
Ans. Gas # A device use to collect liquid refrigerant entering the compressor to prevent trouble in
# Which of the following appliances uses a finned-coil evaporator? compressor is
Ans. Window-type airconditioner Ans. Accumulator
# The operation of condenser is the opposite of that of # A disiccant is a material that has high affinity to water. Which is the common used dissicant
Ans. Evaporator Ans. Silica gel
# Which of the following devices maintains the pressure difference between the evaporator and # The only means of preserving food in its original fresh state is _____________.
the condenser? Ans. Refrigeration
Ans. Flow control # PVC means
# What device is ideal for measuring the flow of liquid metal? Ans. Poly Vinyl Chloride
Ans. Magnetic flow meter # What does ABS means?
# The frictional force that acts parallel but opposite to the direction of motion. Ans. Acrylonitrite Butadiene Styrene
Ans. Drag # What measures the average roughness of imperfection inside the pipe?
# A pump that transform kinetic energy into fluid static pressure. Ans. Specific roughness
Ans. Kinetic pump # Darcy factor, friction factor is not constant but decreases as the Reynolds number increases
# Due to friction losses between the fluid and the pump and mechanical losses in the pump itself. up to a certain point known as:
Ans. The brake pump power will be greater than hydraulic power Ans. Full turbulent flow
# The net energy actually transferred to fluid per unit time is: # What presents the friction factor graphically as 6 functions of Reynolds number and
Ans. Hydraulic power relative roughness.
# The input power delivered by motor to pump. Ans. Moody diagram
Ans. Brake pump power # If the flow in truly laminar, and fluid flowing in a circular pipe then which of the equation
# Boiler tubes are specified by is appropriate to use?
Ans. Outside diameter Ans. Hagen Poisenille Equation
# The maximum size of the boiler down lines valves and fittings shall not exceed # Which is true about Hagen Williams equation?
Ans. 2.5 inches Ans. It is primarily use for water
# The purpose of an expansion tank in a hot water heating system is to provide for the expansion # For highly turbulent flow, what causes shear stress?
of Ans. Momentum Effect
Ans. Water # In a circular pipe laminar flow momentum flux is:
# Heavy accumulation of a soot in the boiler will result in: Ans. Maximum at the pipe wall
# If the head pressure is too high # By sub-cooling the refrigerant in 50apour compression cycle at condenser exit, which of the
Ans. The high pressure cut-out switch should operate following will increase? (Use per unit mass analysis)
# In a combustion engine cycle, what is the ratio of the net output power to the input? Ans. Refrigerating effect
Ans. Thermal efficiency # By sub-cooling the refrigerant in 50apour compression cycle at condenser exit, which of the
# In a reciprocating engines, what is the ratio of actual to ideal volumes of entering gases? following will decrease? (Use per unit mass analysis)
Ans. Volumetric efficiency Ans. Mass flow rate
# In a combustion engine, what is the ratio of the actual power developed to the ideal power # By increasing the vaporizing temperature in vapor compression cycle, which of the following
developed? will increase? (Use per unit mass analysis)
Ans. Mechanical efficiency Ans. COP
# The equivalent of ratio of emissive power to absorptivity for both thermal equilibrium is # By increasing the vaporizing temperature in 50apour compression cycle, which of the
described following will decrease? (Use per unit mass analysis)
by: Ans. temperature difference between evaporator and compressor
Ans. Kirchoffs law # By increasing the condenser pressure in 50apour compression cycle, which of the following
# The ideal cycle based on the concept that the combination process in both diesel and will increase? (Use per unit mass analysis)
gasoline in Ans. Compressor power
the combination of heat transfer process that is constant pressure and constant volume. # By increasing the vaporizing temperature in 50apour compression cycle, which of the
Ans. Dual cycle following will decrease? (Use per unit mass analysis)
# The ratio of the volume at the end of heat addition to the volume at the start of heat Ans. moisture content after expansion
addition is called: # If the pressure drop in the condenser increases in a vapor compression cycle, which
Ans. Cut-off ratio of the following will increase? (Use per unit mass analysis)
# A theoretical body which when heated to incandescence would emit continuous light Ans. heat rejected in the condenser
ray spectrum. # If the pressure drop in the condenser increases in a 50apour compression cycle,
Ans. Blackbody which of the following will decrease? (Use per unit mass analysis)
# Which of the following is the reason for insulating the pipes? Ans. mass flow rate
Ans. Heat loss from the surface is minimize # If the pressure drop in the condenser increases in a 50apour compression cycle,
# Heat transfer due to density differential which of the following will not be affected? (Use per unit mass analysis)
Ans. Convection Ans. compressor power
# A process involving a gas that cannot, even in principle be reserved with being done. # If the pressure drop in the evaporator increases in a 50apour compression cycle,
Ans. An isobaric compression which of the following will increase? (Use per unit mass analysis)
# The maximum amount of heat that can be converted into mechanical energy Ans. heat rejected in the condenser
Ans. Depends on the intake and exhaust temperature # If the pressure drop in the condenser increases in a 50apour compression cycle,
# The maximum amount of mechanical energy converted into heat which of the following will decrease? (Use per unit mass analysis)
Ans. 100% Ans. COP
# The work output of energy heat engine. # By lowering the condenser pressure in Rankine cycle, which of the following will
Ans. Equals the difference between its heat intake and heat exhaust decrease? (Use per unit mass analysis)
# To increase the output of a centrifugal pump, you must Ans. heat rejected
Ans. Speed of rotation # By increasing the boiler pressure in Rankine cycle, which of the following will
# By superheating the refrigerant in vapor compression cycle with useful cooling, which decrease? (Use per unit mass analysis)
of the following will increase? (Use per unit mass analysis) Ans. heat rejected
Ans. Heat rejected from condenser # By superheating the steam to a higher temperature in Rankine cycle, which of the
# By superheating the refrigerant in vapor compression cycle with useful cooling, which following will decrease? (Use per unit mass analysis)
of the following will decrease? (Use per unit mass analysis) Ans. moisture content at the turbine exhaust
Ans. Mass flow rate # By superheating the steam to a higher temperature in Rankine cycle, which of the
# By superheating the refrigerant in vapor compression cycle with useful cooling, which following will increase? (Use per unit mass analysis)
of the following will decrease? (Use per unit mass analysis) Ans. cycle efficiency
Ans. COP # By superheating the steam to a higher temperature in Rankine cycle, which of the
# By superheating the refrigerant in vapor compression cycle with useful cooling, which following will decrease? (Use per unit mass analysis)
of the following will increase? (Use per unit mass analysis) Ans. pump work
Ans. Compressor power # What type of boiler incorporates furnace water cooling in the circulatory system?
# By superheating the refrigerant in vapor compression cycle with useful cooling, which Ans. Integral-furnace boiler
of the following will not be affected? (Use per unit mass analysis) #The main components of a combined cycle power plant are:
Ans. Compressor power Ans. Gas turbine and waste heat boiler
# Which of the following indicators is used to determine the anti-knocking characteristics of Ans. Down in the direction of the flow
gasoline? #The presence of minor loss in the energy grade line will cause the line to slope
Ans. Octane number Ans. Down in the direction of the flow
# Indicate the false statement # If the Mach number is greater than 1 but less than 5
Ans. The heat transfer cannot exceed the work done. Ans. Supersonic
# In fluid flow, linear momentum is: # The flow is called sonic when mach number is
Ans. A vector quantity equal to the product of mass and velocity Ans. Equal to 1
# A fact that a fluid’s velocity increases as the cross-sectional area of the pipe through which it # The flow is sub-sonic when mach number is
flow decreases due to: Ans. Less than 1
Ans. The continuity equation # To check water level in the gauge glass of a steam boiler
# The coefficient of contraction is the ratio of: Ans. Use the
Ans. Area of vena contracta to the orifice area # The flame failure control is tested by?
# The coefficient of discharge is the ratio of: Ans. Shutting off the fuel supply to the burner
Ans. Actual discharge to the theoretical discharge # During purge cycle what is the minimum amount air volume changes required?
# The coefficient of velocity is the ratio of: Ans. Four (4)
Ans. Actual velocity divided the theoretical velocity # The purge cycle time for gas compared to oil is:
# Flow measuring devices include all of the following except: Ans . The same
Ans. Magnetic dynamometers #A high (CO) Carbon monoxide reading, indicates:
# In the series pipe systems, all of the following parameters vary from section to section except Ans. None of these
Ans. Mass flow # The induced draft fan is located:
# The coefficient of velocity accounts for the: Ans. In the breaching
Ans. Small effect of friction and turbulence of the orifice # A balanced draft boiler required:
# Expansion factors take into account the Ans. Both forced and induced draft
Ans. Effects of compressibility # What percent would expect in a well maintain boiler burning number 6 oil?
# The water hammer phenomenon is primarily what kind of fluid mechanics? Ans. 15%
Ans. Dynamic (a time-dependent phenomena) # Laminar friction factor of fluid is flowing through a pipe is a function of all the following
# All of the following are forms of drug on a body moving through a fluid except: except:
Ans. D’Alembert’s paradox drug Ans. Pipe roughness
# The function of a turbine is to: # The stream function is a useful parameter in describing:
Ans. Extra energy from the flow Ans. Conservation of mass
# The fact that there is no bodies moving through an ideal fluids is known as: # The study of the practical law of fluid and the resistance of open pipes and channels is the
Ans. D’Alembert’s paradox _____
# Liquids and gases takes the following characteristics of their contents. Ans. Hydraulics
Ans. Shapes # The most common methods of calculating frictional energy loss for laminar flow fluids in
# All of the following dimensionless parameters are applicable to fluid flow problem excepts non circular pipes:
Ans. Bolt number Ans. Equation
# All of the following can be characteristics of fluids except: # The parameter in the expression for head loss is:
Ans. Hysteresis Ans. Darcy friction factor
# The most common method for calculating frictional energy loss for laminar flowing fluid is # The characteristic length of Reynolds number use to calculate the friction in non-circular full
non- circular pipes is: running pipes is based on the:
Ans. The Darcy equation Ans. Hydraulic diameter
# For computation convenience, fluids are usually classed as # The hydraulic radius of a non-circular pipe is:
Ans. Real and ideal Ans. The ratio of flow area to wetted parameter:
# Which of the following is not a characteristics of real fluids? # An electromagnetic radiation wavelength is in the 0.1 to 10.000 mm range:
Ans. Experience of eddy currents and turbulence Ans. Thermal radiation
# Property of a fluid whereby its own molecules are attracted is known as: # What characteristics makes the difference between a Newtonian and non-newtonian fluids?
Ans. Cohesion Ans. Their viscous behaviour
# The term subsonic flow refers to a flowing gas with a speed. # What instrument use to measure salt solution:
Ans. Less than the local speed of sound Ans. Salimeter
# The difference between stagnation pressure and total pressure # A measure of fluid resistance to flow?
Ans. None of the terms are interchangeable Ans. Viscosity
# The presence of friction in the hydraulic grade line will always cause the line to slope. # What is another for absolute viscosity?
Ans. Dynamic # The work done in an adiabatic (isentropic, process is _____ the work done by the isothermal
# What is the reciprocal of viscosity? process;
Ans. Fluidity Ans. Less than
# The density of fresh water is a ship will float. # The work done in an isentropic compressor is ____ the work done by the isothermal
Ans. Lower in fresh water than in sea warer compressor;
# Bernoulli’s equation is based on? Ans. Greater than
Ans. Conservation of energy # The term used to describe a partially compressed gases withdrawn cooled and compressed
# An express train goes past a station flatform at high speed a person standing in the edge of further;
the flatforms tend to be: Ans. Intercooling
Ans. Attracted to the train # A perfect intercooling refers to the case where the gases cooled to:
# The volume of fluid flowing per second out of an orifice at the bottom. A tank does not Ans. The original inlet temperature
depend on: # Multi stage blowers can reach pressure up to ____ of water;
Ans. Density of fluid Ans. 100 inches
# Addition of detergent to water? # What represents the ratio of the coefficient of the friction at the expanded in pumping a
Ans. Decreases its surface tension fluid;
# Water neither rise or falls in silver capillary. This suggest that the contact angle between the Ans. Fried heat transfer efficiency factor
water and silver is: # What is the proper oil storage temp for No. 6 oil?
Ans. 90 degrees Ans. 20 ºF above pour point
# Most pressure in the Freon system have two dials or graduation on 1 gage what does the two # To prime a pump
dial represent: Ans. Fill casing with water
Ans. Temperature and pressure # Balance draft means;
# The flame detector is directed: Ans. Almost atmosphere pressure in the furnace
Ans. Into the furnace area # A gallon of No. 6 oil contains:
#Which of the following statement is true? Ans. 152,000 Btu
Ans. All of the above # What is the efficiency of a steam plant with a heat rate of 12,000 Btu/KW?
#Which of the following is a pseudo plastic fluid? Ans. 28.4%
Ans. All of the choices # What type of fuel must be preheated to burn properly?
# A type of fluids which are capable of indefinitely resisting a small shear stress but move Ans. Heavy oil
easily when the stress becomes large: # The fuel oil for mechanical atomizing burners is usually heated
Ans. Bingham fluids Ans. 150 ºF
# Which of the following is an example of Bingham fluids? # Fuel suppliers specify a minimum flash point for their oil because a low flash point oil is;
Ans. All of the above Ans. Dangerous
# The Bingham fluids will become pseudo plastic when: # How many cubic feet of furnace volumes are needed to burn one gallon of No. 6 oil?
Ans. The stress increases Ans. 10
# A type of fluids which exhibits viscosities that increases with an increasing velocity gradient. # A fuel oil is heated its viscosity
Ans. Dilatant fluid Ans. Decreases
# The viscosity decreases with time the fluids is said to be: # Only ____ of the turbine work output is required to operate the pump.
Ans. Thixotropic fluid Ans. 0.04%
# Viscosity increases with time the fluids is said to be: # Superheating the steam to higher temperature decreases the moisture content of the steam at
Ans. Rneopectic fluids the __
# What is the dominating cause of viscosity in fluids: Ans. Turbine exit
Ans. Molecular cohesion # Regeneration also provides a convenient means of dearating the feedwater to prevent.
# In a compressor, the piston is said to be at _____ when it is at its most petracted position in Ans. Boiler corrosion
the stroke, # Can be apply in Steam turbine cycle (Rankine), Gas turbine cycle (Brayton) and Combined
Ans. Bottom dead center cycle.
# Reciprocating compressors are characterized by their Ans. Cogeneration plant
Ans. Clearance volume # In a Rankine cycle with fixed turbine inlet conditions. What is the effect of lowering the
# The gases remaining in the clearance volume after the discharge in the dead center are condenser pressure, the heat rejected will;
known as the: Ans. Decrease
Ans. Residual gases # In an ideal Rankine cycle with fixed boiler and condenser pressures. What is the effect of
# The ratio of the clearance volume to the swept volume is known as: superheating the steam to a higher temperature, the pump work input will;
Ans. Percent clearance or just clearance Ans. Remains the same
# How do the following quantities change when the simple ideal Rankine cycle is modified # The ratio of total dissolve solids in the recirculating water to the total solids in the make-up
with regeneration? The heat rejected will; water.
Ans. Decreases Ans. Cycles of concentration
# During a combustion process, the components which exist before the reaction are called # What is the another term for Bryton cycle:
Ans. Reactants Ans. Joule cycle
# Is an obvious reason for incomplete combustion. # The process of transferring heat from low temperature area to high temperature area;
Ans. Insufficient oxygen Ans. Refrigeration
# Higher heating value when H2O in the product of combustion is in #The device to remove heat from water:
Ans. Liquid form Ans. Chiller
# Device which transfer heat from low temperature medium to a high temperature one is a # What device which transfer heat from low temperature area to operate on refrigeration
Ans. Heat pump system:
# A rule of thumb is that the COP improves by _____ for each °C the evaporating temperature Ans. Heat pump
is raised or the condensing temperature is lowered. # Refers to the process of one substance mixing with another substance:
Ans. 2 to 4% Ans. Diffusion
# Are generally more expensive to purchase and install than other heating systems, but they # Which of the following statement about Newtonian fluid is most accurate:
save money in the long run, Ans. Shea stress is proportional to the rate of strain
Ans. Heat pumps # Which is not characteristic of fluids?
# The most widely used absorption system is the ammonia-water system, where ammonia Ans. Hysteresis
serves as refrigerant and H2O as the # The relationship between pressure and altitude in the atmosphere:
Ans. Transport medium Ans. Barometric height relationship
# Known as drum less boiler. # Flash point means:
Ans. Once-through boiler Ans. Ignition temperature
# Reduces the steam temperature by spraying low temperature water from boiler drum. # Fire point means:
Ans. Desuperheater Ans. None of the above
# Carbon dioxide can be removed by; # Bernoulli’s equation is a
Ans. Aeration Ans. Conservation of energy equation
# Is often used to absorb silica from water. # The pressure at given depth due to several immiscible liquid is:
Ans. Magnesium hydroxide Ans. The sum of individual pressure
# Presence of excess hydrogen ions makes the water # The relationship between pressure and altitude in the atmosphere is given by the
Ans. Acidic Ans. Barometric height relationship
# PH of water varies with # The fact that the buoyant force on a floating object is equal to the weight of the water
Ans. Temperature displaced
# PH value of ____ is usually maintained for boiler water to minimize corrosion. Ans. Archimedes principle
Ans. 10.5 # Which of the following terms does not appear in the steady flow energy equation in the
# What type of turbine that has a degree of reaction of 1/2? extended Bernoulli’s equation
Ans. Rarsons turbine Ans. Hysteresis losses
# The cooling water is made to fall in series of baffles to expose large surface area for steam # The pitot tube can be use to measure fluid velocity as described by the Bernouilli equation
fed from below to come in direct contact. and the relationship between;
Ans. Barometric condenser Ans. Kinetic energy and static pressure
# Show the variatiom of river flow (discharge) with time. # The difference between stagnation pressure and total pressure is;
Ans. Hydrograph Ans. None. The terms are interchangeable
# The pressure at the bottom of a vessel filled with liquids does not depend on the; # Fully turbulent flow in a pipe is characterized by all the following except:
Ans. Area of the liquid surface Ans. Parabolic velocity profile
# A person stands on a very sensitive scale, inhales deeply the reading on the scale: # What is the bulk modulus of the water is;
Ans. Depends on the expansion of the person’s chest, relative to the volume inhaled. Ans. 300,000 psi
# Buoyancy occurs because, with increasing depth in a fluid: # Atmospheric pressure does not correspond to approximately;
Ans. The pressure increases Ans. 98 N/m2
# In order for an object to sink when placed in water its average specific gravity must be; # Hydraulic press is able to produce a mechanical advantage because;
Ans. More than 1 Ans. An external pressure extended on a fluid is transmitted uniformly through its
# The average solar energy heating the outer edge atmosphere is approximately 442 Btu per volume
hour-foot squared of 1.41KW per meter squared is known as: # In the operation of a hydraulic press, it is impossible for the output piston to exceed the input
Ans. Solar constant piston
Ans. Work # The following are all examples of indirect (secondary) measurements to measure flow rates
# If one of the management legs is inclined it is known as: using flow meters except;
Ans. Draft gauge Ans. Positive displacement meters
# Why are manometer tube generally large in diameter? # In series pipe systems which of the following parameters vary from section to section
Ans. To avoid significant capillary effect except;
# Bernoulli’s equation is an energy conservation based on several reasonable assumptions: Ans. Mass flow
Ans. All of the above # Venturimeter, pitot static gauges, flow nozzles, manometer vary with;
# Body that emits a constant emissivity, regardless of the wavelength; Ans. Flow velocity and pressure
Ans. Gray body # Other name for R 729?
# What gives the total emissive power from a black body Ans. Air
Ans. Stefan-Boltzmann law of the fourth power # In an indirect refrigeration system, which of the following is used to measure the density of
# What accounts for the spatial arrangement of the body and their emissivity; brine?
Ans. Emissivity factor Ans. Hydrometer
# The gray body shape factor is the product of the # Which of the following is the result of opening a hand expansion valve too much in a
Ans. Black body shape factor and emissivity factor refrigeration system?
# The product of the area and the shape factor is known as; Ans. Evaporator will “freeze back” to compressor
Ans. Geometric flux If one of the manometer legs is inclined, it is known as:
# Surfaces that re-radiates absorb thermal radiation are known as; Ans. Draft gauge
Ans. Refractor materials of refractories Why are monometer tubes generally large in diameter?
# Pitot tube used to measure Ans. To avoid significant capillary effect
Ans. Velocity Bernoulli’s equation is an energy conservation based on several reasonable
# Liquid gases take the following characteristics of the containers: assumptions:
Ans. Shape Ans. All of the above
# For computational convenience usually classed as: Body that emits a constant emissivity, regardless of the wavelength:
Ans. Real and ideal Ans. Gray body
# Which of the following statement about Newtonian fluid is more accurate? What gives the total emissive from a black body:
Ans. Shear stress is proportional to the rate of strain Ans. Stefan-boltzman law of the fourth power
# Which of the following is not a characteristic of a real of fluid? What accounts for the spatial arrangements of the body and their
Ans. Experience of eddy current and turbulence emissivity?
# One could expect the possibility of Reynold’s number similarity in all of the following cases Ans. Emissivity factor
except The gray body shape factor is the product of the
Ans. Closed pipe turbulent flow Ans. Black body shape factor and emissivity factor
# One could expect the possibility of Froude number similarity in all of the following cases The product of the area and the shape factor is known as:
except Ans. Geometric flux
Ans. Subsonic airflows Surfaces that re-radiates absorb thermal radiation are known as:
# The absolute viscosity of a fluid varies in pressure and temperature as a function of; Ans. Refractory materials of refractories
Ans. Shear and angular deformation rate Pito- tube used to measure
# Turbulent flow of a fluid in a pipe. All of the following except; Ans. Velocity
Ans. Reynold’s number will be less than 2300 Liquid gases take the following characteristic of the containers:
# Flow measuring devices include all of the following except; Ans. Shape
Ans. Magnetic dynamometer For computation convenience usually classed as:
# Flow measuring devices include all of the following except; Ans. Real and ideal
Ans. Mercury barometer Which of the following statement about Newtonian fluid is more accurate?
# Flow measuring devices include all of the following except; Ans. Shear stress is proportional to the rate of strain
Ans. Precision tachometer Which of the following is not a characteristic of a real fluid?
# The following are all examples of indirect (secondary) measurements to measure flow rates Ans. Experience of eddy current and turbulence
using obstruction meters except; One could expect the possibility of Reynold’s number similarity in all of the
Ans. Volume tanks following cases except
# The following are all examples of indirect (secondary) measurements to measure flow rates Ans. Closed pipe turbulent flow
using velocity meters except; One could expect the possibility of Froude number similarity in all of the
Ans. Weight and mass scales following cases except
Ans. Subsonic airflows
Difference between energy grade line (friction) and the energy line friction Ans. End of combustion
loss is: In a compression ignition engine, the detonation occurs near the
Ans. Friction and minor losses _____________.
The absolute viscosity of a fluid varies in pressure and temperature as a Ans. Beginning of combustion
function of: Morse test is use to measure the _________ of multi-cylinder engine.
Ans. Shear and angular deformation rate Ans. Indicated power
Turbulent flow of a fluid in a pipe. All of the following are true except: Ignition delay can be minimized by adding ______ to decrease engine
Ans. Reynold’s number will be less than 2300 knocking.
Flow measuring devices include all of the following except: Ans. Ethyl nitrate
Ans. Mercury barometer The work done in throttling valve is
Flow measuring devices include all of the following except: Ans. Zero
Ans. Precision tachometer Stagnation enthalpy represents the enthalpy of a fluid when it is brought to
The following are all examples of indirect (secondary) measurements to rest ______.
measure flow rates using obstruction meters except: Ans. Adiabatically
Ans. Volume tank Represents the temperature an ideal gas attains when it is brought to rest
The following are all examples of indirect (secondary) measurements to adiabatically.
measure flow rates using velocity meters except: Ans. Stagnation temperature
Ans. Weight and mass scales After passing through a nozzle the density of fluid decreases as the fluid
The following are all examples of indirect (secondary) measurements to velocity.
measure flow rates using flow meters except: Ans. Increases
Ans. Positive displacement meter The volume flow passes through a venture meter will
In series pipe systems which of the following parameters vary from section Ans. Constant
to section except: A converging-diverging nozzle is the standard equipment in
Ans. Mass flow Ans. Supersonic aircraft
Venturimeter, pitot static gauges, flow nozzles, manometers vary with: Nozzles efficiencies range from
Ans. Flow velocity and pressure Ans. 90%-99%
Other name for R 729? By reheating the steam before entering the second stage in Rankine cycle,
Ans. Air which of the following will increase?
In an indirect refrigeration system, which of the following is used to Ans. Heat rejected
measure the density of the brine? Contains 90% gasoline and 10% ethanol
Ans. Hydrometer Ans. Gasohol
Which of the following is the result of opening a hand expansion valve too In an evaporator section in refrigerating unit, which of the following is the
much in a refrigeration system? function of evaporator?
Ans. Evaporator will “freeze back” to compressor Ans. Absorption of latent heat of vaporization
Types of turbine used up to 300m head. Which of the following may not be the type of bulb in R-12 refrigeration
Ans. Deriaz turbine system?
A turbine that has a diagonal flow. Ans. Duplex
Ans. Deriaz turbine Which of the following shuts down the compressor motor when discharge
Oil is atomized either by air blast or pressure jet at about pressure rises to a predetermined point, it cuts in motor when pressure drops
Ans. 70 bar to predetermined pressure:
Type of solid injection that use single pump supplies fuel under high Ans. High pressure cut-off switch
pressure to a fuel header. Which of the following shuts down the compressor motor when discharge
Ans. Common rail injection pressure rises to a predetermined pressure and shuts down the compressor
Water flow in diesel engine that is caused by density differential. motor when there is decrease in pressure in the suction line.
Ans. Thermosiphon cooling Ans. Low pressure cut-off switch
Type of lubrication system in diesel engine in which oil from pump is carried The P.O. service pump must have:
to a separate storage tank outside the engine cylinder and used for high Ans. A means of shutting down from outside the machinery space
capacity engine. The relief valve on the P.O. transfer pump discharge
Ans. Dry sump lubricating system Ans. Back to the transfer pump discharge
Produces extreme pressure differentials and violent gas vibration When heating hot water, feed pump should at least be how many mm to
Ans. Detonation prevent vapor lock? Ans.1220 mm
In a spark ignition engine, the detonation occurs near the______________. The water column should be provided with a valve drain at least:
Ans. 15 mm pipe size A dry cooling tower where stream travels through large diameter trunks to a
Lubricating oils have flash points from cross-flow heat exchanger where it is condensed and cooled by air;
Ans. 375-800 degree Fahrenheit Ans. Direct condensing tower
A device receives information in one form of instrument signal and transmits An economizer is used to:
an output in another form; Ans. Heat the feedwater before it enters the boiler
Ans. converter A manometer measures:
Refinement petroleum Naphtha which by its composition is suitable for use in Ans. Air pressure
internal combustion engine; A pyrometer measures:
Ans. Gasoline Ans. Stack pressure
It is possible for water to get into the fuel oil by: An attemperator is another name for a
Ans. All of the above Ans. Desuperheater
Air chambers are attached to the: Waterwalls are used to:
Ans. Discharge side of the reciprocating pumps Ans. Carry of excess heat from the furnace walls
On automatic combustion control: The studs on waterwalls tubes are to:
Ans. Reduce the number of burners when the oil pressure approaches Ans. Give added surface area to tubes
minimum specified The heats remove from the cold area
The volumetric efficiency of the compressor increases as the suction Ans. Use for energy transfer
pressure: On the upstroke of the piston, the low pressure vapor is first compressed and
Ans. Increase then discharged as high pressure vapor through the discharged valves into
As the discharge pressure increases, the volumetric efficiency: the:
Ans. Increases Ans. Head of the compressor
When the suction And discharge pressure are varied in such a direction that The vapor that remains the clearance space at the end discharge stroke is
the compression ratio is increased, the volumetric efficiency of the called
compressor: Ans. Clearance vapor
Ans. Decrease Which of the following steps would you take if you found an accumulation of
For a compressor of any given clearance, the volumetric efficiency oil on the furnace floor?
Ans. Varies with compression ratio Ans. Open all registers
The useful energy transfer in Btu/hr divided by input power in Watts; Which of the following method can be used to clean up the inside of the
Ans. Energy efficiency ratio boiler?
If air is humidified by injecting steam of by pressuring the air through a hot Ans. All of the above
water spray, the dry bulb temperature and enthalpy of air Another term for the oil discharge strainer is the:
Ans. Increase Ans. Hot strainer
Air passing through a solid or absorbed bed, such as silica gel or activated In a controlled feed water system the power water level maintained by:
alumina will: Ans. The fluctuation water level in the boiler
Ans. Decrease in humidity The difference between the enthalpy of the product at a specific state and
A device that passes air through dense spray of recirculating water; the enthalpy of reactants at the same state for a complete reaction:
Ans. Air washer Ans. Enthalpy of reaction
The mass of water sprayed to the mass of air passing through the washer per The amount of heat released during a steady flow combustion process when I
unit time. kg of fuel burned completely at a specific temperature and pressure.
Ans. Spray ratio Ans. Enthalpy of combustion
When the cooling tower is used to provide cold water for the condense of a The enthalpy of a substance at a specific state due to its chemical
refrigerator system the water circulation system will be approximately: composition
Ans. 3 gal/min-ton Ans. Enthalpy of formation
Water lost in small droplets and carried away by the air flow: The inlet side of condenser tube are rolled and flared to:
Ans. Drift Ans. Allows a smoother entrance flow of circulating water
The ratio of total dissolved solids in the circulating water to the dissolved The purpose of inspection plates on the main condenser is to:
solids in the make-up water Ans. Check tubes without removing condenser heads
Ans. Ratio of concentration A pump that has one steam cylinder and two liquid cylinders is called a:
Through windage removes some solids, most must be removed by bleeding Ans. Tandem pump
some of the water. A mixture of dry air and superheated water vapor
Ans. Bleed-off Ans. Unsaturated air
When the air is unsaturated, what is the relation between the dry bulb Soot blowers should be used in proper sequence so that
temperature and wet bulb temperature? Ans. The soot will be upright toward the uptake
Ans. Less than What is the first thing you would check on taking over a watch?
The difference between the dry bulb temperature and wet bulb temperature Ans. The water level
Ans. Wet bulb depression A boiler with a fan/blower located in the uptake is operating on
The relative ratio of water vapor to dry air is: Ans. Induced draft
Ans. Humidity ratio A fire room that is isolated(closed) operates on:
Also known as humidity ratio: Ans. Forced draft
Ans. Specific humidity How many feed water lines are connected to the boiler?
The ratio of the actual humidity ratio to the saturated humidity ratio: Ans. Two
Ans. Degree of saturation If the water in the gauge glass has not been blown for a period of time, the
Also known as saturation ration: level of water in the glass will be:
Ans. Percentage humidity Ans. less than that in the steam and water drum
The partial pressure of the water vapor divided by the saturation pressure: The enthalpy of fluid when it is brought to rest adiabatically
Ans. Relative humidity Ans. Stagnation enthalpy
Rapidly increasing the temperature through the air of a chord will cause the In the absence of any heat and work and any changes in potential energy,
water in the gauge to the stagnation enthalpy of a fluid during a steady flow process:
Ans. Evaporate Ans. remains constant
Since the increase in the water vapor a latent heat contents equals the During the stagnation process, which of the following is converted to
decrease in the air’s sensible heat, the total enthalpies before and after enthalpy?
adiabatic saturation is: Ans. Kinetic energy
Ans. Equal The cross section of a nozzle at the smallest flow area is called:
An adiabatic saturation process follows a line of constant Ans. Throat
Ans. Dew point temperature The properties of a fluid at a location where the Mach number is unity are
Which of the following used with adiabatic saturation process? called:
Ans. Saturation efficiency Ans. Critical properties
The saturation efficiency of a large commercial air washer is typically What is the average fuel oil temperature range of the oil in the discharge
Ans. 80%-90% line?
The presence of oil or scale on the boiler tubes causes: Ans. 180-200 degree F
Ans. Overheating Which of the following is a common type of oil burner?
What would you do before giving a boiler bottom blow off? Ans. All of the above
Ans. Raise the water level The boiler gauge glass should be blown down.
When installing a new gauge glass in water gauge, you should secure the Ans. At the beginning of every watch
bolts from? Gauge pressure of 200 pounds is equivalent to what absolute pressure?
Ans. Center, alternately, toward each end Ans. 215
One of the main purpose of refractories in the boiler furnace is to: In a fuel oil service system, the quick closing fuel oil shut-off valve is located
Ans. Prevent excessive furnace heat losses between the:
A fuel oil service pump steam valves are fitted with: Ans. Master fuel oil shutoff valve and the boiler
Ans. Each rod leading to a location outside the fire room A fuel oil meter placed between the fuel oil service pumps and the fuel oil
Water tube boilers have heaters.
Ans. No fusible plugs Ans. Cold-type meter
The valve that prevents water from backing out of the boiler in the feed The presence of oil or scale on boiler tubes causes:
water line: Ans. Overheating
Ans. Feed check valve What would you do before giving a boiler bottom blow off:
The fuel oil heater is located Ans. raises the water level
Ans. On the discharge side of the service pump When installing a new gauge glass in a water gauge you should secure the
In the forced draft system: bolts from:
Ans. One fan supplies air to all furnaces Ans. Center, alternately towards each ends
In the closed fire room system One of the main purpose of refractories in the boiler furnace is to:
Ans. The fire room is supplied with air from one fan Ans. Prevent excessive furnace heat losses
The air cock on a boiler is located at the: A fuel oil service pump steam valves are fitted with:
Ans. Highest point of the steam and water drum Ans. Each rod leading to a location outside the fire room
Water tube boilers have Ans. Cold-type meter
Ans. No fusible plugs When securing a scotch boiler
The valve that prevents water from backing out of the boiler in the feed Ans. The belly plug must be removed
water line: If the cooling coils temperature is between the airs dew point, the moisture
Ans. Feed check valve will
The fuel oil heater is located Ans. Condense
Ans. On the discharge side of the service pump What occurs when the air passes through a water spray in an air washer?
In the forced draft system: Ans. Evaporative cooling process
Ans. One fan supplies air to all furnaces To prevent ice buildup, the cooled air temperature should be kept from
In the closed fire room system dropping
Ans. The fire room is supplied with air from one fan Ans. Below the freezing point of water
The air cock on a boiler is located at the: What precaution must be taken before using steam soot blowers?
Ans. Highest point of the steam and water drum Ans. Drain thoroughly
Soot blowers should be used in proper sequence so that Which of the following would you do before blowing the tubes with steam
Ans. The soot will be upright toward the uptake soot blower?
What is the first thing you would check on taking over a watch? Ans. All of the above
Ans. The water level Which of the following would cause a flareback?
A boiler with a fan blower located in the uptake is operating on Ans. Trying to relight from hot
Ans. Induced draft Sputtering oil burners might indicate
A fire room that is isolated (closed) operates on: Ans. Water in the fuel oil
Ans. Forced draft The boiler feed water in the feed water heater is heated by:
How many feed water lines are connected to the boiler? Ans. Auxiliary exhaust steam
Ans. Two
If the water in the gauge glass has not been blown down for a period, the If the boiler is smoking white smoke, a possible cause would be:
level of water in the glass will be: Ans. Too much air
Ans. Less than that in the steam and water The Edwards-type air pump has:
The enthalpy of fluid when it is brought to rest adiabatically. Ans. One set of valve
Ans. Stagnation enthalpy A scoop condenser is a:
In the absence of any heat and work and any changes in potential energy, Ans. Single pass condenser
the stagnation enthalpy of a fluid during a steady flow process; The purpose of the steam to baffle in a condenser
Ans. Remains constant Ans. Prevent the steam from hitting directly on the cooler tubes
During the stagnation process, which of the following is converted to The condenser vacuum feed valve is open and the water in the reserve tank
enthalpy? is dry. The result is:
Ans. Kinetic energy Ans. A loss of vacuum
The cross section of a nozzle at the smallest flow area is called: The division plate in a two-pass-condenser
Ans. Throat Ans. Prevents the circulating water from passing directly to overload
The properties of a fluid at a location where a Mach number is unity are discharge
called: The holes in Kinghorn-valve disks
Ans. Critical properties Ans. B
What is the average fuel oil temperature range of the oil in the discharge The equilibrium temperature that a regular thermometer measure if exposed
line? to atmospheric air:
Ans. 180-200 degree Fahrenheit Ans. Dry bulb temperature
Which of the following is a common type of burner? The temperature of air that has gone through an adiabatic saturation process
Ans. All of the above Ans. Wet bulb temperature
The boiler gauge glass should be blown down. If the vapor pressure equals the saturation process, the air is said to be:
Ans. At the beginning of every watch Ans. Saturated
Gauge pressure of 200 pounds is equivalent to what absolute pressure? When the air is saturated, the dry bulb, wet bulb and the dew point
Ans. 215 temperature are
In a fuel oil service system, the quick closing fuel oil shut-off valve is located Ans. Equal
between the: On taking over a watch, one should check
Ans. Master fuel oil shutoff Ans. All of the above
A fuel oil meter placed between the fuel oil service pump and fuel oil heater When you are cleaning fuel oil burner tips, use a
Ans. Brass knife Ans. Dry bulb temperature
The amount of steam generated by a boiler is dependent upon -The temperature of air that has gone through an adiabatic saturation process
Ans. All of the above Ans. Wet bulb temperature
A boiler operated at a pressure not exceeding 1.055 kg/cm2 gauge steam of -if the vapor pressure equals the saturation process the air is said to be
temperature not exceeding 120oC. Ans. Saturated
Ans. Low pressure heating boiler -When the air is saturated, the dry bulb, wet bulb and dew point temperature are
No part of the steam generator should be closer than how many from the Ans. Equal
wall? -On taking over a watch, one should check
Ans. 1 m Ans. All of the above
Steam generator should be mounted over a suitable foundation or concrete -When you are cleaning fuel oil burner tips, use a
not less than 305 mm thick and with sufficient area at base to be supported Ans. Brass knife
the bearing capacity of the soil with a factor of safety of -The amount of steam generated by a boiler is dependent upon
Ans. Not less than 4 Ans. All of the above
No smoke stacks should be closer than how many millimeters from the
exposed woodwork or framing -A boiler operated at pressure not exceeding 1.055 gauge steam of steam temperature not
Ans. 305 mm exceeding 120oC.
The air on an extinguisher fire should be Ans. Low pressure heating boiler
Ans. Kept closed -No part of the steam generator should be closer than how many from the wall?
Too low F.O. temperature will cause Ans. 1m
Ans. Poor combustion and smoky fires -Steam generator should be mounted over a suitable foundation or concrete of not less than
-when securing a scotch boiler 305 mm thick and with sufficient area at base to be supported the bearing capacity of the soil a
Ans. The belly plug must be remove factor of safety of
-if the cooling coils temperature is between the airs dew point, the moisture will Ans. Not less than 4
Ans. Condense -No smoke stacks should be closer than how many millimeters from exposed wood work or
-what occurs when the air passes through a water spray in an air washer? framing?
Ans. Evaporative cooling process Ans. 305 mm
-to prevent ice build up, the cooled air temperature should be kept from dropping -The air register on an extinguisher fire should be
Ans. Below freezing point of water Ans. Kept closed
-What precaution must be taken before using steam soot blowers? -Too low F.O. temperature will cause
Ans. All of the above Ans. Poor combustion and smoky fires
-Which of the following would cause a flareback? -When the fuel-oil temperature is too high it causes
Ans. Trying to relight from hot brickwork Ans. Carbon deposits on the fuel-oil heater coils
-Sputtering oil burners might indicate -A dry cooling tower where steam is condensed by cold water jets
Ans. Water in the fuel oil Ans. Indirect condensing dry cooling tower
-The boiler feed water in the feed water heater is heated by: -The hot condensate is pumped to cross heat exchangers whose it is cooled by.
Ans. Auxiliary exhaust steam Ans. Air
-If a boiler is smoking white smoke, a possible cause could be: -which of the following is the refrigerant “of choice” in entering air conditioning
Ans. Too much air Ans. R-22
-The Edwards-type air pump has; -In new equipment, which of the following replaces R-11
Ans. One set of valve Ans. R-12
-A scoop condenser is a: -The super heater is used to
Ans. Single pass condenser Ans. Remove moisture from steam
- The purpose of the steam baffle in a condenser is to: -Water tube boiler are
Ans. Prevent the steam from hitting directly on the cooler tubes Ans. Rolled in
-The condenser vacuum feed valve is open and the water in the reserve tank is dry. -The recirculating valve is used to
Ans. A loss of vacuum Ans. Recirculate and heat the fuel oil prior to lighting off
-The division plate in a two-pass-condenser -The fusible plug in a scotch boiler are found in the
Ans. Prevents the circulating water from passing directly to overload discharge Ans. Fire tubes
-The holes in kinghorn-valve discs -Which of the following valves are arrange in a Y-branch fitting?
Ans. B Ans. Boiler-drum safety valves
-The equilibrium temperature that a regular thermometer measure if exposed to atmospheric -The coils in the feedwater heater are secured by.
air; Ans. Union fitting
-Natural draft towers can cool the water to within. Ans. Equal to the mass of the suction vapor that the compressor takes in the entire suction inlet
Ans. 10 to 12oF unit time
Force draft towers can cool the water to within -In order to increase back pressure, one must
Ans. 5 to 8oF Ans. Close in on the back pressure valve
-When you light a fire in the burner, you must always -The actual steam that condenses in the feedwater heater
Ans. Use a torch Ans. Discharge to the hotwell
-The recirculating line on the burner valves returns the oil to the -The excess steam pressure in the back pressure system exhausts to the
Ans. Suction side of service pump Ans. Atmosphere through a relief valve
-The valve which shuts off liquid line with the least amount of resistance to flow. -The air pressure supplied to the boiler is measure in
Ans. Gate valve Ans. Inches of water
-The process applied to iron pipe which retards corrosion is called -The condensate from the coils in the fuel oil heater to the fuel oil heaters goes to the
Ans. Galvanizing Ans. Observation tank
-Listed in sequence, iron pipe series are; The valve between the fuel oil heaters and the burner valve is called the
Ans. ¼ ,3/8, ½, ¾ Ans. Root valve
-The two main purpose of the main condenser are; -The safety device located in the crown sheet of a scotch boiler is called a
Ans. Convert exhaust steam to water Ans. Fusible plug
-Pyrometer is a -Tube retarders in scotch boilers are used to
Ans. High temperature thermometer Ans. Slow down the combustion gases
-The temperature and humidity of the air to be used for comfort cooling shall be maintained at -In Order for oil to burn properly it must
effective temperature at air movement within the range from Ans. Al of the above
Ans. 0.0762 to 0.127m/s -The steam that is discharge from safety valves goes to the
-As the air enters the living zone the air motion in such occupied space and which the only Ans. Atmospheric line
source of contaminants shall have a velocity of not more than -The polytrophic exponent approaches one with the change in the process____approaches
Ans. 0.294m/s zero
-In air conditioning and ventilation standards, as the air enters into living zone, the distance Ans. Temperature
above the floor is -The change in kinetic energy of a process ___ as the change in temperature of the process
Ans. 1.603mm path increases
-Carbon dioxide concentration in air when measured 910 mm above the floor shall be Ans. Increases
estimated -The final temperature of an isobaric process if the system work in a _____Quantity and the
Ans. 1000ppm Surrounding heat transfer to the system
-The useful refrigerating effect equals to 211kj/mm is Ans. Positive
Ans. Tons of refrigeration -As the volume of an isothermal expansion process approaches its first value the rate of
-Another name for the fuel oil suction strainer; change in the system pressure.
Ans. Cold strainer Ans. Decreases
-In a manually controlled feed water heater system the proper water level is maintained by -As a fluid flow through a pipe , their potential energy
Ans. Operating the feed check valve Ans. Decreases
-The actual refrigerating capacity of the compressor -The heat of compression___ as suction vapor becomes super heated
Ans. Always less than the theoretical capacity Ans. Increases
-The ratio of the actual displacement of the compressor to its piston displacement -The heat content of the refrigeration cycle____ when a liquid- suction line heat exchanger is
Ans. Total or real volumetric efficiency installed
-Cyclone super heater are located in the Ans. Remains the same
Ans. Steam and water drum -The COP of refrigeration cycle___ with subcooling
-The purpose of the blow valve is Ans. Increases
Ans. Removes oil and scum from surface of water
-The purpose of the salinometer cock is to -Which defrost method is commonly used small commercial application where the forms on
Ans. Obtain a sample of boiler water for testing the evaporator surfaces?
-Salt in boiler water usually contains from Ans. Hot gas
Ans. Leaky condenser tubes -Which of the following liquid-chilling evaporator types incorporates overflow heat
-Before blowing down a boiler always exchangers?
Ans. Open skin valves Ans. Baudelot
-The mass flow rate produced by the compressor is -Which of the following is not a unit of energy?
Ans. Watt
-The unit of special heat are: Ans. None of the above
Ans. Btu/lbm.oF -Which of the following is not category of refrigeration application?
-Phase change process are constant________process. Ans. Transportation air conditioning
Ans. None of the above -Which of the following is not an industrial application
-Which of the following sequences accurately indicates the responses that occur when heat is Ans. Meat display cases
transferred from a gas? -Which of the following variable is not most often maintain by a commercial air conditioning
Ans. Sensible heat of vapors, latent heat of vaporization, sensible heat of liquids, latent heat of unit?
fusion, sensible heat of fluids Ans. Temperature
-Btu/lbm are units of -Providing clean, filtered air for trouble free operation of equipment and instrumentation is a
Ans. Specific enthalpy function of?
-As a liquid changes phase to vapor, its enthalpy Ans. Industrial air conditioning process
Ans. Increases -______ is one of the most common application of mechanical refrigeration.
- As a liquid changes phase to a vapor , its entropy Ans. Preservation of perishable commodities.
Ans. Decreases -The air conditioning systems for cabins on a luxury ocean liner belongs to the ____ category.
-The vaporization process that occurs at temperatures below the triple point of a substance Ans. Commercial air conditioning
is_____ Mechanical refrigeration system makes it possible to;
Ans. Sublimation Ans. All of the above
-The Vaporization process that occurs when the vapor pressure of a substance is equal to the -As the liquid changes to a vapor. The enthalpy
atmospheric pressure is: Ans. Increases
Ans. Boiling -Which of the following systems has the most complicated oil return system?
-As the pressure of a vapor increases, the amount of work increases and its enthalpy__ Ans. Direct staged
Ans. Increases -Which of the following process does not after the kinetic energy level of a substance?
-The entropy of R-134a in a saturated liquid phase at 40 psi is approximately Ans. Fusion
Ans.21 BTU/lb-oF -200 degrees F is equal to
-The latent heat of vaporization of R-134a in its saturated vapor phase at 0 degree C is Ans. None of the above
approximately. -An evaporator in a refrigeration unit makes use of which heat transfer mode?
Ans.196.7Kj/Kg Ans. All of the above
-The condenser of a commercial display cause that is located within the unit is called -When a service technician places his/her hand on a suction line to check the operation of a
Ans. B and C system, he/she is using which heat transfer modes?
-Which of the following characteristics of early refrigeration systems which applies to today’s Ans. Conduction
units. -Energy added to a vapor is known as Ans. B and C
Ans. Relative expensive -The rate of changes velocity is called:
-Mechanical refrigeration system make it possible in. Ans. Acceleration
Ans. All of the above -Which of the following devices should be used to measure a pressure of 90kpa?
-_______is method used to reduce the dehydration of sensitive fruits and vegetable during the Ans. All of the above
chilling process -The height of the mercury column in a barometer placed on a mountain with a local pressure
Ans. Water chilling of 12.5 psi will be?
-Which of the following is not a step in preparing vegetables for frozen storage? Ans. All of the above.
Ans. Coating with syrup -A compound gauge measuring a pressure of 22.44 inches of mercury is equivalent to an
-Loss of food juices by osmosis is a consequence of the ______ freezing process. absolute pressure of
Ans. Immersion Ans. 11.0 Psia
-Heat transfer that occurs primarily by conduction is used for ____ freezing process. -The electric meter on a home or building measure the amount of that was consumed over the
Ans. All of the above billing period.
-A disadvantage of sharp freezing is the Ans. Energy
Ans. All of the above -Which of the following is not a method of food preservation?
-Frozen storage chamber temperature set points are usually. Ans. Grilling
Ans. -20.5 degree C -Which of the following is not plant, animal or fungi?
-Which of the following is not a unit of density? Ans. Enzyme
Ans. Lb/ft3 -Which of the following refrigerant group denote a more toxic vapor?
-Wrapping and refrigerating food products extends the storage life of Ans. B3
Ans. Meat -What is the molecular weight of helium in 3600 Btu lb K?
-The approximate life of strawberries is. Ans. 4.0 lb mol
-In an isobaric process changes in pressure can be caused by changes in. Ans. Increase
Ans. None of the above -As the water temperature entering a water-cooled condenser decrease, the power drawn by
-Boyles law states that pressure and volume changes in gas process is ___related. the compressor.
Ans. Inversely Ans. Decrease
-As a gas is heated in an isothermal gases the volume -Which of the does not increase the volumetric efficiency of a compressor?
Ans. Increases Ans. Decreasing the discharge pressure
-As a gas is heated in an isobaric gases the volume -What is the compressor operating with a 30 psig suction pressure and a discharge pressure?
Ans. Decreases Ans. 3.13
-As a gas is heated in an isometric gases the volume -As the length of the tubes in a chiller barrel increases, the pressure drop across the inlet and
Ans. Remains constant outlet of the barrel:
-As the volume of a gas decreases, its specific gas constant Ans. Increases
Ans. Remains constant -_____ decreases the volumetric efficiency of a compressor
-Vapor do not behave as ideal gases because they experience Ans. Decreasing the suction pressure
Ans. Friction -The mechanical efficiency of a compressor_____ as the compressor load increases.
-The unit of latent heat of vaporization are: Ans. Remain the same
Ans. None of the above -The saturated suction temperature used in the selection of a compressor corresponds to the
-The unit of latent heat of fusion are: pressure of the vapor at the
Ans. Btu-lbm Ans. Inlet of the compressor
-A smaller temperature difference between the refrigerant in the liquid line and that in the -A smaller temperature difference between the refrigerant in the liquid line and that in the
evaporator___ the mass flow rate of the refrigerant needed per ton of refrigeration effect. evaporator ____ the mass flow rate of the refrigerant needed per ton of refrigerating effect.
Ans. Decreases Ans. Decreases
-Oil separator are used in system where: -The difference between the saturation pressure in the evaporator and that in the condenser___
Ans. All of the above as the suction temperature decreases.
-Which evaporator design incorporates secondary surface heat transfer? Ans. Increases
Ans. Finned-tube -The COP and the efficiency of a refrigeration cycle____ as the vaporizing temperature
-Eutectic filled plate evaporator have a higher_____ than evacuated plate evaporators: increases
Ans. All of the above Ans. Increases
-Which evaporator circuit configuration is less effective when used in expansion load -The volume of vapor that the compressor must handle per minute per ton____ as the
applications. vaporizing temperature increases.
Ans. Cross-flow Ans. Decreases
-Which of the following evaporators feed methods produces the greatest surface without the -The quantity of heat rejected at the condenser per unit capacity per minute____ as the
used of a liquid pump. vaporizing temperature decreases.
Ans. Flooded Ans. Decreases
-What type of force convection unit cooler is used in vegetable storage application that require -As the amount of scale on the refrigerant tubes of an evaporative condenser increases, the
a velocity of approximately 120m/min? scale formation of hot surfaces
Ans. Medium velocity Ans. Decreases
-Which of the following is not a hydrocarbon refrigerants -As the cycles of concentration in a tower or evaporative increases, the scale formation of hot
Ans. inane surfaces:
-Which of the following refrigerants is most hazardous? Ans. Increases
Ans. R-717 -The____ can only be effectively employed in industrial process that has constant refrigeration
-A mixture of two or more refrigerants is called a___ load.
Ans. Zoetrope Ans. Hand expansion valve
-Dessicants are used to remove___ from refrigerants -The device used to transfer forces across a sealed boundary is called:
Ans. Moisture Ans. Diaphragm
-As an oil-miscible refrigerant mixes with oil, the viscosity of the oil___ -The device used to measure the effects of a large pressure drop across the evaporator is called
Ans. Decreases a
-The society that sponsors research on refrigerants is called: Ans. External equalizer
Ans. ASHRAE -The mass flow rate of refrigerant per unit capacity____ as the condensing pressure
-The scale factor for hard water used in a condenser is: decreases.
Ans. 0.002 Ans. Decreases
-As the altitude of a forced air-condenser increases, the surface are of the coil or volume flow -It consists of a tapered glass tube set vertically in the fluid or gaseous piping system with its
rateof the fan must: large end at the top and a metering float free to move vertically in the tube
Ans. Rotameter
-Also termed as monitor light, it indicates which among a number of normal conditions of a Branch of which is larger than the run. Ans. Bull head tee
system or device exist.
Ans. Pilot light A larger pipe or drum into which each group of boiler is connected: Ans. Header
-Which of the following is not a caliper?
Ans. Feeler gage In pipe identification, the color fro pipe used for electricity: Ans. Light orange
-An underground formation that contains sufficient saturated permeable material to yield
significant quantities of water. Has the same equipment as the refrigerating circuits each with a condenser, evaporator, and a
Ans. Aquifer pressure imposing elements, where the evaporator of one circuit cools the condenser of the
-Chemical used to speed up sewage sedimentation: other circuit. Ans. Cascade
Ans. Lime
-Power expenditure when a current of the one ampere flow across two points having a voltage The temperature of air to be used for comfort cooling shall be in the range of: Ans. 20 deg to
drop of one volt. 23.3 deg
Ans. Watt
-Prandtl number for air is generally in the order of A valve held closed by a spring or other means and designed automatically relieve pressure in
Ans. 0.70 excess of its settling. Ans. Pressure relieve device.
Relative humidity of air to be used in standard air cooling: Ans. 55%-60%
A refrigerating system in which the pressure-imposing element is mechanically operated: Ans. A refrigerants which should not be used in public assembly occupancies. Ans. Group 3
Compression refrigerants
A cushioning device all the end of a trolley, bridge or other moving parts of a crane operating
on rails to minimize shock Fire involving flammable liquids and gases. Ans. Class B fires
in the event of collision: Ans. Buffer
Mechanics of water or other liquids whether at rest or motion. Ans. Hydraulics
A boom type mobile crane mounted on endless track or treated belts: Ans. Crawler crane
Measure the pressure of water discharging from a nozzle by having its open end on the water
An apparatus for raising or lowering a load by the application of a pulling force but does not and the end connected to a manometer. Ans. Pitot Tube
include a car or platform riding in guides: Ans. Hoist
A vertical turbine pump with the pump and the motor closed coupled and designed to be
Most widely used industrial pressure gage applied to both pressure and vacuum: Ans. Bourdon installed underground. Ans. Submersible pump
tube gage
Which of the following is a type of deep well pump. Ans. All of the above.
Hydrocarbons found in liquefied petroleum gas: Ans. All of the above
Steel pipe coated with zinc to resist corrosion. Ans. Galvanized pipe
Which of the following is not a solid fuel: Ans. Tar
A fitting with a number of branches in the connecting the smaller pipes. Ans. Manifold
Amount of cooling produced by 2000 lbs of ice in melting over a period of 24 hrs.: Ans. B
and C The ratio of peak load to the Average load is termed as _____ in variable load nomenclature.
Ans. Load factor
It smoothens the flow due to the nature of flow of the liquid from a reciprocating pump: Ans.
Air Chamber A heat exchange device used to provide heat transfer between exhaust gases and the air prior
to its entrance to the combustor. Ans. Regenerator
Science of force exerted by water in motion: Ans. Hydrodynamics
In a sensible heating process, the moisture content: Ans. Remains constant
Lowest permissible water level of a boiler without internal furnace: Ans. 1/3 height of shell
Flow on both sides on a normal shockwave: Ans. Supersonic on one side, Subsonic on the
In pumps, it transmits power from the driver to the impeller: Ans. Shaft other

Cavitation occurs when the pressure at any point inside a pump drops below the vapor A change of phase directly from vapor to solid, without passing through the liquid state: Ans.
pressure corresponding to the temperature of liquid. Its effect include: Ans. All of the above Deposition

Positive displacement pump consisting of a fixed casing containing gears, cams, screws, What reaction occurs when the enthalpy of the product is less than the enthalpy of the
vanes, plungers or similar elements actuated by rotation of the drive shaft. Ans. Rotary pumps reactants? Ans. Exothermic
Refrigerant used in passenger aircraft bins. Ans. Air
A well designed engine has a volumetric efficiency within the ranged: Ans. 75%-90%
It refers to atoms of the same atomic number but differ in atomic masses and molecular
It is referred to as the maximum continuous power available from a hydroelectric power plant weights. Ans. Isotopes
even under the most adverse hydraulic condition: Ans. Firm power
Piston rings are normally made of: Ans. Cast Iron
There are how many feed water lines connected to the boiler? Ans. Two
Two isothermal and two reversible adiabatic process comprise a: Ans. Carnot cycle
It is the difference in pressure as measured above or below the atmospheric pressure. Ans.
Draft In terms of viscosity, density can be expressed as: Ans. Dynamic viscosity/ kinematic viscosity

The compression ratio of a gas turbine is in the range of 5-8. Ans. 5 to 8 Compressor often used in supercharging Diesel engine: Ans. Root blower type

The average pH of a normal rainfall is generally: Ans. Slightly less than 7 In treating a person whose eyes accidentally got in contact with Freon use: Ans. Sterile
mineral oil
Another term used for liquid valve. Ans. King valve
The coefficient of performance of a domestic (local) refrigerator is always: Ans. Greater than A device in vapor compression refrigeration system whose primary function is to meter the
one flow of
Refrigerant to the evaporator: Ans. Thermostatic expansion valve
A belt-Coleman cycle is a reversed : Ans. Joule cycle
From the mathematical perspective, a thermodynamic property is: Ans. A point function
In an air compressor system, the function of a receiver is to: Ans. Collects water and grease
suspended in air In the production of beer, a raw materials called yeast is added in the: Ans. Starting tubs

General layout plan for each floor drawn to scale should not be less than scale of: Ans. 1:200 Purpose is to keep the moisture from entering the system. Ans. Dehydrator

Highest vapor drop in a vapor compression refrigeration cycle occurs in the: Ans. Expansion Medium pressure as applied to valves and fittings implies that a working pressure of _____ is
valve suitable enough. Ans. 862-1200psi

Which of the following pairs represent the two broad classification of lubricating oils? Ans. A liquid-vapor mixture with a dryness factor is allowed to absorb heat. Which of the following
Straight and Additives is likely to occur? Ans. Increases

A form of misalignment between the pump and the driver shaft wherein the shafts are Work rooms referring to the maintenance shop and machine rooms shall be _____ in the
concentric but not parallel. Ans. Angular Misalignment height from floor to the ceiling? Ans. 3,000mm

In the field of metal corrosion, it is the process wherein it exhibits in the quality deterioration Defined as a passageway made of sheet metal or other suitable material not necessarily leak
of metals: Ans. Passivation tight, used for conveying air or other gases at low temperature. Ans. Duct

It is the ratio of the density of liquid substance to the density of water at standard conditions: As a good practical rule, the foundation depth may be taken as _____ times the engine stroke,
Ans. Specific gravity the lower factor for well-balanced multi-cylinder engine and the higher factor for engines with
fewer cylinder s or on less firm soil: Ans. 3.2 to 4.2
Term associated with an increase on pressure on a pipe caused by sudden velocity decrease.
Ans. Water hammer What is the suggested maximum permissible dos (MDP) of a gamma ray exposed for
individuals now working in a nuclear environment in rem year? Ans. ½
A process which takes place without change in volume. Ans. Isochoric
Most commercially petroleum lubricating oil deteriorates starting from temperature of: Ans.
Weir refers to an opening: Ans. Having partial full flow 200 F

The Westphal balance is a laboratory instrument used to: Ans. Specific gravity States that the external pressure applied to a confined liquid increases the pressure of every
point in the fluid by an amount equal to the external pressure: Ans. Pascal’s Law
Generally, permissible velocity of water flowing through concrete tunnel is: Ans. 4-5 m/s
The average fuel-oil temperature range of the oil in the discharge line to the boilers. Ans. 180- The chemical formula of R-12 or dichlorofluoromethane. Ans. CCL₂F₂
200 F
Which of the following is not desirable property of a refrigerant? Ans. Low thermal
Boiler gage glasses should be blown down: Ans. At the start of every watch conductivity

The lowest portion of the storage basin where the water is not drawn: Ans. Dead storage Refrigerant widely used in room air conditions. Ans. R-22

Which among the following do not measure relative humidity? Ans. Piezometer What occurs when pumps are connected in parallel? Ans. Increase discharge, same head

Water behind the dam of a hydro electric power plant? Ans. Pondage Which of the following is not a cause of cavitation? Ans. Low velocity

Converts energy of water to mechanical energy: Ans. Turbine Pipe attached to the penstock to be able to let the water be at atmospheric pressure. Ans. Surge
chamber
Ratio of average load to the rating of the equipment supplying the load. Ans. Plant factor
Which of the following is not considered as gaseous fuel? Ans. Bunker
Ratio of actual maximum demand load to the connected load. Ans. Demand factor
Ratio of the density of a liquid to the density of water. Ans. Specific Gravity
A device which preheats feed water by utilizing the heat of the flue gases. Ans. Economizer Measurement of randomness or disorder. Ans. Entropy
Study of the proportion of water vapor content of air. Ans. Air conditioning
Energy cannot be created nor destroyed but can only be transformed from one form to another.
Actual temperature of air. Ans. Dry bulb temperature Ans. First law of thermodynamics

For a machine foundation with class A mixture what is the proportion of cement, sand and In standard Otto cycle when the compression ratio is increased then the thermal efficiency
gravel? Ans. 1:2:4 will: Ans. Increase

Component included in the proximate analysis of fuel. Ans. Ash, moisture, Volatile matter, Heat energy produced by the movement of molecule within a substance caused by its
fixed carbon temperature. Ans. Internal energy

The prime mover must have its main steam line: Ans. In loops The law which states that one cannot operate a 100% efficient machine. Ans. 2nd law of
thermodynamics
Intercooler are primarily used with: Ans. Gas compressors
In relation to brake power the instrument used to measure torque: Ans. Dynamometer
In the psychrometric chart, the diagonal lines represents: Ans. Wet bulb temperature
Ratio of the radiation of an actual body to the radiation of the black body. Ans. Emittance
Moderator in certain types of nuclear reactors. Ans. Heavy water
Refrigerant used in ice plant. Ans. Ammonia
The main advantage of turbulous boilers. Ans. Steam pressure can be raised in short time
Measures the pressure of water discharging from the nozzle by having its open end in the
The law which states that the entropy of all perfect crystalline solids is zero at absolute zero water and the other end connected to a manometer. Ans. Pitot tube
temperature. Ans. 3rd law of thermodynamics
Defined as a wall designed to prevent the spread of fire having a fir resistance for four hours.
Operates between two constant temperature reservoir. Ans. Carnot engine Ans. Fire wall

Specific measurements of moisture content of air. Ans. Degree of saturation The work done per unit charge when the charge is moved from one point to another? Ans.
Potential at a point
Simultaneous on site generation of electric energy and steam from the same plant output. Ans.
Co-generation In the relation of PVⁿ is constant, what value makes the process isobaric? Ans. Zero

Instrument used for pressure readings. Ans. Manometer In actual gas behavior, molecular collisions are: Ans. Inelastic

Instrument used to analyze gases. Ans. Orsat Apparatus An ideal refrigeration should have: Ans. Low freezing point
The immediate undesirable products from the petroleum based lubricating oil subjected to Difference between the indicated power and the brake power. Ans. Friction power
high pressure and temperature is referred as: Ans. Carbon Dioxide
In Diesel power plant, its purpose is to reduce the weight-to-power ratio: Ans. Supercharging
A rotary dynamic pump, fluid is at no time confined by moving boundaries in its passage
through the pump. Ans. Non-positive displacement pump Change of phase from solid to gas. Ans. Sublimation

Also known as tangential or Pelton wheel. Ans. Impulse turbine The memory lost when operating power is removed. Ans. Volatile memory

Term given to a chimney made of steel. Ans. Stack A refrigeration system using direct method in which refrigerant is delivered to two or more
evaporators in space room or in refrigerators. Ans. Multi-pressure
An integral part of reaction turbine used to recover energy head. Ans. Draft tube
The carbon dioxide concentration of air in all rooms when measured 910mm above the floor
Based on good practice, the vertical distance from the floor soil level to the top edge of the space not exceed: Ans. 100ppm
foundation must be around _____ as minimum distance in machiner foundation. Ans. 6 inches
Consists of helmets shall be removed immediately after having been used or the seal broken,
The geometric view factor for a black body is always. Ans. One must be removed atleast every: Ans. 2 years
For every 1000ft ascent, there is a corresponding pressure decrease approximately. Ans. 1 in
of Hg. Pump whose purpose is to increase the effective water pressure by sucking water from public
service main or private use water system: Ans. Booster pump
Used for impounding water storage and for creating head for the power plant. Ans. Dam
A small fitting with a double offset, or shaped like the letter C with the ends turned out. Ans.
The temperature at which the phase change takes place at a given pressure. Ans. Saturation Cross-cover
pressure
Material interposed between two relatively moving machine elements to reduce wear and tear:
Distinguishes the system from its surroundings may be at rest or in motion. Ans. Boundary Ans. Lubricant

Quality is a measure of: Ans. Dryness Measure temperature by electromotive force: Ans. Thermocouple

The highest pressure under which distinguished liquid vapor phase can exist in equilibrium: According to SAE which of the following is a type of lubricating oil? Ans. All of the above
Ans. Critical pressure
A device that information in one form of an instrument signal and transmits an output signal
In a thermodynamic gaseous substance, the relationship between P-V-T given by the: Ans. on another form: Ans. Digital
Equation of state
A device whose function to pass on information in an unchanged form or in some modified
A correction factor for gas approaching real gas behavior: Ans. Compressibility factor term. Ans. Switch

Furl production process wherein heavy oil is changed into gasoline by means of a high A device that receives information on one form of one or more physical quantities modifies
pressure, high temperature and longer exposure time. Ans. Thermal cracking the information and/or its form and produces a resultant output signal. Ans. Transducer

A shut off valve for controlling the flow of refrigerant. Ans. Stop valve Which of the following is not a requirement in designing pipe installations? Ans. Galvanized
pipe should be used for steam
Measured by the amount of its pressure below the prevailing atmospheric pressure. Ans.
Partial vaccum Part of a valve used to guide and support valve stem: Ans. Bonnet

A relief valve that can positively lift the disc from its seat at least 1.5 mm when there is no In piping installations, the color of pipe tube used for water is: Ans. Green
pressure in the boiler. Ans. Water relief valve
The color code for pipes used in communication. Ans. White
Vapor produced in the vaporization of a small amount of liquid refrigerant downstream of the
expansion valve. Ans. Flash gas Which of the method is used for classifying coal. Ans. All of the above

Product of the rate of evaporation and the factor of evaporation. Ans. Equivalent evaporation Alcohol frequently considered as fuel for internal combustion engine. Ans. Ethyl alcohol
Measure of resistance to flow. Ans. Viscosity PV = mRT
280(2) = m(8.314/44)(40+273)
Converts Solar energy to electrical energy. Ans. Photovoltaic cell m = ____________ ANS

Which of the following is not an instrument used to measure flow rates? Ans. Velometer # A thermal power plant has a heat rate of a 11,363 Btu/kw-hr. Find the thermal efficiency of
the plant.
Refrigeration follows what cycle? Ans. Reversed Carnot cycle Solutionj:
eth = 3412/Heat rate = 3412/11,363 = _________ANS.
An evaporator constructed on pipe tubing. Ans. Expansion coil
# What is the hydraulic gradient of a 1 mile, 17 inches inside diameter pipe when 3300
Reinstalled or second hand boilers shall have a minimum factor of safety of? Ans. 6 gal/min of water flow with f = 0.03.
A line that shows the rotation of the consumption and the load a steam turbine generator. Ans. Solution:
Willans Line V = (3300/7.481)/(π/4)(17/12)2(60) = 4.66 ft/s
L = 1 mile = 5280 ft
Instrument that indicate the percentage of carbon dioxide in flue gases in a power plant. Ans. hL = fLv2/2gD = 0.03(5280)(4.66)2/2(32.2)(17/12) = 37.7 ft
Ranarex indicator Hydraulic gradient = 37.7/5280 = ___________ANS.

For real process, the net entropy change in the universe is: Ans. Positive # Find the loss of head in the pipe entrance if speed of flow is 10 m/s.
Solution:
The sum of the internal energy and the product of the pressure and specific volume. Ans. Loss at entrance = 0.5(v2/2g) = 0.5[102 / 2(9.81)] = ___________ANS.
Enthalpy
Wet material, containing 220% moisture (dry basis) is to be dried at the rate of 1.5 kg/s in a
Ratio of the maximum demand of the system to the rated capacity of the system. Ans. continuous dryer to give a product containing 10% (dry basis). Find the moisture removed,
Utilization factor kg/hr.
Solution:
Source of thermal energy. Ans. All of the above Solid in wet feed = solid in dried product
[1/(1 + 2.2)](1.5) = [1/(1 + 0.1)](x)
Cycle that has two isentropic and two isometric process. Ans. Otto cycle x = 0,5156 kg/s (total dried product)
Moisture removed = 1.5 – 0.5156 = 0.984 kg/s = ___________ANS.
Cycle that has two isentropic and two isobaric process. Ans. Brayton cycle
# Copra enters a dryer contacting 70% moisture and leaves at 7% moisture. Find the moisture
# Two kilogram of gas is confined in a 1 m3 tank at 200 kpa and 88°C. What type of gas is in removed on each pound of solid in final product.
the tank? Solution:
Solution: Solid in wet feed = solid in dried product
PV=mRT 0.3x = 1
200(1) = 2(8.314/M) (88+273) x = 3.333 lbs
M = 30 1= 0.93y
Therefore: gas is Ethane Y = 1.07527 lb
Moisture removed = x – y = 3.333 – 1.07527 = _________ ANS.
# Find the enthalpy of helium if its internal energy is 200 KJ/kg.
Solution: # A 1 m x 1.5 m cylindrical tank is full of oil with SG = 0.92. Find the force acting at the
R = 8.314/4 = 2.0785 bottom of the tank in dynes.
K = 1.667 for helium Solution:
Cp = k R/(k-1) = 2.0785/(1.667 – 1) = 3.116 KJ/kg-K P = w h = (0.92 x 9.81) (1.5) = 13.5378 kpa
F = PA = 13.5378(π/4 x 12) = 10,632 KN = 10,632.56 N x 10,000 dynes/N
h/ = Cp/Cv
F = ____________ANS.
/200 = 5.195/3.116
# Find the pressure at the 100 fathom depth of water in kpag
= ______________ANS Solution:
H = 100 fathom x 6 = 600 ft
# Compute the mass of a 2 m3 propane at 280 kpa and 40°C. P = w h = (600/3,281)(9.81) = ______________ANS.
Solution:
Propane is C3 H8 --------- M = 12(3) + 8(1) = 44
# Find the depth in furlong of the ocean (SG = 1.03) if the pressure at the sea bed is 2,032.56 BP = 40 (0.90) = 36 kw
kpag. Wc = 36(1-0.20) = 28.80 kw
Solution: COP = RE/Wc
P=wh 6 = RE/28.80
2,032.56 = (1.03 x 9.81) h RE = 172.8/3.516 = ______________ANS.
H = 201.158 m x 3.281ft/m x 1 yd/3ft x 1 furlong/220 yd = 1 furlong
# A 23 tons refrigeration system has a heat rejected of 100 kw. Find the energy efficiency ratio
# Find the mass of 10 quartz of water. of the system.
Solution: Solution:
V = 10 quartz x 1gal/4quartz x 3.785li/gal x 1m3/1000li QR = RE + Wc
V= 0.0094625 x 10-3 m3 100 = 23(3.516) + Wc
W = m/V Wc = 19.132 kw
1000 = m/0/0094625 x 10-3 COP = RE/Wc = (23 x 3.516) / 19.132 = 4.23
m = __________ ANS. EER = 3.412 COP = 3.412(4.23) = ____________ANS.

# Find the mass of carbon dioxide having a pressure of 20 psia at 200°F with 10 ft3 volume. # A 200 mm x 250 mm, 8-cylinder, 4-stroke diesel engine has a break power of 150 kw. The
Solution: mechanical efficiency is 80%. If two of the cylinders were accidentally cut off, what will be
PV = m R T the new friction power?
(20 x 144)(10) = m (1545/44)(200 + 460) Solution:
m = ___________ANS. em = BP/IP
0.8 = 150/IP
# Find the heat needed to raise the temperature of water from 30°C to 100°C with 60% quality. IP = 187.5 kw
Consider and atmospheric pressure of 101.325 kpa. Use the approximate enthalpy formula of FP1 = IP – BP = 187.5 – 150 = 37.50 kw
liquid. FP1 = FP2 = ____________ANS.
Solution:
At 100°C # If the energy efficiency ratio of the refrigeration system is 12.6, what is the COP of the
hf = Cp t = 4.187(100) 418.7 KJ/kg system?
hfg = 2257 KJ.kg Solution:
h2 = hf + xhfg = 418.7 + 0.60(2257) = 1,772.9 KJ/kg EER = 3.412 COP
Q = 1(4.187)(100-30) + 1(1772.9 – 418.7) = _________ANS. 12.6 = 3.412 COP
COP = ____________ANS.
# Find the enthalpy of water at 212°F and 14.7 psi if the dryness factor is 30%. Use the
approximate enthalpy formula of liquid. # An air compressor has a power of 40 kw at 4% clearance. If clearance will increase to 7%,
Solution: what is the new power?
hf = (°F – 32) = (212 – 32) = 180 Btu/lb Solution:
hfg = 970 Btu/lb The power of compressor will not be affected with the changes in clearance.
h = hf + xhfg Therefore the power will still be 40 kw.
h = 180 + 0.3(970) = ________ANS.
# What is the approximate value of temperature of water having enthalpy of 208 Btu/lb?
# An air compressor consumed 1200 kw-hr per day of energy. The electric motor driving the Solution:
compressor has an efficiency of 80%. If indicated power of the compressor is 34 kw, find the h = °F – 32
mechanical efficiency of the compressor. 208 = °F -32
Solution: °F = _________ANS.
Pim = 1200kw-hr/24 hrs = 50 kw
BP = 50(0.80) = 40 kw # An Otto cycle has a compression ratio of 8. Find the pressure ration during compression.
em = 34/40 = _______ANS. Solution:
P1V1k = P2V2k
# A refrigeration system consumed 28,800 kw-hr per month of energy. There are 20% of (V1/V2)k = (P2/P1)
energy is lost due to cooling system of compressor and motor efficiency is 90%. If COP of the rK = rp
sustem is 6, find the tons of refrigeration of the system. rp = (8)1.4 = ___________ANS.
Solution:
Pim = 28,800/(24 x 30) = 40 kw
# A diesel cycle has a cut off ratio of 2.5 and expansion ratio of 4. Find the clearance of the # A perfect gas has a value of R = 58.8 ft-lb/lb-R and k = 1.23. if 20 Btu are added to 10 lbs of
cycle. this gas at constant volume when initial temperature is 90°F, find the final temperature.
Solution
# Determine the atmospheric pressure at a location where barometric reading is 740 mm Hg Q = m cv (t2 – t1)
and gravitational acceleration is g = 9.7 m/s2. Assume the temperature of mercury to be 10°C,
at which the density is 13,570 kg/m3. cv = R / (k -1) = = 0.29086 Btu/lb-F
Solution: 20 = 10(0.29086)(t2 – 90)
P = (wg)h = (13,570 x 9.7)(9.74) (1Kpa/1000 N/m2) = ___________ANS. t2 = ___________ANS.
# The barometer of a mountain hiker reads 930 mbars at the beginning of a hiking trip and 780 # Ammonia weighing 22kgs is confirmed inside a cylinder equipped with a piston has
mbars at the end. Neglecting the effect of altitude on local gravitational acceleration,
determine the vertical distance climb. Assume g = 9.7 m/s2. an initial pressure of 413KPa at 38 C. If 3200KJ of heat is added to the ammonia until its final
Solution: pressure and temperature are 413KPa and 100oC respectively, what is the amount of work
P1 – P2 = w h done by the fluid in KJ?
(0.93 – 0.78)(100 kpa/bar) = (1.2 x 0.00981) h SOLUTION
h = 1274.21 m Since the molecular weight of ammonia is 17. Then
h = 1274.21 (9.81/9.7) = ___________ANS. R = 8.3143/M = 8.3143/17 = 0.489 KJ/kgoK
P1V1 = mRT1
# The lower half of a 10 m high cylindrical container is filled with water and the upper half 413V1 = 22(0.489)(38 + 273)
with oil that has SG = 8.85. Determine the pressure difference between the top and bottom of V1 = 8.101 m3
the cylinder. P2V2 = mRT2
Solution: 413V2 = 22(0.489)(100 + 273)
V2 = 9.716 m3
P = Pw + Po = 9.81(5) + (0.85 x 9.81)(15) = ________ANS. W = P(V1-V2) = 413(9.716 – 8.101) = ANS.
# A tank contains 90ft3 of air at a pressure of 350 psig; if the air is cooled until its
# An ideal gas at 0.80 atmospheres and 87°C occupies 0.450 liter. How many moles are in the pressure and temperature decreases to 200 psig and 70oF respectively. What is the decrease in
sample? (R = 0.0821 liter-atm/mole – K) internal energy?
Solution: SOLUTION
PV = nRT m = PV/RT = (200 + 14.7)(90)(144)/(53.342)(70 + 460) = 98.50 lbs
for constant volume process:
(0.80 atm)(0.450 li) = n (0.0821 )(87+273)K P1/T1 = P2/T2
n = ___________ANS. T2 = 70 + 460 = 530oR

# A certain gas at 101.325 Kpa and 10°C whose volume is 2.83 m3 are compressed into a =
storage vessel of 0.31 m3 capacity. Before admission, the storage vessel contained the gas at a T1 = 900oR
pressure and temperature of 173.8 Kpa and 26°C, after admission the pressure has increased to ΔU = mcv(T2 – T1) = 98.50(0.171)(530 – 900) = ANS.
1171.8 Kpa. What should be the final temperature of the gas in the vessel in Kelvin? # A large mining company was provided with a 3m3 of compressed air tank. Air
Solution: pressure in the tank drops from 700KPa while the temperature remains constant at 28oC. What
Solving for the mass of gas which is to be compressed: percentage has the mass of air in the tank been reduced?
PV = mRT SOLUTION
101.325(2.83) = m1R(10 + 273) Percent mass reduced = (700 – 150)/700 = ANS.
m1 = 1.01325/R # A 4m3/hr pump delivers water to a pressure tank. At the start, the gauge reads 138KPa
Solving for the mass of gas initially contained in the vessel: until it reads 276KPa and then the pump was shut off. The volume of the tank is 180liters. At
PV = mRT 276KPa the water occupied 2/3 of the tank volume. Determine the volume of the water that
137.8(0.31) = m2R(26 + 273) can be taken out until the gauge reads 138KPa.
m2 = 0.14286/R SOLUTION
Solving for the final temperature: Consider the air pressure:
m3 = m1 + m2 V2 = 1/3 (180) = 60liters
m3 = 1.01325/R + 0.14286/R = 1.156/R P1V1 = P2V2
1171.8(0.31) = (1.156/R)RT3 (138 + 101.325)(V1) = (276 + 101.325)(60)
T3 = ____________ANS. V1 = 94.59 liters
Amount of water to be removed = 2/3 (180) – (180 – 94.59) = ANS.
# A refrigeration plant is rated at 15tons capacity. How many pounds of air per hour will h = hf + xhfg = 814.93 + 0.17614(1972.7) = ANS.
it cool from 70 to 90oF at constant pressure. # Mixture with 70% quality at 500KPa is heated isothermally until its pressure is
SOLUTION 300KPa. Find the heat added during the process.
Tons of refrigeration = mcp(t2 – t1)/12000 At 500KPa: sf = 1.8607, sfg = 4.9606
At 300KPa and 151.86oC s = 7.088 KJ/kg
SOLUTION
s1 = sf + xsfg = 1.8607 + 0.70(4.9606) = 5.333 KJ/kg
m= ANS. s2 = 7.088 KJ/kg
# An air standard engine has a compression ratio of 18 and a cut – off ratio of 4. If the Q = T(s2 - s1) = (151.86 + 273)(7.088 – 5.333) = ANS.
intake pressure and temperature are 100KPa and 27oC, find the work in KJ per kg. # A tank contains exactly one kilogram of water consisting of liquid and vapor
SOLUTION equilibrium at 1MPa. If the liquid contains one – third and the remaining is vapor of the
volume of the tank, what is the enthalpy of the contents of the tank?
At 1MPa: vf = 0.0011273 vfg = 0.19444
hf = 762.81 hfg = 2015.3
T1 = 27 + 273 = 300oK SOLUTION
Process 1 to 2 is isentropic procees: Let V = total volume of the tank
T2 = T1(V1/V2)k-1 = 300(18)1.4-1 = 953.301oK T2 = T1(V1/V2)k-1 = 300(18)1.4 – 1 = 953.301oK
Process 2 to 3 is constant pressure process: Process 2 to 3 is constant process:
T3/T2 = V2/V3 = rc
T3 = 953.301 (4) = 3813.205oK
QA = mcp(T3 – T2) = 1(3813.205 -953.301) = 2860 KJ/kg
W = (.5531)(2860) = ANS. T3 = 953.301(4) = 3813.205oK
# Determine the air – standard efficiency of an engine operating on the diesel cycle with QA = mcp(T3 – T2) = 1(3813.205 - 953.301) = 2850 KJ/kg
clearance of 6% when the suction pressure is 99.97KPa and the fuel injected for 7% of the W = (0.5531)( 2850) = ANS.
stroke. Assume k = 1.4. # Determine the air standard efficiency of an engine operating on the diesel cycle with
SOLUTION clearance of 6% when the suction pressure is 99.97KPa and the fuel is injected for 7% of the
V3 – V2 = 0.07VD stroke. Assume k = 1.4.
V2 = 0.06VD SOLUTION
V3 - 0.06VD = 0.07VD V3 - V2 = 0.07VD
V3 = 0.13VD V2 = 0.06VD
rc = V3/ V2 = 0.13VD/0.06VD = 2.167 V3 = 0.07VD + 0.06VD
rk = (1 + 0.06)/0.06 = 17.667 V3 = 0.13VD
rc = V3/ V2 = 0.13VD /0.06VD = 2.167
rk = (1 + 0.06)/(0.060) = 17.667
= ANS.
# Steam at 2MPa and 250oC in a rigid cylinder is cooled until the quality is 30%. Find
the heat rejected from the cylinder. e= = ANS.
At 2MPa and 250oC: v = 0.11144m3/kg u = 2679.6 KJ/kg # A steam at 2MPa and 250oC in a right rigid cylinder is cooled until the quality is 30%.
At 2MPa, (saturated): vf = 0.0011767m3/kg, vg = 0.09963m3/kg uf = 906.44 Find the heat rejected from the cylinder.
KJ/kg, At 2MPa and 250oC: v = 0.11144 m3/kg u = 2679.6 KJ/kg
ufg = 1693.8KJ/kg At 2MPa, (saturated): vf = 0.0011767 m3/kg, vg = 0.09963 m3/kg
SOLUTION uf = 906.44 KJ/kg ufg = 1693.8 KJ/kg
Q = U2 – U1 SOLUTION
U1 = 2679.6 KJ/kg Q = (u2 – u1)
U2 = Uf + xUfg = 906.44 + 0.30(1693.8) = 1414.58 KJ/kg u1 = 2679.6KJ/kg
Q = 1414.58 – 2679.6 = ANS. u2 = uf + x ufg = 906.44 + 0.3(1693.8) = 1414.58KJ/kg
# At 1.3MPa, mixture steam and water has an entropy of 3KJ/kg - oK. Find the enthalpy Q = (1414.58 – 2679.6) = ANS
of the mixture. # At 1.3MPa, mixture steam and water has an entropy of 3KJ/kg-oK. Find the enthalpy
At 1.3MPa: sf = 2.2515, sg = 6.4953, hf = 814.93, hfg = 1972.7 of the mixture.
SOLUTION At 1.3MPa: sf = 2.2515, sg = 6.4953, hf = 814.93, hfg = 1972.7
s = sf + xsfg SOLUTION
3 = 2.2515 + x(6.4953) s = sf + x(sg – sf)
x = 0.17614 3 = 2.2515 + x(6.4953 – 2.2515)
x = 0.17637 # A throttling calorimeter is connected to the desuperheated steam line supplying steam
h = hf + xhfg = 814.93 + 0.17637(1972.7) = ANS. to the auxiliary feed pump on a ship. The line pressure measures 2.5MPa. The calorimeter
# Mixture with 70% quality at 500KPa is heated isothermally until its pressure is pressure is 110KPa and 150oC. Determine the entropy of the steam line.
300KPa. Find the heat added during the process. At 110KPa and 150.C: h2 = 2775.6KJ.kg
At 500KPa: sf = 1.8607, sfg = 4.9606 At 2.5MPa: hf = 962.11KJ/kg, hfg = 1841 KJ/kg, sf = 2.5547, sfg = 3.7028
At 300KPa and 151.86oC: s = 7.0888 SOLUTION
SOLUTION For throttling process: (h1 = h2)
For isothermal process, t1 = t2 h1 = h2 = hf + x hfg
s1 = sf + xsfg = 1.8607 + 0.7(4.9606) = 5.333 2775.6 = 962.11 + x(1841)
s2 = 7.0888 x = 98.5%
Q = T(s2 – s1) = (151.86 + 273)(7.0888 – 5.333) = ANS. s1 = sf + x sfg = 2.5447 + 0.985(3.7028) = ANS.
# A tank contains exactly one kilogram of water consisting of liquid and vapor in # Atmospheric pressure boils at 212oF. At the vacuum pressure at 24 in Hg, the
equilibrium at 1MPa. If the liquid contains one third and the remaining is vapor of the volume temperature is 142oF. Find the boiling temperature when the pressure is increased by 40psia
of the tank, what is the enthalpy of the contents of the tank? from the atmospheric.
At 1MPa: vf = 0.0011273 vfg = 0.19444 SOLUTION
hf = 762.81 hfg = 2015.3 P2 = 14.7 + 40 = 54.7 psia
SOLUTION P1 = -24(14.7/29.92) + 14.7 = 2.908 psia
Let V = total volume of tank By interpolation:
mL = VL/vL = (V/3) / 0.0011273 = 295.69 V
mv = Vv / vv = (2V/3) / 0.1944 = 3.429V

t2 = ANS.
x= = ANS.
# A certain coal has the following ultimate analysis:
# Water substance at 70 bar and 65oC enters a boiler tube of constant inside diameter of
C = 69% N2 = 5% H2 = 2.5% S = 7%
35mm. The water leaves the boiler tube at 50 bar and 700oK at velocity of 100m/s. Calculate
Determine the amount of oxygen of the heating value of fuel is 26 961.45 KJ/kg.
the inlet volume flow(li/sec).
SOLUTION
SOLUTION
From steam tables:
At 70bar(7MPa) and 65oC Qh = 33 820C + 144 212 (H - ) + 9304S
v1 = 0.001017m3/kg 26961.45 = 33820(.69) + 144212 (0.025 – O/8) + 9304(0.07)
at 50bar(5MPa) and 700oK(427oC) O= ANS.
v2 = 0.06081m3/kg # A diesel engine consumed 945 liters of fuel per day at 35oC. If the fuel was purchased
m1 = m2 at 15.5oC and 30oAPI at P29.00/li, determine the cost of fuel to operate the engine per day.
Q1/ v1 = Q2/ v2 SOLUTION

16

V1 = 1.672m/sec
Q1 = A x v = (π/4)(0.035)2(1.672) = ANS.
# Steam leaves an industrial boiler at 827.4KPa and 171.6oC. A portion of the steam is
passed through a throttling calorimeter and is exhausted to the atmosphere when the
calorimeter pressure is 101.4KPa. How much moisture does the steam leaving the boiler
contain if the temperature of the steam at the calorimeter is 115.6oC?
At 827KPa(171.6oC): hf = 727.25 KJ/kg, hfg = 2043.2 KJ/kg
From table 3: At 101.4KPa and 115.6oC: h2 = 2707.6 KJ/kg
SOLUTION Cost = P29.00/li(935.44li) = ANS.
Let x = quality of steam entering the throttling calorimeter # A cylindrical tank 4m long and 3m diameter is used for oil storage. How many days
h1 = h2 can the tank supply the engine having 27oAPI with fuel consumption of 60kg.hr?
hfg + xhfg = h2 SOLUTION
727.25 + x(2043.2) = 2707.6 V = π/4 D2 h = π/4 (3)2(4) = 28.274 m3
x = 0.9692; y = 1 - 0.9692 = ANS.

Density of fuel = 0.89274(1000kg/m3) = 892.74kg/m3


w = m/V 0.14H2 + 0.070O2 = 0.14H2O
V = 60/892.74 = 0.0672 m3/hr 0.03CH4 + 0.060O2 = 0.03CO2 + 0.06H2O
Number of days = 28.274/0.0672 = 420.75hrs = ANS. 0.27Co + 0.135O2 = 0.27CO2
# A logging firm in Isabella operates a Diesel Electric Plant to supply its electric energy 0.265O2
requirements. During a 24 period, the plant consumed 250 gallons of fuel at 80oF and Actual O2 in product = 0.265 O2 – 0.006 O2 = 0.259 O2
produced 2900KW-hrs. Industrial fuel used is 30oAPI and was purchased at P30.00/li at 60oF. Molal A/F = 0.259 + 0.259(3.76) = ANS.
Determine the overall thermal efficiency of the plant. # A volumetric analysis of a gas mixture is as follows:
SOLUTION CO2: 12% N2: 80%
Qh = 41130 + 139.6xoAPI = 41130 + 139.6(30) = 45318 KJ/kg O2: 4% CO: 4%
60oF = 15.6oC What is the percentage of CO2 on a mass basis?
80oF = 26.6oC SOLUTION
Converting to mass basis:
CO2 = 0.12 x 44 = 5.28
O2 = 0.014x32 = 1.28
N2 = 0.82x28 = 22.96
At 26.6oC CO = 0.02x28 = 0.56
mf = 250gal/24hrs x 3.785li/gal x 0.869kg/li x 1hr/3600sec Total mass of product = 5.28 + 1.28 + 22.96 + 0.56 = 30.08 kg
mf = 0.00952 kg/sec % mass of CO2 = 5.28/30.08 = ANS.
load = 2900/24 = 120.833KW # The following coal has the following ultimate analysis by weight:
C = 70.5% H2 = 4.5% O2 = 6.0% N2 = 1.0%
overall efficiency = = ANS. S = 3.0% ash = 11% moisture = 4%
# The dry exhaust gas from oil engine has the following gravimetric analysis: A stocker fired boiler of 195000kg/hr steaming capacity uses this coal as fuel. Calculate
CO2 = 21.6% O2 = 4.2% N2 = 74.2% volume of air in m3/kg with air at 60oF and 14.7 psia pressure of boiler efficiency is 70% and
Specific heats at constant pressure for each component of the exhaust gas in Kcal/kgoC are: FE = 1.10.
CO2 = 0.203 O2 = 0.219 N2 = 0.248 SOLUTION
Calculate the specific gravity if the molecular weight of air is 28.97 kg/kg-mol.
SOULTION Theo. A/F = 11.5C + 34.5(H – O/8) + 4.3S
Converting the gravimetric analysis to volumetric:
CO2 = 0.219/44 = 0.004909 = 11.5(0.705) + 34.5(0.045 – 0.06/8) + 4.3(0.03)
O2 of nitrogen in lb/= 0.042/32 = 0.001312
N2 = 0.742/28 = 0.026500 = 9.53
0.032721 mols/kg-mol
Molecular weight = 1/0.032721 = 30.56kg/kg-mol Actual A/F = 9.53(1.3) = 12.389 kg air/kg fuel
SG = 30.56/28.97 = ANS.
# A bituminous coal has the following composition:
C = 71.5% H = 5.0% O = 7.0% N = 1.3% S = 3% Ash =
7.6%
W = 3.4%
Determine the theoretical weight of nitrogen in lb/lb of coal.
SOLUTION
Theo. A/F = 11.5C + 34.5(H – O/8) + 4.3S
= 11.5(0.715) + 34.5 (0.05 – 0.07/8) + 4.3(0.03)
= 9.8 lb air / lb coal
N2 in air by weight = 76.8%
therefore:
theoretical weight of N2 = 0.768(9.8) = ANS.
# A gaseous fuel mixture has a molal analysis:
H2 = 14% CH4 = 3% CO = 27%
O2 = 0.6% CO2 = 4.5% N2 = 50.9%
Determine the air fuel ratio for complete combustion of molal basis.
SOLUTION
Chemical reaction with oxygen:
h3 = 340.49 KJ/kg
; mf = 23107.56 (12.389) = 286279.57 kg/hr
h4 = hf + vf (P2 – P1) = 340.49 + 0.00103(2500 - 50) = 342.98 KJ/kg

101.325(V) = 286279.57(0.287)(15.6 + 273)

V= ANS. Efficiency = ANS.

# 23.5 kg of steam per second at 5MPa and 400oC is produced by a steam generator. # A two-stage air compressor air at 100 Kpa and 22⁰C discharges to 750Kpa. If intercooler
The feedwater enters the economizer at 145oC and leaves at 205oC. The steam leaves the intake is 105⁰C, determine the value of n.
boiler drum with a quality of 98%. The unit consumes 3kg of coal per second as received Solution:
having a heating value of 25102 KJ/kg. What would be the overall efficiency of the unit in
percent? Px = = 273.86 Kpa

Steam properties:

At 5MPa and 400oC: h = 3195.7KJ/kg At 5MPa: hf = 1154.23, hg =1640.1

At 205oC: hf = 875.04 At 145oC: hf = 610.63

SOLUTION 1.281 =
n = _____________ ANS.
# A single acting air compressor has a volumetric efficiency of 89%, operates at 500 rpm. It
takes in air at 100kpa and 30⁰C and discharges it at 600 Kpa. The air handled is 8 m3/min
= ANS.
measured at discharge condition. If compression is isentropic, find mean effective pressure in
Kpa
# In a Rankine cycle steam enters the turbine at 2.5MPa (enthalpies and entropies given) SOLUTION:
and condenser of 50KPa (properties given), what is the thermal efficiency of the cycle? P1V1k = P2V2k
100(V11.4) = 600(8)1.4
At 2.5MPa: hg = 2803.1 sg = 6.2575 V1 = 28.37687 m3/min
VD = 28.768/0.89 = 32.32 m3/min
At 50KPa: sf = 1.0910 sfg = 6.5029 hf = 340.49 hfg = 2305.4 vf =
0.0010300

SOLUTION W= =

h1 = 2803.1 KJ/kg
W = Pm x VD
solving for h2: 7562.19 = Pm x 32.32
Pm = ________ ANS.
s = sf + xsfg # A water-jacketed air compressor handles 0.343 m3/s of air at 96.5 kpa and 21⁰C and leaving
at 480 kpa and 132⁰C; 10.9 kg/h of cooling water enters the jacket at 15⁰C and leaves at 21⁰C.
6.2575 = 1.0910 + x(6.5029) Determine the compressor break power.
SOLUTION:
x = 0.7945

h2 = hf + xhfg = 340.49 + 0.7945(2305.4) = 2172.13 KJ/kg


n = 1.249

W= = 62.57 KW P1 = ___________ ANS.


Q = heat loss = mCp(t2 – t1) = (10.9/3600)(4.187)(21 – 15) = 0.076 KW # A submersible, multi-stage, centrifugal deep well pump 260 gpm capacity is installed in a
Brake power = W + Q = 62.57 + 0.076 = _______ ANS. well 27 feet below the static water level and running at 3000 rpm. Drawdown when pumping
# A double suction centrifugal pump delivers 20 ft3/sec of water at a head of 12 m and at rated capacity is 10 ft. the pump delivers the water into a 25,000 gallons capacity overhead
running at 650 rpm. What is the specific speed of the pump? storage tank. Total discharge head developed by pump, including friction in piping is 243 ft.
SOLUTION: Calculate the diameter of the impeller of this pump in inches if each impeller diameter
developed a head of 38 ft.
SOLUTION:
Ns = V=πDN

Q= V=
h = 12 x 3.281 = 39.37 ft
π D (3000/60) =
D = 0.315 ft = _________ ANS.
Ns = __________ANS. # A fan draws 1.42 m3/sec of air at a static pressure of 2.54 cm of water through a duct 300
# Determine the number of stages needed for a centrifugal pump if it is used to deliver 400 mm diameter and discharges it through a duct of 275 mm diameter. Determine the static fan
gal/min of water and pump power of 15 Hp. Each impeller develops a head of 30 ft. efficiency if total fan mechanical is 75% and air is measured at 25⁰C and 760 mmHg.
SOLUTION: SOLUTION:
Wp = w Q h
15 x 0.746 = 9.81(400 gal/min x 0.003785 m3/gal x 1/60) h
wa =
h =45.20 m x 3.281 ft/m = 148.317 ft
# A total head of fan is 187 m and has a static pressure of 210 mm of water gage, what is the
Number of stages = 148.317/30 = 4.94 stages
velocity of air flowing if density of air is 1.15 kg/m3?
# The suction pressure of a pump reads 3 in. of mercury vacuum and discharge pressure reads
SOLUTION:
140psi is use to deliver 120 gpm of water with specific volume of 0.0163 ft3/lb. Determine the
hs = 0.21(1000-1.15) = 182.61 m
pump work.
h = hs + hv
SOLUTION:
187 = 182.61 + hv
P1 = -3 in Hg x 101.325/29.92 = -10.15 Kpa
hv = 4.39 m
P2 = 140 psi x 101.325/14.7 = 965 Kpa
4.39 = v2 /2(9.81)
W = 1/v = 1/.0163 = 61.35 lb/ft3 x 9.81/62.4 = 9.645 KN/m3
v = _________ANS.
# A fan delivers 5.7 m3/sec at a static pressure of 5.08 cm of water when operating at a speed
h= = = 101.105 m of 400 rpm. The power input required is 2.963 KW. If 7.05 m3/sec are desired in the same fan
Q = 120 gal/min x 3.785 li/1gal x 1m3/1000li x 1/60 = 0.00757 m3/sec and installation, find the pressure in cm of water.
P = w Q h = 9.645(0.00757)(101.105) = ____________ ANS. SOLUTION:
# A submersible pump delivers 350 gpm of water to a height of 5 ft from the ground. The
pump was installed 150 ft below the ground level and a draw down of 8 ft during the
operation. If water level is 25 ft above the pump, determine the pump power.
SOLUTION:
h = 5+ 150 – (25 – 8) = 138/3.281 = 42.06 m
Q = 350 gal/min x 0.003785 m3/gal x 1min/60sec = 0.02246 m3/sec
Wp = w Q h = 9.819.02246)(42.06) = __________ ANS. N2 = 494.74 rpm
# A vacuum pump is used to drain a flooded mine shaft of 20⁰C water. The pump pressure of
water at this temperature is 2.34 Kpa. The pump is incapable of lifting the water higher than
16 m. What is the atmospheric pressure?
SOLUTION:

h2 = __________ANS.
# A rigid container is closed at one end and measures 8 in diameter by 12 in long. The d = 0.835 m = ________ANS.
container is held vertically and is slowly moved downward until the pressure in the container # The type of flow occupying in a 1 cm diameter pipe which water flows at a velocity of 2.50
is 17 psia. What will be the depth of the top of the container from the free water surface? m/s. Use v=1.13x10-6 m2/s for water.
SOLUTION: SOLUTION:
Patm = Pgage + Patm Re =dV/ v
17 = Pgage + 14.7
Pgage = 2.30 psi Re =
Pgage = w h Since it is greater than 2000, then it is _________ANS
2.30(144) = 62.4 (h) # An insulated rigid tank initially contains 1.5 lb of helium at 80⁰F and 50 psia. Apaddle
h = 5.3077ft x 12 =__________ans. wheel with power rating of 0.02 hp is operated within the tank for 30 min. Determine the final
# An empty, open can is 30 cm high with a 15 cm diameter. The can, with the open end temperature.
down, is pushed under water with a density of 1000 kg/m3. Find the water level in the can SOLUTION:
when the top of the can is 50 cm below the surface. W= ΔU = mcv(T2 – T1)
SOLUTION: 0.02hp (0.50 hr)(2545 Btu/hr/hp) = 1.5 (0.171)(T2 – 80)
Consider the water pressure T2 = __________ANS.
Pw = wh + 101.325 = (0.8 - x)(9.81) + 101.325 = 109.173 – 9.81x # A 4 m2 asphalt pavement with emissivity of 0.85 has a surface temperature of 50⁰C. Find
Consider the air pressure: the maximum rate of radiation that can be emmited from the surface.
P1V1 = P2V2 SOLUTION:
101.325(A x 0.3) = P2[A(0.3 – x)]
Qr = e ksvA
Ksv = 5.67 x 10-8 (Stefan Boltzman constant)
P2 =
Qr = 0.85(5.67 x 108)(4)(50 +273)4 = ____________ANS.
Pw = P2
# Air at 10⁰C and 80 kpa enters a diffuser of a jet engine steadily with a velocity of 200 m/s.
The inlet area of diffuser is 0.40 m2. Determine the mass flow rate of air.
109.173 – 9.81x = SOLUTION:
9.81x2 – 112.116x + 2.3705 = 0 w = P/RT = 80/0.287(10+273) = 0.985 kg/m3
By quadratic formula: m = wvA = 0.985(200)(0.40) = ___________ANS.
X = 0.02118 m = _____________ANS # Consider a refrigeration whose 40watts light bulb remains on continuously as a result of a
# A cylindrical pipe with water flowing downward at 0.03 m3/s having top diameter of 0.08, malfunction of the switch. If the refrigerator has a COP of 1.3 and the cost of electricity is 8
bottom diameter of 0.04 m and height of 1.5 m. Find the pressure between the pipe. cents per kw-hr, determine the increase in the energy consumption of the refrigeration and its
SOLUTION: cost per year if the switch is not fixed.
SOLUTION:
COP = Re / Wref
Z1 – Z2 = 1.5 m 1.3 = 40 / Wref
Z2 – Z1 = -1.5 m Wref = Wb + Wref = 40 + 30.769 = 70.77 watts
W = 0.07077 KW
V1 = Cost = 0.07077(8760)(P0.08) = ___________ANS.
V2 = # A 75 hp motor that has an efficiency of 91% is worn out and is replaced by a high-efficiency
motor that has an efficiency motor that has an efficiency of 95.4%. Determine the reduction in
heat gain of the room due to higher efficiency under full-load conditions.
+ (-1.5) SOLUTION:
P1 – P2 = __________ANS. P01 = (75x0.746)(0.91) = 50.91 KW
# Determine the size of pipe which will deliver 8 liters of medium oil (v = 6.10 x 10-6 m2/s) P02 = (75x0.746)(0.954) = 53.376 KW
assuming laminar flow conditions. Qreduced = 53.376 – 50.91 = _________ANS.
# A household refrigerator that has a power input of 450 watts and a COP of 2.5 is to cool five
large watermelons, 10 kg each, to 8⁰C. If the watermelons are initially at 20⁰C, determine
V=
how long will it take for the refrigerator to cool them. The watermelons can be treated as water
whose specific heat is 4.2 KJ/kg-⁰K.
Re = SOLUTION:
For laminar flow, Re = 2000 COP = Re/Wc
2.5 = Re/450
2000 = Re = 1,125 watts
Re = mcp(t2 – t1) Solution:
450t = (10 x 5)(4.2)(20 – 8)
t = ____________ANS.
# When a man returns to his wall-sealed house on a summer day, he finds that the house is at W = e Q = 0.7315 (3000) = 2194.5 Btu/s = _____________ANS
32⁰C. He returns on the air conditioner which cools the entire house to 20⁰C in 15 minutes. If A heat engine receives hat from a source at 1200°K at a rate of 500KJ/s and rejects the waste
COP is 2.5, determine the power drawn by the air conditioner. Assume the entire mass within heat to a medium at 300°K. The power output of the heat engine is 180 KW. Determine the
the house is 800 kg of ait which cv= 0.72 KJ/kg-K, cp=1.0 KJ/kg-K. irreversibility rate for this process.
SOLUTION: Solution:
Re = mcv(t2 – t1) = (800/15x60)(0.72)(32 – 20) e = (1200-300) / 1200 = 0.75
Re = 7.68 KW W= 0.75(500) = 375 KW
Wc = 7.68 / 2.5 = ___________ANS. Irreversibilities = 375-180 = _____________ANS
A dealer advertises that he has just received a shipment of electric resistance heaters for
A heat source at 800°K losses 2000 KJ of heat to a sink at 500°K. Determine the entropy residential buildings that have an efficiency of 100 percent. Assuming an indoor temperature
generated during the process. of 21°C and outdoor temperature of 10°C, determine the second law efficiency of these
Solution: heaters.
∆Ssource = -2000/800 = -2.5 Solution:
∆Ssink = 2000/500 = 4 COP1 = 100 efficient = 1
∆Sgen = -2.5/4 = _____________ANS COP2 = (21+273) / (21-10) = 26.72
Helium gas in compressed in an adiabatic compressor from an initial state of 14 psia and 50°F esl = COP1 / COP2 = 1 / 26.72 = _____________ANS
to a final temperature of 320°F in a reversible manner. Determine the exit pressure of Helium. A thermal power plant has a heat rate of 11,363 Btu/KW-hr. Find the thermal efficiency of the
Solution: plant.
T2/T1 = (P2/P1)n-1/n Solution:
(320+460)(50+460) = (P2/14)1.667-1/1.667 e = 3412 / Heat rate = 3412 / 11363 = _____________ANS
P2 = _____________ANS A rigid tank contains 2 kmol of N2 and 6 kmol of CO2 gases at 300°K and 115 Mpa. Find the
Air pass thru a nozzle with efficiency of 90%. The velocity of air at the exit is 600 m/s. Find tank volume using ideal gas equation
the actual Velocity at the exit. Solution:
Solution: Pm Vm = Nm Ru Tm
e = (Va/V3)2 15,000 Vm = (6+2)(8.314)(300)
0.9 = (Va/600)2 Vm = _____________ANS
Va = _____________ANS A spherical balloon with a diameter of 6 m is filled with helium at 20°C and 200 kpa.
A 50 kg block of iron casting at 500°K is thrown into a large lake that is at a temperature of Determine the mole number.
285°K. The iron block eventually reaches the thermal equilibrium with the lake water. Solution:
Assuming average specific heat of 0.45 KJ/kg-K for the iron, determine the entropy generated PV=NRT
during the process.
Solution:
(200) = N (8.314) (20+273)
∆Siron = m c ln(T2/T1) = 50(0.45)ln (285/500) = -12.65 KJ/K
N = _____________ANS
∆Slake = Q/T = [50(0.45)(500-285)] / 285 = 16.97 KJ/K
The air is an automobile tire with a volume of 0.53 ft3 is at 90°Fand 20 psig. Determine the
∆Sgen = -12.65 + 16.97 = _____________ANS
amount of air that must be added to raise the pressure to the recommended value of 30 psig.
A windmill with a 12 m diameter rotor is to be installed at a location where the wind is
Assume the atmospheric pressure to be 14.7 psia and the temperature and the volume to
blowing at an average velocity of 10 m/s. Using standard condition of air (1 atm, 25°C),
remain constant.
determine the maximum power that can be generated by the windmill.
Solution:
Solution:
PV=mRT
w = P/RT = 101.325 / (0.287)(25+273) = 1.1847 kg/m3
(20+14.7)(144)(0.53) = m1 (53.3)(90+460)
m = w A v = 1.1847(π/4 x 122)(10) = 1,339.895 kg/s
m1 = 0.09034 lb
KE = v2/2000 = 102/2000= 0.05 KJ/kg
(30+14.7)(144)(0.53) = m2 (53.3)(90+460)
Power = m KE = 1,339.895(0.05) = _____________ANS
m2 = 0.11634 lb
Consider a large furnace that can supply heat at a temperature of 2000°R at a steady rate of
madded = m2 – m1 = 0.11634 - 0.09034 = _____________ANS
3000 Btu/s. Determine the exergy of this energy. Assume an environment temperature of
A rigid tank contains 20 lbm of air at 20 psia and 70°F. More air is added to the tank until the
77°F.
pressure and temperature rise to 35 psia and 90°F, respectively. Determine the amount of air
added to the tank.
Solution:
P1 V1 = m1 R1 T1
(20 x 144) (V1) = 20(53.3)(70+460)
V = 196.17 ft3
P2 V2 = m2 R2 T2 t2 = _____________ANS
(35 x 144)(196.17) = m2(53.3)(90+460) For a heat transfer purposes, a standing man can be modeled as a 30 cm diameter, 170 cm long
m2 = 33.73 lbs vertical cylinder with both the top and bottom surfaces insulated and with the side surface at
madded = m2 – m1 = 33.73 – 20 = _____________ANS an average temperature of 34°C. For a convection heat transfer coefficient of 15 W/m2-°C,
A rigid tank contains 5 kg of an ideal gas at 4 atm and 40°C. Now a valve is opened and half determine the rate of heat loss from this man by convection in an environment at 20°C.
of mass of the gas is allowed to escape. If the final pressure in the tank is 1.5 atm, the final Solution:
temperature in the tank is: Qc = k A (t2-t1) = 15(π x 0.30 x 1.7) (34-20) = _____________ANS
Solution: A 5 cm diameter spherical ball whose surface is maintained at a temperature of 70°C is
PV=mRT suspended in the middle of a room at 20°C. If the convection heat transfer coefficient is 15
(4 x 9.81)(V) = 5(0.287)(40+273) W/m2-°C and the emissivity of the surface is 0.8, determine the total heat transfer from the
V = 11.446 m3 ball.
PV=mRT Solution:
(1.5 x 9.81)(11.446) = (5/2)(0.287)(T) A = 4 π r2 = 4 π (0.05)2 = 0.0314 m2
T = _____________ANS Qc = h A (t2-t1) = 15(0.0314) (70-20) = 23.56 watts
The pressure of an automobile tire is measured to be 200 kpa(gage) before the trip and 220 Qr = (0.80) (5.67 x 10-8) (0.0314) [(70+273)4-(50+273)4] = 9.22 watts
kpa(gage) after the tip at a location where the atmospheric pressure is 90 kpa. If the Q = Qc + Qr = 23.56 + 9.22 = _____________ANS
temperature of the air in the tire before the trip is 25°C, the air temperature after the trip is: A frictionless piston-cylinder device and a rigid tank contain 1.2 kmol of an ideal gas at the
Solution: same temperature, pressure, and volume. Now heat is transferred, and the temperature of both
T2/T1 = P2/P1 system is raised by 15°C. The amount of extra heat that must be supplied to the gas in the
T2 / (25+273) = (220+90) / (200+90) cylinder that is maintained at constant pressure.
T2 = 318.55°K Solution:
t2 = _____________ANS Q = m Cp (t2-t1) = (1.2 x 8.314)(1)(15) = _____________ANS
Water is boiling at 1 atm pressure in a stainless steel pan on an electric range. It is observed A supply of 50 kg of chicken at 6°C contained in a box to be frozen to -18°C in a freezer.
that 2 kg of liquid water evaporates in 30 min. The rate of heat transfer to the water is: Determine the amount of heat that needs to be removed. The latent heat of the chicken is 247
Solution: KJ/kg, and its specific heat is 3.32 KJ/kg-C above freezing and 1.77 KJ/kg-C below freezing.
The container box is 1.5 kg, and the specific heat of the box material is 1.4 KJ/kg-C. Also the
Q = mL = = _____________ANS freezing temperature of chicken is -2.8°C.
Consider a person standing in a breezy room at 20°C. Determine the total rate of heat transfer Solution:
from this person if the exposed surface area and the average outer surface temperature of the Qchicken = 50[3.32(6+2.8) + 247 + 1.77(-2.8+18)] = 15,156 KJ
person are 1.6 m2 and 29°C, respectively, and the convection heat transfer coefficient is 6W/ Qbox = 1.5(1.4)(6+18) = 50.4 KJ
m2 with emissivity factor of 0.95. Q = 15,156 + 50.4 = _____________ANS
Solution: Water is being heated in a closed pan on top of a range while being stirred by a paddle wheel.
Qc = h A (t2-t1) = (6)(1.6)(29.20) = 86.40 watts During the process, 30 KJ of heat is transferred to the water and 5 KJ of heat is lost to the
Qf = (0.95)(5.67 x 10-8)(1.6)[(29+273)4-(20+273)4] = 81.7 watts surrounding air. The paddle-wheel work amounts to 500 N-m. Determine the final energy of
Q = Qc + Qf = 86.40 + 81.7 = _____________ANS the system if its initial energy is 10 KJ.
Water is boiled in a pan on a stove at sea level. During 10 minutes of boiling, it is observed Solution:
that 200 grams of water has evaporated. Then the rate of heat transfer to the water is: Final energy = QA + ∆U – Qlose + W = 30 + 10 – 5 + 0.50 = _____________ANS
Solution: A classroom that normally contains 40 people is to be air-conditioned with window air-
Q = mL = (0.2/10) (2257) = _____________ANS conditioning units of 5 KW cooling capacity. A person at rest may be assumed to dissipate heat
An aluminum pan whose thermal conductivity is 237 W/m-C has a flat bottom whose diameter at rate of about 360 KJ/hr. There are 10 light bulbs in the room, each with a rating of 100
is 20 cm and thickness 0.4 cm. Heat is transferred steadily to boiling water in the pan through watts. The rate of heat transfer to the classroom through the walls and the windows is
its bottom at a rate of 500 watts. If the inner surface of the bottom of the pan is 105°C, estimated to be 15,000 KJ/hr. If the room to be maintained at a constant temperature of 21°C,
determine the temperature of the surface of the bottom of the pan. determine the number of window air-conditioning units required.
Solution: Solution:
A = π/4 (0.20)2 = 0.0314m2 Q = total heat load = 40(360/3600) + 10(0.100) +15,000/3600 = 9.167 KW
No. of air conditioning = 9.167/5 = 1.833 = _____________ANS
A 4m x 5m x 6m room is to be heated by a baseboard resistance heater. It is desired that the
resistance heater be able to raise the air temperature in the room from 7 to 23°C within 15
minutes. Assuming no heat losses from the room and an atmospheric pressure of 100 kpa,
determine the required power of the resistance heater. Assume constant specific heats at room Solution:
temperature. e = (TH-TL)/TH = [(28+273)-(15+273)(28+273)] = 0.03679
Solution: e = W/QA
w = P/RT = 100 / (0.287)(7+273) = 1.244 kg/m3 0.03679 = 10,000/QA
m = 1.244(4 x 5 x 6) = 149.28 kg QA = 271,812.99 KW
Q= m Cv (t2-t1) = 149.28(0.7186)(23-7) = 1,716.36 KJ QR = QA – W = 271,812.99 -10,000 = 261,813 KW
Power = 1,716.36 / (15 x 60) = _____________ANS QR = m Cp (∆t)
A student living in a 4m x 6m x 6m dormitory room turns on her 150 watts fan before she 261,813 = m (3)(4.187)
leaves the room on a summer day, hoping that the room will be cooler when she comes back in m = 20,843.32 kg/s
the evening. Assuming all the doors and windows are tightly closed and disregarding any heat 20,843.32 kg/s or 20,843.32 lit/s = 20.843 m3/s
transfer through walls and the windows, determine the temperature in the room when she h = P/w = 12/9.81 = 1.223 m
comes back 10 hours later. Use specific heat values at room temperature, and assume the room Wc = w Q h = 9.81(20.843)(1.223) = _____________ANS
to be at 100 kpa and 15°C in the morning when she leaves. A plane-type of solar energy collector with an absorbing surface covered by a glass plate is to
Solution: receive an incident radiation of 800 W/m2. The glass plate has a reflectivity of 0.12 and a
w = P/RT = 100 / (0.287)(15+273) = 1.2098 kg/m3 transmissivity of 0.85. The absorbing surface has an absorptivity of 0.90. The area of the
m = 1.2098(4 x 6 x 6) = 174.216 kg collector is 5 m2. How much solar energy in watts is absorbed by the collector?
Q= m Cv (t2-t1) Solution:
0.15(10 x 3600) = 174.216(0.7186)(t2 - 15) Q = heat absorbed from the sun
t2 = _____________ANS Q = 16.3(π/4 x 0.008)[15-(-190)] = _____________ANS
A piston-cylinder device whose piston is resting on top of a set of stops initially contains 0.50 An elastic sphere containing gas at 120 kPa has a diameter of 1.0 m. Heating the sphere causes
kg of helium gas at 100 kpa and 25°C. The mass of the piston is such that 500 kpa of pressure it to expand to a diameter of 1.3 m. During the process the pressure is proportional to the
is required to raise it. How much heat must be transferred to the helium before the piston starts sphere diameter. Calculate the work done by the gas in KJ.
rising? Solution:
Solution: PαD
For helium: Cv = R/(k-1) = (8.314/4)(1.667-1) = 3.116 KJ/kg-K P = kD
T2 = (25+273)(500/100) = 1,490°K 120 = k(1)
T1 = 25+273 = 298°K k = 120
Q= m Cv (T2-T1) = 0.50(3.116)(1490-298) = _____________ANS P = 120 D
In order to cool 1 ton (1000 kg) of water at 20°C in an insulated tank, a person pours 80 kg of V = 4/3 π (10/2)3
ice at -5°C into the water. Determine the final equilibrium temperature in the tank. The dV = 12/24 π D2 dD
melting temperature and the heat of fusion of ice at atmospheric pressure are 0°C and 333.7
KJ/kg, respectively.
Solution: W = _____________ANS
Qwater = Qice
1000(4.187)(20-tB) = 80(2.09)(0+5) + 80(333.7) + 80(4.187)(tB-0) # An ideal gas with a molecular weight of 7.1 kg/kg mol is compressed from 600 kPa and 280
tB = _____________ANS K to a final specific volume of 0.5 m3/kg. During the process the pressure varies according to
A fan is powerd by a 0.5 hp motor delivers air at a rate of of 85 m3/min. Determine the highest p = 620 + 150v + 95v2 where p is in kPa and v in m3/kg. Calculate the work of compression
value for the average velocity of air mobilized by the fan. Take the density of air to be 1.18 in KJ/kg.
kg/m3. Solution:
Solution: V1 = RT/P = (8.314/7.1)(280)/(600) = 0.546 m3/kg
P=wQh
0.50(0.746) = (1.18 x 0.00981)(85/80)(h) W=
__________________ANS.
= _____________ANS
An Ocean-Thermal Energy Conversion power plant generates 10,000 KW using a warm # A one cubic meter container contains a mixture of gases composed of 0.02 kg-mol of oxygen
surface water inlet temperature of 28°C and a cold deep-water temperature of 15°C. On the and 0.04 kg-mol of helium gas at a pressure of 220 kPa. What is the temperature of this ideal
basis of a 3°C drop in the temperature of the warm water and a 3°C rise in the temperature of gas mixture in degrees Kelvin?
the cold water due to removal and addition of heat, calculate the power required in KW to Solution:
pump the cold-deep water to the surface and through the system heat exchanger if the required V = V1 + V2
pumping pressure increase is 12 kpa.Assume a Carnot cycle efficiency and density of cold VT = m1R1T1/P1 + m2R2T2/P2
water to be 1000 kg/m3. 1 = (0.02x32)(8.314/32)(T)/220 + (0.04x32)(8.314/4)(T)/220
T = __________ANS.
Solution:
# Methyl alcohol (CH3OH) is burned with 25% excess air. How much unburned oxygen in kg- Cv of helium = 3118.9 J/kg-C
mol-oxygen/kg-mol-fuel will there be in the products if the combustion is complete?
m g h = m Cv
Solution:
CH3OH + O2 +3.76N2 = CO2 + H2O + 3.76N2 m (9.81) (4500) = m (3118.9)
CH3OH + 1.5O2 + (1.5) (3.76) N2 = 1CO2 + H2O + 1.5(3.76) N2
Consider 25% excess air: = ___________ANS.
CH3OH + 1.5O2 + 1.25(1.5) (3.76) N2 = 1CO2 + H2O + 1.25(1.5) (3.76) N2 + 0.25(1.5) O2
Unburned O2 = 0.25(1.5) = _________ANS. # Consider two Carnot heat engines operating in series. The first engine receives heat from the
reservoir at 2400 K and rejects the waste heat to another reservoir at temperature T. The
# A 12 DC electrical motor draws a current of 15 amps. How much work in KJ does this motor second engine receives heat by the first one, convert some of it to work, and rejects the rest to
produce over a 10-minute period of operation? a reservoir at 300K. If thermal efficiencies of both engines are the same, determine the
Solution: temperature T.
W=E=QV Solution:
W = (15 x 10 x 60)(12) = 108,000 J = _____________ANS. e1 = e2

# A 4liter (2-liter per revolution at standard pressure and temperature) spark ignition engine
has a compression ratio of 8 and 2200 KJ/kg heat addition by the fluid combustion. T = ___________ANS.
Considering a cold air-standard Otto cycle model, how much power will the engine produce
when operating at 2500 rpm? # An ideal gas mixtrure consists of 2 kmol of N2 and 6 mol of CO2. The mass fraction of CO2
Solution: is:
w= 1.2 kg/m3 (standard density of air) Solution:
m = 2 li/rev x 2500 rev/min / 1.2 kg/m3 x 1 m3/1000 li x 1 min/60 sec = 0.10 kg/s M = (2/8)(28) + (6/8)(44) = 40
e = W/QA R = 8.314/M = 8.314/40 = __________ANS.
e = 1 – 1/81.4-1 = 0.5647
0.5647 = W/2200 # A Carnot cycle operates between the temperature limits of 300K and 1500K, and produces
W = 1,242.34 KJ/kg (0.10 kg/s) = 124.23 KW = _____________ANS. 600 KW of net power. The rate of entropy change of the working fluid during the head
addition process is:
# A simple Rankine cycle produces a 40 MW of power, 50 MW of process heated and rejects Solution:
50MW of heat to the surroundings. What is the utilization factor of this cogeneration cycle W=
neglecting the pump work?
Solution: 600 =
QA = WT + Wprocess WP = 40 + 50 + 50 = 140 KW
= ___________ANS.
UF = (Qprocess + WT)/QA = (50 +40)/140 = _______________ANS.
# Air in an ideal Diesel cycle is compressed from 3 L to 0.15 L and then it expands during the
# The rate of heat transfer to the surroundings from a person at rest is about 400 KJ/hr.
constant pressure heat addition process to 0.3 L. Under cold air standard conditions, the
Suppose that the ventilation system fails in an auditorium containing 120 people and assuming
thermal efficiency of this cycle is:
that the energy goes into the air of volume 1500 m3 initially at 300K and 101 kPa, calculate
Solution:
the rate in °C/min of air temperature change.
rK = 3/0.15 = 20
Solution:
rC = 0.3 / 0.15 = 2
Q = m CV e = 1 – (1/rKk-1)[(rCK – 1)/k(rC – 1)] = 0.6467 = ________ANS.
PV = mRT
101(1500) = m(0.287)(300) # Helium gas in an ideal Otto cycle is compressed from 20°C and 2 L to 0.25 L and its
m = 1,759.58 kg temperature increases by an additional 800°C during the head addition process. The
temperature of helium before the expansion process is:
Q = m CV Solution:
120(400/60) = 1759.58(0.7186) rk = 2/0.25 = 8
T2 = (20 + 273)(8)1.667-1 = 1,172 K
= __________ANS. T3 = T2 + 800 = 1172 + 800 = 1972K = ______________ANS.

# An insulated box containing helium gas falls from a balloon 4.5 km above the earth’s # In an ideal Otto cycle, air is compressed from 1.20 kg/m3 and 2.2 L to 0.26 L and the net
surface. Calculate the temperature rise in °C of the helium when box hits the ground. work output of the cycle is 440 KJ/kg. The mean effective pressure for the cycle is:
Solution: # A thermoelectric refrigerator that resembles a small ice chest is powered by a car battery, and
VD = V2 – V1 = 2.2 x 10-3 m3 – 0.26 x 10-3 m3 = 1.94 x 10-3 m3 has a COP of 0.10. If the refrigerator cools a 0.350 L canned drink from 20°C to 4°C in 30
W = 440 KJ/kg (1.2 kg/m3 x 2.2 x 10-3 m3) = 1.1616 KJ min, determine the average electric power consumed by the thermoelectric refrigerator.
Pm = W/VD = 1.1616 / 1.94 x 10-3 = _____________ANS. Solution:
Q = m cp (t2 – t1) =[(1 x 0.35)/(30 x 60)] (4.187) (20-4) = 13 watts
# An ideal Brayton cycle has a net work output of 150 KJ/kg and backwork ratio of 0.4. Of COP = RE / WC
both the turbine and the compressor had an isentropic efficiency of 80%, the net work output 0.10 = 13/0.10 = __________ANS.
of the cycle would be?
Solution: MACHINE DESIGN ELEMENTS
Backwork Ratio = WC / WT
WC = 0.40 WT C # What is the polar section modulus of a 4-inch solid shaft?
Wnet = WT - WC A. 25.13 in3 B. 12.57 in4 C. 12.57 in3 D. 25.13 in4
Wnet = 250 KJ/kg A # A hollow shaft carries a torque 3.4 KN-m at a shearing stress if 55 Mpa. The outside
Wnet’ = 250(0.80) – 0.40(200)/0.80 = ___________ANS. diameter is 1.25 times that of the inside diameter. Find the inside diameter in mm.
A. 64.67 B. 46.67 C. 84.67 D. 74.84
# Air enters a turbojet engine at 200 m/s at a rate of 20 kg/s, and exits at 800 m/s relative to the A # It is specified that the angular deformation in a shaft should not exceed 1” in length of
aircraft. The thrust developed by the engine is: 1.8m. The allowable shearing stress is 53 Mpa. Determine the diameter of the shaft. The shaft
Solution: material has G = 77 x 106 Mpa.
Thrust developed = m (v2 – v1) = 20(800-200) = ____________ANS. A. 222.34 mm B. 234.22 mm C. 23.42 cm D. 24.22 cm
B # What modulus of elasticity in tension is required to obtain a unit deformation of 0.00105
# A thermal power plant has a net power 10 MW. The backwork ratio of the plant is 0.005. from a load producing a unit stress of 45,000 psi?
Determine the compressor work. A. 40 x 106 psi B. 43 x 106 psi C. 45 x 106 psi D. 46 x 106
Solution: A # A thrust washer has an inside diameter of 0.5 inch and an outside diameter of 3 inches. For
Wnet = WT + WP an allowable bearing pressure of 90 psi, how much axial load can it sustain?
BW = WP / WT A. 618.5 lb B. 537.2 lb C. 702.2 lb D. 871.2 lb
WP = 0.005WT D # An air cylinder has a bore of 25 mm and is operated with shop air at a pressure of 90 ps.
10,000 = WT – 0.005WT Find the push force extended by the piston rod in N.
WT = 10,050.25 KW A. 127 B. 70 C. 402 D. 305
WC = 0.005(10,050.25) = ____________ANS. B # A line shaft is to transmit 200 Hp at 900 rpm. Find the diameter of the shaft.
# A heat engine receives heat from a source at 1200K at a rate of 500 KJ/s and rejects the A. 2.18 inches B. 2.26 inches C. 3.18 inches D. 3.26 inches
waste heat to a sink at 300K. If the power output of the engine is 200 KW, the second law B # A main transmitting shaft transmits 350 KW to drive a generator at 2500 rpm, what is the
efficiency of this heat engine is? required diameter of the shaft?
Solution: A. 58.5 mm B. 82.7 mm C. 85.3 mm D. 56.2 mm
ea = 200/500 = 0.40 D # A round steel shaft rotates at 200 rpm and is subjected to a torque of 275 N-m and a
et = (TH - TL)/TH = (1200-300)/1200 = 0.75 bending moment of 415 N-m. Determine the equivalent twisting moment.
es = 0.40/0.75 = _________ANS. A. 597.84 N-m B. 456.42 N-m C. 546.43 N-m D. 497.85 N-m
A # A 4 inches shaft using a flat key, whose width is 1 inch, is transmitting a torque of 53,000
# A water reservoir contains 100,000 kg of water at an average elevation of 60m. The in-lb. If the design shearing stress is 5000 psi, determine the safe length of key.
maximum amount of electric power that can be generated from this water is: A. 6.3 inches B. 5.3 inches C. 4.3 inches D. 7.0 inches
Solution: A # a 75 mm diameter shaft is transmitting 300 KW at 600 rpm. A flange coupling is used and
P = mh = (100,000x0.00981)(60) = 58,860 KJ has 6 bolts, each 18 mm in diameter. Find the required diameter if the bolt circle based on an
P = 58,860 KJ x KWh/3600KJ = ____________ANS. average shearing stress of 27.5 Mpa.
A. 227.4 mm B. 477.2 mm C. 274.7 mm D. 247.7 mm
# A house is maintained at 22°C in winter by electric resistance heaters. If the outdoor C # A heavy duty shaft coupling is to be secured woth 25 mm bolts at a distance of 150 mm
temperature is 5°C, the second law efficiency of the resistance heaters is: from the shaft center. The shaft transmits 4330 KW of power at a speed of 1200 rp,. If the
Solution: allowable shearing stress for bolts is 100 Mpa, how many bolts are required?
ea = 100% resistance heaters A. 3 B. 6 C. 5 D. 4
et = (22-15)/(22+273) = 5.8% B # A 1.75-inch diameter shaft is supported by two sleeve bearings. The total load on the wo
es = 5.8/100 = _________ANS. bearings is 2900 lb. Find the friction power loss, in Hp, if the coefficient of friction between
shaft and bearing is 0.10 and the shaft rotates 200 rpm.
A. 0.88 Hp B. 0.78 Hp C. 0.98 Hp D. 0.68 Hp
B # Find the horsepower lost when a collar is loaded with 1000 lb, rotates at 25 rpm, and a diameter = 200 mm, length of conical elements in contact-125 mm, rotating speed = 870 rpm,
coefficient of friction at 0.15. The outside diameter of the collar is 4 inches and the inside coefficient of friction = 0.30, and allowable pressure = 70 kpa.
diameter is 2 inches. A. 19.2 KW B. 21.9 KW C. 29.1 KW D. 12.9 KW
A. 0.0629 Hp B. 0.0925 Hp C. 0.0269 Hp D. 0.0692 Hp D # A flywheel has a mean diameter of 4ft and is required to handle 2250 ft-lb of kinetic
C # A sleeve bearing has an outside diameter of 38.1 mm and a length of 50.1 mm, the wall energy. It has a width of 8 inches, mean operating speed is 300 rpm and the coefficient of
thickness is 3/16 inch. The bearing is subjected to radial load of 450 lb, determine the bearing fluctuation is to be 0.05. Find the weight of rim, assuming that the arms and hub are equivalent
pressure. to 10% of the total rim weight. The flywheel is made up of cast iron with specific weight of
A. 100 psi B. 150 psi C. 200 psi D. 250 psi 0.26 lb per cubic inch.
A # A vertical steel cylinder water tank is 30 m in diameter and 45 m high. The allowable A. 334 lb B. 434 lb C. 433 lb D. 343 lb
stress of the steel plate is 120 Mpa. Without reinforcing angle bars and rods, determine the A # A 20o involute spur gear has a tooth whole depth of 16.95 mm, a tooth thickness of 13.2
thickness of the steel plate. mm, and a pitch of 3. Determine the circular pitch of the gear.
A. 55.2 mm B. 56.2 mm C. 65.2 mm D. 52.6 mm A. 26.6 mm B. 16.6 mm C. 25.6 mm D. 24.6 mm
B # Determine the bursting steam pressure of a hemispherical steel shell with a diameter of C # A parallel helical gear-set consists of a 19-tooth pinion driving a 57-tooth gear. The pinion
100 inches and made of 0.0635 m thick steel plate. The joint efficiency is 70% and the tensile has a left-hand helix of 20o, a normal pressure angle of 14 1/2 deg . and a normal diametral
strength is 60,000 psi. pitch of 10 tooth/inch. If the pinion is to transmit 50 Hp at a speed of 1750 rpm, determine the
A. 4020 psi B.4200 psi C. 2500 psi D. 2040 psi center distance of the two gears.
A # A cylinder having an internal diameter of 508 mm and external diameter of 814.4 mm is A. 2.02 inch B. 6.06 inch C. 4.04 inch D. 2.06 inch
subjected to an internal pressure of 69 Mpa and an external of 14 Mpa. Determine the hoop A # A right-handed single-thread hardened steel worm has a catalog rating of 2.25 KW at 650
stress at the inner surface of the cylinder rpm when meshed with a 48-tooth cast-steel gear. The axial pitch of the worm is 25 mm,
A. 90.11 Mpa B. 91.10 Mpa C. 911.0 Mpa D. 19.10 Mpa normal pressure angle is 14.5o, and the pitch diameter of the worm is 100 mm. The coefficient
A # The root diameter of a double square thread is 0.55 inch. The screw has a pitch of 0.2 of friction is 0.085. Determine the shaft center distance.
inch. Find the outside diameter and the number of threads per inch. A. 241 mm B. 142 mm C. 412 mm D. 124 mm
A. 0.75 inch and 5 threads/inch C. 0.50 inch and 5 threads/inch A # A 20o straight-tooth bevel pinion having 14 teeth and a diametral pitch of 6 tooth/inch
B. 0.75 inch and 4 threads/inch D. 0.50 inch and 4 threads/inch drives a 42-tooth gear. The two shafts are at right angles and in the same plane. Find the pitch
B # Two shafts 3.6 m between centers carry pulleys 1.2 m in diameter and 0.91 m in diameter angle of the pinion.
respectively, connected by a crossed belt. It is desired to put the belt on as an open belt. How A. 18.4o B. 20o C. 14.5o D. 20.5o
long a piece must be cut of it? A # A triple thread worm has a lead angle of 17o and a pitch diameter of 2.2802 inches. Find
A. 300 mm B. 350 mm C. 400 mm D. 250 mm the center distance when the worm is mated with a wheel of 48 teeth.
B # A flat belt is 6 inches wide and 1/3 inch thick and transmits 15 Hp. The center distance is 8 A. 6.72 inches B. 7.26 inches C. 6.27 inches D. 7.62 inches
ft. The driving pulley is 6 inches in diameter and rotates at 2000 rpm such that the loose side B # A double thread worm has a pitch diameter of 3 inches. The wheel has 20 teeth and a pitch
of the belt is on top. The driven pulley is 18 inches in diameter. The belt material is 0.035 diameter of 5 inches. Find the gear helix angle.
lb/in3 and the coefficient of friction is 0.30. Determine the net belt tension. A. 4.69o B. 9.46o C. 6.49o D. 6.94o
A. 167.56 lb B. 157.56 lb C. 156.75 lb D. 175.56 lb C # What is the polar section modulus of a 4-inch solid shaft?
A # Two pulleys, 80 cm apart, carry a belt in an open connection. If the diameter of the pulleys A. 25.13 in3 B. 12.57 in4 C. 12.57in3 D. 25.13in4
are 40 cm and 15 cm, what is the length of the belt needed? A # A hollow shaft carries a torque 3.4 KN-m at a shearing stress Mpa. The outside diameter is
A. 248.35 cm B. 348.35 cm C. 265.45 cm D. 305.35 cm 1.25 times that of the inside diameter. Find the inside diameter in mm.
A # A pulley 600 mm in diameter transmits 40 KW at 500 rpm. The arc of contact between the A. 64.87 B. 46.87 C. 84.67 D. 74.64
belt and pulley is 144o , the coefficient of friction between belt and pulley is 0.35 and the safe A # It is specified that the angular deformation in a shaft should not to exceed 1” in length of
working stress of the belt is 2.1 Mpa. Determine the belt tension ratio, neglecting the effect of 1.8 m. The allowable shearing stress is 83 Mpa. Determine the diameter of the shaft. The shaft
centrifugal force. material has G = 77 x 108 Mpa.
A. 2.41 B. 2.14 C. 1.24 D. 4.12 A. 222.34 mm B. 234.22 mm C. 23.42 mm D. 24.22 mm
B # A roller chain and sprocket is to drive vertical discharge bucket elevator. The pitch of B # What modulus of elasticity in tension is required to obtain a unit deformation of 0.00105
chain connecting sprockets is 1.75”. The sprocket is rotating at 120 rpm and has 11 teeth while from a load producing a unit stress of 45,000 psi?
the driven sprocket is rotating at 38 rpm. Determine the number of teeth of driven sprocket. A. 40 x 106 psi B. 43 x 106 psi C. 45 x 106 psi D. 46 x 106 psi
A. 33 teeth B. 35 teeth C. 30 teeth D. 37 teeth A # A thrust washer has an inside diameter of 0.5 inch and an outside diameter of 3 inches. For
C # A disc clutch has 6 pairs of contacting friction surfaces with an outside diameter of 200 an allowable bearing pressure of 90 psi, how much axial load can it sustain?
mm and an inside diameter of 100 mm. The coefficient of friction of the clutch materials is 0.4 A. 618.5 lb B. 537.2 lb C. 702.2 lb D. 871.2 lb
and the axial force is 1500 N. The shaft speed is 1200 rpm. Find the Hp that can be transmitted D # An air cylinder has a bore of 25 mm and is operated with shop air at a pressure of 90 psi.
by the clutch assuming uniform pressure. Find the push force exerted by the piston rod in N.
A. 35.2 Hp B. 23.5 Hp C. 47.2 Hp D. 27.4 Hp A. 127 B. 70 C. 402 D. 305
A # Determine the power capacity of a cone clutch under uniform pressure and assuming the B # A line shaft is to transmit 200 Hp at 900 rpm. Find the diameter of the shaft.
following pressure and assuming the following conditions; major diameter = 250 mm, minor A. 2.18 inches B. 2.28 inches C. 3.18 inches D. 3.28 inches
B # A main transmitting shaft transmits 350 KW to drive a generator at 2500 rpm, what is the B # Two shafts 3.6 m between centers carry pulleys 1.2 m in diameter and 0.91 m in diameter
required diameter of the shaft? respectively, connected by a crossed belt. It is desired to put the belt on as an open belt. How
A.58.5 mm B. 62.7 mm C. 65.3 mm D. 56.2 mm long a piece must be cut of it?
D # A round steel shaft rotates at 200 rpm and is subjected to a torque of 275 N-m and a A. 300 mm B. 350 mm C. 400 mm D. 250 mm
bending moment of 415 N-m. Determine the equivalent twisting moment. B # A flat belt is 6 inches wide and 1/3 inch thick and transmits 15 Hp. The center distance is 8
A. 597.84 N-m B. 456.42 N-m C. 546.43 N-m D. 497.85 N-m ft. The driving pulley is 6 inches in diameter and rotates at 2000 rpm such that the loose side
A # A 4 inches shaft using a flat key, whose width is 1 inch, is transmitting a torque of 63,000 of the belt is on top. The driven pulley is 18 inches in diameter. The belt material is 0.035
in-lb. If the design shearing stress is 5000 psi, determine the safe length of key. lb/in3 and the coefficient of friction is 0.30. Determine the net belt tension.
A. 6.3 inches B. 5.3 inches C. 4.3 inches D. 7.0 inches A. 167.56 lb B. 157.56 lb C. 156.75 lb D. 175.56 lb
A # A 1200 mm cast iron pulley is fastened to a 112.5 mm shaft by means of a 28.13 mm A # Two pulleys, 80 cm apart, carry a belt in an open connection. If the diameter of the pulleys
square key 175 mm long. The key and the shaft have a shearing stress of 14,000 psi. are 40 cm and 15 cm, what is the length of the belt needed?
Determine the force acting at the pulley rim that will shear the key. A. 248.35 cm B. 348.35 cm C. 265.45 cm D. 305.35 cm
A. 10015 lb B. 11005 lb C. 11050 lb D. 10501 lb A # A pulley 600 mm in diameter transmits 40 KW at 500 rpm. The arc of contact between the
A # A 75 mm diameter shaft is transmitting 300 KW at 600 rpm. A flange coupling is used and belt and pulley is 144o , the coefficient of friction between belt and pulley is 0.35 and the safe
has 6 bolts, each 18 mm in diameter. Find the required diameter of the bolt circle based on an working stress of the belt is 2.1 Mpa. Determine the belt tension ratio, neglecting the effect of
average shearing stress of 27.5 Mpa. centrifugal force.
A. 227.4 mm B. 477.2 mm C. 274.7 mm D. 247.7 mm A. 2.41 B. 2.14 C. 1.24 D. 4.12
C # A heavy duty shaft coupling is to be secured with 25 mm bolts at a distance of 150 mm B # A roller chain and sprocket is to drive vertical discharge bucket elevator. The pitch of
from the shaft center. The shaft transmits 4330 KW of power at a speed of 1200 rpm. If the chain connecting sprockets is 1.75”. The sprocket is rotating at 120 rpm and has 11 teeth while
allowable shearing stress for bolts is 100 Mpa, how many bolts are required? the driven sprocket is rotating at 38 rpm. Determine the number of teeth of driven sprocket.
A. 3 B. 6 C. 5 D. 4 A. 33 teeth B. 35 teeth C. 30 teeth D. 37 teeth
B # A 1.75-inch diameter shaft is supported by two sleeve bearings. The total load on the two C # A disc clutch has 6 pairs of contacting friction surfaces with an outside diameter of 200
bearings is 2800 lb. Find the friction power loss, in Hp, if the coefficient of friction between mm and an inside diameter of 100 mm. The coefficient of friction of the clutch materials is 0.4
shaft and bearing is 0.10 and the shaft rotates 200rpm. and the axial force is 1500 N. The shaft speed is 1200 rpm. Find the Hp that can be transmitted
A. 0.88 Hp B. 0.78 Hp C. 0.98 Hp D. 0.68 Hp by the clutch assuming uniform pressure.
B # Find the horsepower lost when a collar is loaded with 1000 lb, rotates at 25 rpm, and a A. 35.2 Hp B. 23.5 Hp C. 47.2 Hp D. 27.4 Hp
coefficient of friction at 0.15. The outside diameter of the collar is 4 inches and the inside A # Determine the power capacity of a cone clutch under uniform pressure and assuming the
diameter is 2 inches. following pressure and assuming the following conditions; major diameter = 250 mm, minor
A. 0.0629 Hp B. 0.0926 Hp C. 0.0269 Hp D. 0.0692 Hp diameter = 200 mm, length of conical elements in contact-125 mm, rotating speed = 870 rpm,
C # A sleeve bearing has an outside diameter of 38.1 mm and a length of 50.1 mm, the wall coefficient of friction = 0.30, and allowable pressure = 70 kpa.
thickness is 3/16 inch. The bearing is subjected to radial load of 450 lb, determine the bearing A. 19.2 KW B. 21.9 KW C. 29.1 KW D. 12.9 KW
pressure. D # A flywheel has a mean diameter of 4ft and is required to handle 2250 ft-lb of kinetic
A. 100 psi B. 150 psi C. 200 psi D. 250 psi energy. It has a width of 8 inches, mean operating speed is 300 rpm and the coefficient of
A # A vertical steel cylinder water tank is 30 m in diameter and 45 m high. The allowable fluctuation is to be 0.05. Find the weight of rim, assuming that the arms and hub are equivalent
stress of the steel plate is 120 Mpa. Without reinforcing angle bars and rods, determine the to 10% of the total rim weight. The flywheel is made up of cast iron with specific weight of
thickness of the steel plate. 0.26 lb per cubic inch.
A. 55.2 mm B. 56.2 mm C. 65.2 mm D. 52.6 mm A. 334 lb B. 434 lb C. 433 lb D. 343 lb
B # Determine the bursting steam pressure of a hemispherical steel shell with a diameter of A # A 20o involute spur gear has a tooth whole depth of 16.95 mm, a tooth thickness of 13.2
100 inches and made of 0.0635 m thick steel plate. The joint efficiency is 70% and the tensile mm, and a pitch of 3. Determine the circular pitch of the gear.
strength is 98 psi. A. 26.6 mm B. 16.6 mm C. 25.6 mm D. 24.6 mm
A. 4020 psi B. 4200 psi C. 2500 psi D. 2040 psi C # A parallel helical gear-set consists of a 19-tooth pinion driving a 57-tooth gear. The pinion
A # A cylinder having an internal diameter of 508 mm and external diameter of 914.4 mm is has a left-hand helix of 20o, a normal pressure angle of 14 1/2 deg . and a normal diametral
subjected to an internal pressure of 69 Mpa and an external of 14 Mpa. Determine the hoop pitch of 10 tooth/inch. If the pinion is to transmit 50 Hp at a speed of 1750 rpm, determine the
stress at the side surface of the cylinder. center distance of the two gears.
A. 2.02 inch B. 6.06 inch C. 4.04 inch D. 2.06 inch
A. 90.11 Mpa B. 91.10 Mpa C. 911.0 Mpa D. 19.10 Mpa A # A right-handed single-thread hardened steel worm has a catalog rating of 2.25 KW at 650
A # The root diameter of a double square thread is 0.55 inch. The screw has a pitch of 0.2 rpm when meshed with a 48-tooth cast-steel gear. The axial pitch of the worm is 25 mm,
inch. Find the outside diameter and the number of threads per inch. normal pressure angle is 14.5o, and the pitch diameter of the worm is 100 mm. The coefficient
A. 0.75 inch and 5 threads/inch C. 0.50 inch and 5 threads/inch of friction is 0.085. Determine the shaft center distance.
A. 0.75 inch and 4 threads/inch D. 0.50 inch and 4 threads/inch A. 241 mm B. 142 mm C. 412 mm D. 124 mm
A # A 20o straight-tooth bevel pinion having 14 teeth and a diametral pitch of 6 tooth/inch B # A 1.75-inch diameter shaft is supported by two sleeve bearings. The total load on the two
drives a 42-tooth gear. The two shafts are at right angles and in the same plane. Find the pitch bearings is 2800 lb. Find the friction power loss, in Hp, if the coefficient of friction between
angle of the pinion. shaft and bearing is 0.10 and the shaft rotates 200rpm.
A. 18.4o B. 20o C. 14.5o D. 20.5o A. 0.88 Hp B. 0.78 Hp C. 0.98 Hp D. 0.68 Hp
A # A triple thread worm has a lead angle of 17o and a pitch diameter of 2.2802 inches. Find B # Find the horsepower lost when a collar is loaded with 1000 lb, rotates at 25 rpm, and a
the center distance when the worm is mated with a wheel of 48 teeth. coefficient of friction at 0.15. The outside diameter of the collar is 4 inches and the inside
A. 6.72 inches B. 7.26 inches C. 6.27 inches D. 7.62 inches diameter is 2 inches.
B # A double thread worm has a pitch diameter of 3 inches. The wheel has 20 teeth and a pitch A. 0.0629 Hp B. 0.0926 Hp C. 0.0269 Hp D. 0.0692 Hp
diameter of 5 inches. Find the gear helix angle. C # A sleeve bearing has an outside diameter of 38.1 mm and a length of 50.1 mm, the wall
A. 4.69o B. 9.46o C. 6.49o D. 6.94o thickness is 3/16 inch. The bearing is subjected to radial load of 450 lb, determine the bearing
C # What is the polar section modulus of a 4-inch solid shaft? pressure.
A. 25.13 in3 B. 12.57 in4 C. 12.57in3 D. 25.13in4 A. 100 psi B. 150 psi C. 200 psi D. 250 psi
A # A hollow shaft carries a torque 3.4 KN-m at a shearing stress Mpa. The outside diameter is A # A vertical steel cylinder water tank is 30 m in diameter and 45 m high. The allowable
1.25 times that of the inside diameter. Find the inside diameter in mm. stress of the steel plate is 120 Mpa. Without reinforcing angle bars and rods, determine the
A. 64.87 B. 46.87 C. 84.67 D. 74.64 thickness of the steel plate.
A # It is specified that the angular deformation in a shaft should not to exceed 1” in length of A. 55.2 mm B. 56.2 mm C. 65.2 mm D. 52.6 mm
1.8 m. The allowable shearing stress is 83 Mpa. Determine the diameter of the shaft. The shaft B # Determine the bursting steam pressure of a hemispherical steel shell with a diameter of
material has G = 77 x 108 Mpa. 100 inches and made of 0.0635 m thick steel plate. The joint efficiency is 70% and the tensile
A. 222.34 mm B. 234.22 mm C. 23.42 mm D. 24.22 mm strength is 98 psi.
B # What modulus of elasticity in tension is required to obtain a unit deformation of 0.00105 A. 4020 psi B. 4200 psi C. 2500 psi D. 2040 psi
from a load producing a unit stress of 45,000 psi? A # A cylinder having an internal diameter of 508 mm and external diameter of 914.4 mm is
A. 40 x 106 psi B. 43 x 106 psi C. 45 x 106 psi D. 46 x 106 psi subjected to an internal pressure of 69 Mpa and an external of 14 Mpa. Determine the hoop
A # A thrust washer has an inside diameter of 0.5 inch and an outside diameter of 3 inches. For stress at the side surface of the cylinder.
an allowable bearing pressure of 90 psi, how much axial load can it sustain? D # A 35-tooth pinion turning at 300 rpm drives a 120-tooth gear of 14.5° involute full depth
A. 618.5 lb B. 537.2 lb C. 702.2 lb D. 871.2 lb pressure angle. Determine the rpm of the driven gear.
D # An air cylinder has a bore of 25 mm and is operated with shop air at a pressure of 90 psi. A. 60 rpm B. 45 rpm C. 75 rpm D. 90 rpm
Find the push force exerted by the piston rod in N. B # Two parallel shafts connected by cylinders in pure rolling contact and turning in the same
A. 127 B. 70 C. 402 D. 305 direction having a speed ratio of 2.75. Determine the center distance of the two shafts if the
B # A line shaft is to transmit 200 Hp at 900 rpm. Find the diameter of the shaft. diameter of the smaller cylinder is 22 cm.
A. 2.18 inches B. 2.28 inches C. 3.18 inches D. 3.28 inches A.18.25 B. 19.25 cm C. 20.25 cm D. 17.25 cm
B # A main transmitting shaft transmits 350 KW to drive a generator at 2500 rpm, what is the A # Three extension springs are hooked in parallel that supports a single weight of 100 kg. The
required diameter of the shaft? first spring is rated at 4 KN/m and the other two springs are rated at 6 KN/m each. Determine
A. 58.5 mm B. 62.7 mm C. 65.3 mm D. 56.2 mm the equivalent stiffness of the three springs.
D # A round steel shaft rotates at 200 rpm and is subjected to a torque of 275 N-m and a A. 1.71 KN/m B. 5 KN/m C. 2.71 KN/m D. 3.71 KN/m
bending moment of 415 N-m. Determine the equivalent twisting moment. B # Three extension springs are hooked in parallel that support a single weight of 100 kg. The
A. 597.84 N-m B. 456.42 N-m C. 546.43 N-m D. 497.85 N-m springs are rated 4 KN/m, 5 KN/m, and 6 KN/m, respectively. Determine the equivalent spring
A # A 4 inches shaft using a flat key, whose width is 1 inch, is transmitting a torque of 63,000 constant of the three springs.
in-lb. If the design shearing stress is 5000 psi, determine the safe length of key. A. 10 KN/m B. 15 KN/m C. 9 KN/m D. 11 KN/m
A. 6.3 inches B. 5.3 inches C. 4.3 inches D. 7.0 inches A # Two extension coil springs are hooked in series that support a single weight of 100 kg.
A # A 1200 mm cast iron pulley is fastened to a 112.5 mm shaft by means of a 28.13 mm The first spring is rated at 4 KN/m and the other spring is rated at 6 KN/m. Determine the total
square key 175 mm long. The key and the shaft have a shearing stress of 14,000 psi. deflection of the springs.
Determine the force acting at the pulley rim that will shear the key. A. 410 mm B. 310 mm C. 510 mm D. 210 mm
A. 10015 lb B. 11005 lb C. 11050 lb D. 10501 lb A # If the ultimate shear strength of a steel plate is 42,000 psi, what force is necessary to
A # A 75 mm diameter shaft is transmitting 300 KW at 600 rpm. A flange coupling is used and
has 6 bolts, each 18 mm in diameter. Find the required diameter of the bolt circle based on an punch inch diameter hole in a 5/8 inch thick plate/
average shearing stress of 27.5 Mpa. A. 61,850 lb B. 65,810 lb C. 61,580 lb D. 60,185 lb
A. 227.4 mm B. 477.2 mm C. 274.7 mm D. 247.7 mm C # In a straight bevel gear, the angle between an element of the pitch cone and an element of
C # A heavy duty shaft coupling is to be secured with 25 mm bolts at a distance of 150 mm the face cone is called:
from the shaft center. The shaft transmits 4330 KW of power at a speed of 1200 rpm. If the A. Face angle B. Pitch angle C. Addendum angle D. Dedendum
allowable shearing stress for bolts is 100 Mpa, how many bolts are required? angle
A. 3 B. 6 C. 5 D. 4 D # A four bar mechanism in which one of the links can perform a full rotation relative to the
other three links.
A. Geneva mechanism C. Triple rocker mechanism A # It is the capacity of a material to absorb energy when it is deformed elastically and then,
B. Crossover-position mechanism D. Grashof mechanism upon unloading, to release this energy.
A # A Grashof four-bar mechanism in which the shorted link is the frame or the fixed link and A. Resilience B. Toughness C. Rigidity D. Ductility
the other two cranks completely rotate with their axes. D # It is the strain energy per unit volume required to stress a material from an unloaded state
A. Drank link mechanism C. Double-rocker mechanism to the point of yielding.
B. Crank-rocker mechanism D. Triple-rocker mechanism A. Modulus of roughness C. Modulus of rigidity
C # “For planar four bar linkage, the sum of the shortest and the longest lengths cannot be B. Modulus of elasticity D. Modulus of resilience
greater than the sum of the remaining two links” lengths if there is to be a continuous relative A # The ability of the material to absorb energy up to fracture.
rotation between the two member” The preceding statement is known as: A. Toughness B. Rigidity C. Resilience D. Stiffness
A. Grubler’s law B. Coriolli’s law C. Grashof’s law D. B # The Maximum Shear Stress Theory, as a failure prediction theory, is also known as:
Freudentein’s law A. von Mises criterion B. Tresca yield criterion C. Coulomb-Mohr
B # Which of the following is not true for an instant center or centro of planar linkages? Theory
A. Centro is a point common to two bodies having the same velocity in each. D. Modified Mohr Theory
B. Centro is a point in one body about which another body does tend to rotate. B # A failure prediction theory, which states that a part subjected to any combination of loads,
C. Centro is a point in one body about which another body actually turns. will fail (by yielding or fracturing) whenever the maximum shear stress exceeds a critical
D. Centro is a point in one body about which another body tends to turn. value.
A # The most common work holding devices of a shaper machine with the base graduated in A. Distorsion-energy theory C. Internal friction theory
degrees that make it possible to swivel any angle. B. Maximim-shear-stress theory D, Modified Mohr theory
A. Shaper vise B. Parallel bars and hold down bars C. Lathe holder D. A # A theory in a cyclic and impact loading, which states that damage at any stress level is
Swivel head proportional ro the number of cycles.
A # Shaper operation which is shaping the given stock and having the excess material A, Miner’s rule B. Paris Power Law C. Goodman Rule D. Manson-
remain with a tolerable allowance for finishing. Coffin Relationship
A. Roughing B. Finishing C. Angular Cutting D. Contouring B # A journal bearing where the radius of the journal is less than the radius of the bushing or
C # A cutting tool that has two or more cutting edges as in drill presses and milling machine bearing.
cutters. A. Fitted journal bearing C. Partial journal bearing
A. Grinder B. Single-point cutting tool C. Multi-point cutting tool D, B. Clearance journal bearing D. Full journal bearing
Two-point cutting tool B # A lubrication where the load-carrying surfaces of the bearing are separated by a relatively
A # The trade name for a patented alloy made up chiefly of cobalt, chromium, and tungsten thick film of lubricant, so as to prevent metal to metal contact, and where the stability of the
varying proportions. film can be explained by the laws of fluid mechanics.
A. Stellite B. Carbology C. Stainless Steel D. Copper A. Hydrostatic lubrication C. Elastohydrodynamic lubrication
A # The transformation of concepts and ideas into useful machinery is called as: B. Hydrodynamic lubrication D. Boundary lubrication
A. Design B.Synthesis C. Analysis D. Theorem A # A lubrication condition where non-conformal surfaces are completely separated by
C # It is a combination of mechanisms and other components that transform, transmits, or uses lubricant film and no asperities are in contact.
energy, load, or motion for a specific purpose. A. Elastohydrodynamic lubrication C. Hydrodynamic lubrication
A. Mechanism B. Engine C. Machine D. Linkage B. Boundary lubrication D. Hydrostatic lubrication
C # A speed at which rotating shaft becomes dynamically unstable.
B # It is defined as synergistic collection of machine elements; synergistic because as a design A. Normal Speed B. Variable Speed C. Critical Speed D. Average
it represents an idea or concept greater than the sum of the individual parts. Speed
A. System of mechanism B. Mechanical system C. Design system D. B # A ball bearing with race contacting pronounced groove for rolling elements.
Expert system A. Crown bearing B. Conrad bearing C. Angular-contact bearing D.
C # It may be defined the displacement per length produced in a solid as the result of stress. Cylindrical bearing
A. DeformationB. Elongation C. Strain D. Stress A # A machining process for producing internal straight cylindrical surface or profiles, with
D # The combination of applied normal and shear stresses that produces maximum principal process characteristics and tooling similar to those for turning operations.
normal stress, with a third principal stress between or equivalent to the extremes. A. Boring B. Drilling C. Reaming D. Milling
A. Principal shear stress C. Maximum shear stress B # A machining operation for all types of metallic and nonmetallic materials and is capable of
B. Principal normal stress D, Bending and shear stress producing circular parts with straight of various profiles.
C # It is a load applied transversely to longitudinal axis of member. A. Boring B. Turning C. Drilling D. Milling
A. Combined loads B. Concentrated load C. Bending load B # A set of specification for parts, materials, or processes intended to achieve uniformity,
D. Distributed load efficiency, and a specified quality.
C # It is the intensity and direction of internal force acting at a given point on particular plane. A. Code B. Standard C. Law D. Theorem
A. Load B. Strain C. Stress D. Sustained load
A # A set of specification for the analysis, design, manufacture, and construction of something; # A journal bearing where the radius of the journal is less than the radius of the bushing or
the purpose of which is to achieve a specified degree of safety, efficiency, and performance or bearing.
quality. A. Fitted journal bearing C. Partial journal bearing
A. Code B. Standard C. Law D. Theorem B. Clearance journal bearing D. Full journal bearing
# It is defined as synergistic collection of machine elements; synergistic because as a design it
represents an idea or concept greater than the sum of the individual parts. # A lubrication where the load-carrying surfaces of the bearing are separated by a relatively
A. System of mechanism B. Mechanical system C. Design system thick film of lubricant, so as to prevent metal to metal contact, and where the stability of the
D. Expert system film can be explained by the laws of fluid mechanics.
A. Hydrostatic lubrication C. Elastohydrodynamic lubrication
# It may be define the displacement per length produced in a solid as a result of stress. B. Hydrodynamic lubrication D. Boundary lubrication
A. Deformation B. Elongation C. Strain D. Stress
# A lubrication condition where non-conformal surfaces are completely separated by lubricant
# The combination of applied normal and shear stresses that produces maximum principal film and no asperities are in contact.
normal stress, with a third principal stress between or equivalent to the extremes. A. Elastohydrodynamic lubrication C. Hydrodynamic lubrication
A. Principal shear stress C. Maximum shear stress B. Boundary lubrication D. Hydrostatic lubrication
B. Principal normal stress D. Bending and shear stress
# A speed at which rotating shaft becomes dynamic unstable.
# It is a load applied transversely to longitudinal axis of member. A. Normal speed B. Variable speed C. Critical speed D. Average speed
A. Combined load B. Concentrated load C. Bending load
D. Distributed load. # A ball bearing with race containing pronounced groove for rolling elements.
A. Crown bearing B.Conrad bearing C.Angular-contact bearing
# It is the intensity and direction of internal force acting at given point on particular plane. D.Cylindrical bearing
A. Load B. Strain C. Stress D. Sustained load
# A machining process for producing internal straight cylindrical surface or profiles, with
# It is the capacity of a material to absorb energy when it is deformed elasticity and then, process characteristics and tooling similar to those for turning operations.
upon unloading, to release this energy. A.Boring B.Drilling C.Reaming D.Milling
A. Resilience B. Toughness C. Rigidity D. Ductility
# A machining operation for all types of metallic and nonmetallic materials and is capable of
# It is the strain energy per unit volume required to stress a material from an unloaded state to producing circular parts with straight or various profiles.
the point of yielding. A.Boring B.Turning C.Drilling D.Milling
A. Modulus of roughness B. Modulus of elasticity C. Modulus of rigidity # An American nonprofit society, founded in 1921, whose objectives are to improve and
D. Modulus of resilience advance the use of fabricated structural stress.
A. American Iron Steel Institute (AISI)
# The ability of the material to absorb energy up to the fracture. B. American Institute of Steel Construction (AISC)
A. Toughness B. Rigidity C. Resilience D. Stiffness C. American Society for Metals (ASM)
D. American Society of Testing and Materials (ASTM)
# The Maximum Shear Stress Theory, as a failure prediction theory, is also known as:
A. von Mises criterion B. Tresca yield criterion C. Coulomb-Mohr theory # A sketch of a machine, a machine element, or part of a machine element that shows all
D. Modified Mohr-theory acting forces, such as applied load and gravity forces, and all reactive forces.
A. Schematic diagram B. Free body diagram C. Moment diagram
# A failure prediction theory, which states that a part subjected to any combination of D. Skeletal diagram
loads, will fail (by yielding and fracturing) whenever the maximum shear stress exceeds a
critical value. # The size to which a limit of deviations is assigned and is the same for both members of the
A. Distorsion-energy theory C. Internal friction theory fit, it is the exact theoretical size.
B. Maximum-shear-stress theory D. Modified Mohr theory A. Nominal stress B. Basic size C. Maximum size D. Minimum size

# A theory in cyclic and impact loading, which states that damage at any stress level is # The algebraic difference between a size and the corresponding basic size.
proportional to the number of cycles. A. Tolerance B. Allowance C. Deviation D. Limit
A. Mine’s rule B. Paris Power Law C. Goodman Rule
D. Manson-coffin Relationship # The algebraic difference between the maximum limit and the corresponding basic.
A. Fundamental deviation B. Upper Deviation C. Lower deviation D.
Tolerance
# A principal or method that a deflection at any point in bar is equal to sum f deflection caused
# The algebraic difference between the minimum limit and the corresponding basic. by each load acting separately.
A. Fundamental deviation B. Upper Deviation C. Lower deviation D. A. Summation Method B. Method of balancing C. Method of superposition
Tolerance D. Shear and Moment diagram method

# Either the upper of the lower deviation, depending on which is closer to the basic size. # A failure prediction theory in which failure is caused by the elastic energy associated with
A. Fundamental deviation B. Upper Deviation C. Lower deviation D. shear deformation.
Tolerance A. Maximum-shear-stress theory B. Distorsion-energy theory C. Maximum-normal-stress
theory
# The difference between the maximum and minimum size limits of a part. D. Internal friction theory
A. Allowance B. Tolerance C. Deviation D. Basic size

# The stated maximum and minimum dimensions.


A. Tolerance B. Limits C. Nominal size D. Basic size

# A general term that refers to the mating of cylindrical parts such as bolt or a hole; it is used
only when the internal member is smaller that the external member.
A. Clearance B. Interference C. Allowance D. Tolerance

# The opposite of clearance, for mating cylindrical parts in which the internal member is
larger than the external member.
A. Clearance B. Allowance C. Tolerance D. Interference

# The minimum stated clearance or the maximum stated interference for mating parts.
A. Clearance B. Allowance C. Tolerance D. Interference

# The property of a material that measures the degree of plastic deformation sustained at
fracture.
A. Toughness B. Stiffness C. Ductility C. Brittleness

# Compounds of metallic elements, most frequently oxides, nitrides, and carbides.


A. Plastic B. Polymers C. Ceramics D. Alloy

# A material having different properties in all directions at point in solid.


A. Isotropic material B. Anisotropic material C. Orthropic material
D. Ceramic material

# A material having different properties in three mutually perpendicular directions at point in


solid and having three mutually perpendicular planes of material symmetry.
A. Orthotropic material B. Isotropic material C. Anistropic material
D. Thermoplastic material

# The combination of two or more materials, usually consisting of fiber and thermosetting
polymer.
A. Brittle materials B. Composite materials C. Polymers D. Ceramics

# A theorem stating that “when a body is elastically deformed by a system of loads, the
deflection at any point p in any direction a is equal to the partial derivatives of the strain energy
(with the system of loads acting) with respect to a load at p in the direction a”.
A. Poisson’s Theorem B. Newton’s Theorem C. Castigliano’s Theorem
D. Mohr’s Theorem

Potrebbero piacerti anche